Rheumatologie

अब Quizwiz के साथ अपने होमवर्क और परीक्षाओं को एस करें!

Rheuma 34 A 24-year-old woman with systemic lupus erythematosus is evaluated during a follow-up visit for a lupus flare. She was diagnosed with lupus 5 years ago. For the past year, her disease had been controlled with hydroxychloroquine and prednisone, 10 mg/d. Three months ago, she developed malar rash; oral ulcers; fatigue; pleuritis; and arthralgia involving the hands, wrists, and elbows. After her prednisone dosage was increased to 30 mg/d, her symptoms began to resolve, and the prednisone dosage was again tapered to 10 mg/d over a 6-week period. Three weeks ago, her symptoms recurred and her prednisone dosage was again increased to 30 mg/d. During an evaluation 2 weeks ago, she was asymptomatic. At that time, her prednisone dosage was decreased to 20 mg/d and azathioprine was added. She also takes calcium and vitamin D supplements. On physical examination, temperature is 36.4 °C (97.6 °F), blood pressure is 130/76 mm Hg, pulse rate is 76/min, and respiration rate is 16/min. She has cushingoid features. Cardiopulmonary examination is normal. She does not have oral ulcers, lymphadenopathy, or rash. Abdominal examination is unremarkable. Musculoskeletal examination reveals no synovitis. Laboratory studies: Hemoglobin 3 months ago 12.6 g/dL (126 g/L) 2 weeks ago 13.5 g/dL (135 g/L) Today 9.5 g/dL (95 g/L) Leukocyte count 3 months ago 3200/µL (3.2 × 109/L) 2 weeks ago 6300/µL (6.3 × 109/L) Today 1600/µL (1.6 × 109/L) Platelet count 3 months ago 176,000/µL (176 × 109/L) 2 weeks ago 245,000/µL (245 × 109/L) Today 135,000/µL (135 × 109/L) Complement (C3 and C4) 3 months ago Decreased 2 weeks ago Normal Today Normal Anti-double-stranded DNA antibodies 3 months ago Positive 2 weeks ago Negative Today Negative Which of the following is the most appropriate treatment for this patient? A Discontinue azathioprine B Discontinue hydroxychloroquine C Increase prednisone D Start intravenous immune globulin

A Cytopenia is a manifestation of systemic lupus erythematosus but also can be induced by a viral infection or the use of certain drugs, such as azathioprine. This patient has new-onset pancytopenia. This condition is a common manifestation of systemic lupus erythematosus (SLE) but also can be induced by a viral infection or the use of certain drugs. This patient developed pancytopenia 2 weeks after starting azathioprine, which suggests that azathioprine toxicity is the inciting agent. The most appropriate treatment for this patient is discontinuation of azathioprine. Azathioprine is a purine analog that acts as a cell cycle-specific antimetabolite. This agent is metabolized by the enzymes xanthine oxidase and thiopurine methyltransferase; low thiopurine methyltransferase activity or inhibition of xanthine oxidase by drugs such as allopurinol can cause an increase in serum levels of azathioprine and can result in azathioprine toxicity. This toxicity may manifest as myelosuppression. Most, but not all, patients with severe azathioprine-induced myelosuppression have low or absent thiopurine methyltransferase activity. Some experts recommend thiopurine methyltransferase activity measurement before beginning azathioprine therapy. Discontinuation of hydroxychloroquine is not indicated for this patient. This agent is not associated with cytopenias, and she has been using it for 5 years. Furthermore, discontinuing this agent would most likely worsen her disease activity. Although this patient has recently experienced a flare of her SLE, she is now asymptomatic, has normal complement levels, and does not have anti-double-stranded DNA antibodies. These features demonstrate that this patient's SLE is well controlled and that her condition has improved since her flare 3 months ago. Therefore, her pancytopenia is unlikely to be caused by her SLE, and an increase in her prednisone dosage or the addition of a new agent to help manage her SLE, such as intravenous immune globulin, would not be warranted.

Rheuma 30 A 25-year-old woman is evaluated during a follow-up visit for a 6-month history of diffuse muscle and joint pain above and below the waist, fatigue, and difficulty sleeping. She has a 2-year history of hypothyroidism treated with levothyroxine. Her only other medication is hydrocodone-acetaminophen, which has not relieved her pain. On physical examination, temperature is 37.0 °C (98.6 °F), blood pressure is 125/78 mm Hg, pulse rate is 85/min, and respiration rate is 12/min. Cardiopulmonary examination is normal. Musculoskeletal examination reveals diffuse periarticular tenderness, including bilateral tenderness in the biceps brachii, thighs, and calves. Muscle strength testing cannot be completed because of pain. The joints are not swollen, and she does not have lower-extremity edema. Laboratory studies: Complete blood count Normal Complete metabolic panel Normal Erythrocyte sedimentation rate 10 mm/h Creatine kinase 100 U/L Antinuclear antibodies Titer of 1:640 Thyroid-stimulating hormone 1.5 µU/mL (1.5 mU/L) Urinalysis Normal Which of following is the most likely diagnosis? A Fibromyalgia B Polymyositis C Sjögren syndrome D Systemic lupus erythematosus

A Fibromyalgia is characterized by diffuse pain on both sides of the body and above and below the waist as well as axial skeletal pain. This patient most likely has fibromyalgia. This condition is characterized by diffuse pain on both sides of the body and above and below the waist as well as axial skeletal pain, or, according to the original American College of Rheumatology criteria, the presence of pain in at least 11 of 18 specified potential tender points. However, expert opinion now states that these tender points are arbitrary and not essential in the diagnosis of fibromyalgia. Most patients with this condition have fatigue and sleep disturbance. Fibromyalgia also may be associated with dry eyes and mouth. Studies that have assessed the comorbidity of fibromyalgia with other symptom-defined syndromes have found high rates of chronic fatigue syndrome, migraine, irritable bowel syndrome, pelvic pain, and temporomandibular joint pain in patients with fibromyalgia. Polymyositis may manifest as muscle pain and fatigue but is unlikely in the absence of significant proximal muscle weakness or an elevated creatine kinase level. Up to 25% of patients with systemic inflammatory conditions, such as systemic lupus erythematosus (SLE) and rheumatoid arthritis, have symptoms consistent with fibromyalgia in the initial stages of their illness. This patient's fatigue, polyarthralgia, dry eyes and mouth, and strongly positive titers of antinuclear antibodies are consistent with SLE and Sjögren syndrome. However, patients with SLE usually have anemia, leukopenia, or lymphopenia. Similarly, joint involvement in Sjögren syndrome typically manifests as inflammatory arthritis. Furthermore, patients with SLE and Sjögren syndrome may have systemic manifestations, including cutaneous, neurologic, and renal involvement, which are absent in this patient. The presence of antinuclear antibodies is not diagnostic of SLE or Sjögren syndrome. These antibodies are often present in the general population and particularly in patients with autoimmune thyroid disease or in first-degree relatives of patients with SLE. In addition, high titers of antinuclear antibodies do not necessarily indicate the presence of autoimmune disease.

Rheuma 3 A 52-year-old man is evaluated in the emergency department for a 2-day history of acute pain and swelling in the left knee. He lives in Memphis, works in an office, and does not participate in outdoor recreational activities. There is no history of skin rash or trauma. He has type 2 diabetes mellitus. Medications are insulin glargine and insulin lispro. On physical examination, temperature is 38.0 °C (100.4 °F), blood pressure is 144/88 mm Hg, pulse rate is 88/min, and respiration rate is 18/min. The left knee is swollen and warm, has overlying erythema, and is tender to palpation. Range of motion of the left knee elicits pain and is limited. The remainder of the musculoskeletal examination is normal. Laboratory studies: Hemoglobin 10 g/dL (100 g/L) (normal indices) Leukocyte count 11,300/µL (11.3 × 109/L) (76% neutrophils) Erythrocyte sedimentation rate 78 mm/h Uric acid 8.2 mg/dL (0.48 mmol/L) Serum creatinine 2.0 mg/dL (152.6 µmol/L) Which of the following is the most appropriate next step in this patient's management? A Arthrocentesis B Prednisone and allopurinol C Radiography of the left knee D Serologic testing for Lyme disease

A Acute monoarticular arthritis should be presumed to be infectious until proven otherwise by synovial fluid analysis and culture via arthrocentesis. Acute monoarticular arthritis should be presumed to be infectious until proven otherwise by synovial fluid analysis and culture via arthrocentesis; the presence of a synovial fluid leukocyte count higher than 50,000/µL would strongly suggest an infectious process. The most appropriate next step in this patient's management is therefore arthrocentesis of the left knee. Because infectious arthritis is associated with significant morbidity and mortality, patients whose clinical presentation is suspicious for this condition should be treated immediately with empiric antibiotic therapy until culture results are available. This patient's diabetes mellitus places him at increased risk for infectious arthritis, and the presence of fever as well as erythema and swelling of the involved joint further raises suspicion for this condition. The differential diagnosis of acute monoarticular arthritis includes gouty arthritis, fracture, and Lyme disease. Allopurinol therapy would not be indicated for a first attack of gout or during an acute flare of gout. Also, crystal-induced arthritis and infectious arthritis may coexist, and infection must be excluded via synovial fluid analysis before beginning therapy for gout. In patients with long-standing gout, joint radiographs may reveal accumulated effects of inflammation, such as joint-space narrowing and destruction of the joint surface. Although highly suggestive of gout, radiographic abnormalities are not diagnostic of this condition and are usually absent in a first attack of acute gouty arthritis. Patients with acute calcium pyrophosphate deposition disease may have no radiographic abnormalities, and, conversely, chondrocalcinosis may be present in asymptomatic patients. In infectious arthritis, changes seen on joint radiographs may include bone damage but are relatively late findings. Early in the course of septic arthritis, soft-tissue fullness and joint effusions are often the only initial radiographic findings. Serologic testing for Lyme disease would not be the most appropriate next step in the management of a patient who does not have definite risk of exposure for this condition, such as living in an endemic area. Furthermore, arthritis typically is a late manifestation of Lyme disease and manifests subacutely. In patients with Lyme arthritis, swelling is usually more prominent than pain and significant erythema is rare.

Rheuma 35 A 28-year-old woman is hospitalized with a 7-week history of daily fever spikes to 38.0 °C and 39.0 °C (101.4 °F and 102.2 °F) followed by return to normal. She also has chest pain and pain in the hips, shoulders, proximal interphalangeal joints, wrists, and knees. She does not have dyspnea, nausea, vomiting, diarrhea, or rash. Her only current medication is ibuprofen, which has somewhat alleviated her joint pain. On physical examination, temperature is 37.1 °C (98.8 °F), blood pressure is 122/76 mm Hg, pulse rate is 78/min, and respiration rate is 14/min. Cardiac examination reveals a normal S1 and S2, and a three-component friction rub. The lungs are clear to auscultation. Shotty anterior cervical lymphadenopathy is present. There are no oral ulcers, and the oropharynx is clear. The wrists and proximal interphalangeal joints are swollen and tender bilaterally. Range of motion of the shoulders, hips, and knees elicits pain, but palpation of these joints reveals no appreciable swelling. The spleen tip is palpable, and there is no hepatomegaly. Laboratory studies: Hemoglobin 9.2 g/dL (92 g/L) Leukocyte count 14,500/µL (14.5 × 109/L) Platelet count 525,000/µL (525 × 109/L) Alanine aminotransferase 138 U/L Aspartate aminotransferase 105 U/L Ferritin 12,000 ng/mL (12,000 mg/L) Rheumatoid factor Negative Serum complement (C3 and C4) Normal Antinuclear antibodies Negative ANCA Negative Anti-double-stranded DNA antibodies Negative Urinalysis Normal A chest radiograph reveals an enlarged cardiac silhouette, no pulmonary infiltrates, and clear costophrenic angles. Blood cultures are negative. Which of the following is the most likely diagnosis? A Adult-onset Still disease B Relapsing polychondritis C Systemic lupus erythematosus D Wegener granulomatosis

A Adult-onset Still disease is an inflammatory disease characterized by polyarthritis and fever that spikes once or twice daily. This patient most likely has adult-onset Still disease (AOSD). AOSD is a clinical diagnosis established in patients who have five features cited in the diagnostic criteria for this condition, including at least two major criteria. The major criteria present in this patient are fever that spikes once or twice daily, polyarthritis, and leukocytosis. She also has lymphadenopathy, splenomegaly, and abnormal liver chemistry studies, which are among the minor criteria. Additional manifestations of AOSD may include cough, serositis manifested as pleuritic chest pain, and pericarditis. Patients with AOSD also may have normocytic, normochromic anemia; elevated inflammatory markers; a markedly elevated serum ferritin level that may exceed 2500 ng/mL (2500 mg/L); and an absence of antinuclear antibodies or rheumatoid factor. Affected children also often develop a salmon-colored rash during febrile episodes, whereas rash occurs less frequently in adults with this condition. AOSD is typically a diagnosis of exclusion; when establishing a diagnosis of this condition, excluding infection and a malignant etiology is therefore critical. Relapsing polychondritis is associated with inflammation of cartilaginous areas, including the trachea, airways, ears, and joints. This condition may manifest as polyarthritis; cough; fever; nasal congestion and rhinorrhea; and, in rare cases, pericarditis. Relapsing polychondritis is unlikely in the absence of auricular inflammation or inflammatory eye disease. This patient's fever, polyarthritis, pericarditis, lymphadenopathy, and splenomegaly are consistent with systemic lupus erythematosus (SLE). However, this patient does not have antinuclear antibodies, which are present in more than 99% of patients with SLE. Typical laboratory findings in patients with SLE also may include positive titers of anti-doubled-stranded DNA antibodies and hypocomplementemia, which are absent in this patient. Patients with Wegener granulomatosis may have polyarthritis; serositis; fever; and rhinorrhea and sinus pain. However, these findings typically would not be present in the absence of pulmonary and renal involvement. Wegener granulomatosis also is associated with positive ANCA titers, which this patient does not have. Wegener granulomatosis and relapsing polychondritis also would not explain this patient's lymphadenopathy and splenomegaly.

Rheuma 14 A 32-year-old woman is evaluated in the emergency department for a 4-day history of pain and swelling of the right wrist and low-grade fever. She has a 7-year history of severe rheumatoid arthritis. She does not recall any specific trauma involving the wrist but has recently been very physically active. Medications are methotrexate, a folic acid supplement, etanercept, prednisone, and ibuprofen. On physical examination, temperature is 37.8 °C (100.0 °F), blood pressure is 118/68 mm Hg, pulse rate is 90/min, and respiration rate is 18/min. BMI is 22. Cardiopulmonary examination is normal. There is no rash. The right wrist is swollen and tender and has a decreased range of motion. There are a subcutaneous nodule and small flexion deformity on the left elbow but no active synovitis. Mild synovitis is present on the second metacarpophalangeal joints bilaterally. The hips, knees, and feet are not tender or swollen and have full range of motion. Which of the following diagnostic studies of the wrist will be most helpful in establishing this patient's diagnosis? A Arthrocentesis B Arthroscopy C Bone scan D MRI E Radiography

A All patients who present with acute monoarthritis should be presumed to have septic arthritis until synovial fluid analysis via arthrocentesis excludes this condition. This patient most likely has septic arthritis, which usually manifests as acute monoarthritis and is characterized by pain on passive range of motion in the absence of known trauma. Arthrocentesis of the wrist will most likely help to establish a diagnosis in this patient. Septic arthritis should particularly be suspected in patients with underlying rheumatologic disorders such as rheumatoid arthritis who present with a sudden single joint flare that is not accompanied by other features of the pre-existing disorder. However, all patients who present with acute monoarthritis should be presumed to have septic arthritis until synovial fluid analysis via arthrocentesis excludes this condition. Synovial fluid analysis is the only definitive way to diagnose septic arthritis and is critical to guide antibiotic treatment. Patients with suspicion for this condition should begin empiric systemic antibiotic therapy until culture results are available. Surgical drainage or débridement via arthroscopy may be warranted in patients with septic arthritis who do not respond to repeated percutaneous drainage and appropriate antibiotic therapy but would not be an appropriate initial intervention. Joint and bone damage due to infection are relatively late radiographic findings. In acute septic arthritis, nonspecific soft-tissue fullness and joint effusions are often the only initial radiographic findings and do not establish the diagnosis of infection. Bone scans are more sensitive in detecting inflammatory lesions in bones and joints but also are not specific for infection. MRI of the affected joint is especially useful in detecting avascular necrosis, soft-tissue masses, and collections of fluid not visualized by other imaging modalities but would not establish the diagnosis of infection.

Rheuma 2 A 35-year-old woman with a 3-year history of systemic lupus erythematosus is admitted to the hospital with a blood pressure of 180/90 mm Hg and evidence of acute kidney injury. Her last lupus flare was 1 year ago, and she is currently asymptomatic. Five years ago, she developed deep venous thrombosis and pulmonary embolism after an automobile accident. She has had three first-trimester miscarriages. Her only medication is hydroxychloroquine. On physical examination, temperature is normal, blood pressure is 200/96 mm Hg, pulse rate is 102/min, and respiration rate is 20/min. Cardiopulmonary examination is normal except for an S4 gallop. Abdominal examination is unremarkable. There is no rash, lymphadenopathy, or oral ulcers. Laboratory studies: Hemoglobin 12.3 g/dL (123 g/L) Leukocyte count 5300/µL (5.3 × 109/L) Platelet count 122,000/µL (122 × 109/L) Reticulocyte count 1.9% of erythrocytes Serum creatinine 3.2 mg/dL (244.2 µmol/L) Serum complement (C3 and C4) Normal Antinuclear antibodies 1:1280 (speckled pattern) Anti-double-stranded DNA antibodies Negative IgM-specific anticardiolipin antibodies >100 U/mL IgG-specific anticardiolipin antibodies >100 U/mL Lupus anticoagulant Positive Urinalysis 2+ protein; 1+ blood; 2-3 leukocytes, 3-5 erythrocytes/hpf Urine protein-creatinine ratio 1.2 mg/mg A direct antiglobulin test (Coombs test) is negative. Peripheral blood smear reveals rare schistocytes. Renal ultrasonography reveals normal-sized kidneys with no obstruction or renal vein thrombosis. Renal biopsy shows capillary congestion and intracapillary fibrin thrombi consistent with thrombotic microangiopathy. Immunofluorescence testing reveals deposition of fibrin but not IgG, IgM, or C3. Which of the following is the most appropriate next step in this patient's treatment? A Heparin B Prednisone C Prednisone plus cyclophosphamide D Plasmapheresis plus fresh frozen plasma E Rituximab

A Anticoagulation therapy with heparin followed by warfarin is indicated in patients who have thrombotic microangiopathy associated with the antiphospholipid antibody syndrome. This patient has a 3-year history of systemic lupus erythematosus (SLE). Her history of deep venous thrombosis, pulmonary embolism, and three first-trimester miscarriages also is strongly suggestive of the antiphospholipid antibody syndrome (APS). This diagnosis is confirmed by the presence of anticardiolipin antibodies and a lupus anticoagulant. APS is characterized by the presence of at least one type of autoantibody known as an antiphospholipid antibody and at least one recognized clinical feature of this condition, which may include venous or arterial thromboses, recurrent fetal loss, or thrombocytopenia. This patient now presents with new-onset hypertension, proteinuria, renal failure, and a thrombotic microangiopathy seen on renal biopsy. These findings are most likely caused by her APS, and the most appropriate treatment is heparin. Although corticosteroids, plasmapheresis, and rituximab have been used in patients with catastrophic APS, anticoagulation is considered first-line treatment for APS-related microangiopathy. Determining whether acute kidney injury in patients with SLE who have antiphospholipid antibodies is associated with inflammation, which would be suggestive of SLE, or thrombosis, which would be suggestive of APS, is critical in order to initiate appropriate treatment. The presence of a thrombotic microangiopathy on renal biopsy in a patient with anticardiolipin antibodies and a lupus anticoagulant suggests that this patient's renal failure is a manifestation of APS and therefore thrombotic in origin. The therapy for APS is the same regardless of whether the disorder is primary or secondary to SLE. Anticoagulation therapy with heparin, followed by warfarin, is indicated in patients with thrombotic-associated manifestations of this condition. Prednisone and cyclophosphamide are used in the treatment of lupus nephritis. However, this patient currently has no symptoms of active SLE. In addition, her renal biopsy reveals no evidence of inflammation consistent with lupus nephritis, anti-double-stranded DNA antibody assay is negative, and her complement levels are normal. Plasmapheresis plus fresh frozen plasma is indicated to treat thrombotic thrombocytopenic purpura-hemolytic uremic syndrome (TTP-HUS), which may be associated with SLE. Rituximab also has been used to treat this condition. The presentation of TTP-HUS is similar to that of APS-associated thrombotic microangiopathy, and renal biopsy results in patients with TTP-HUS and APS-associated thrombotic microangiopathy can be indistinguishable. However, TTP-HUS would not explain the presence of antiphospholipid antibodies. In addition, patients with TTP-HUS often have fever, mental status changes, and severe thrombocytopenia. The microangiopathic hemolytic anemia associated with TTP-HUS is characterized by numerous schistocytes on peripheral blood smear and usually is associated with a high reticulocyte count, both of which are absent in this patient.

Rheuma 72 A 22-year-old woman is evaluated for pain and swelling of the left knee of 3 months' duration. She has pain with weight bearing, at rest, and during the night but no morning stiffness, warmth, or erythema. Four years ago, she was diagnosed with systemic lupus erythematosus. She also has a 2-year history of glomerulonephritis. She recently completed an extended course of mycophenolate mofetil and a tapering course of high-dose prednisone. Current medications are low-dose prednisone and hydroxychloroquine. On physical examination, temperature is normal, blood pressure is 128/72 mm Hg, pulse rate is 88/min, and respiration rate is 20/min. There is a large effusion on the left knee but no warmth or erythema. Range of motion of the left knee elicits pain. Synovial fluid leukocyte count is 800/µL (78% lymphocytes, 22% macrophages). Plain radiograph of the left knee shows no bony abnormalities. Which of the following is the most likely diagnosis? A Avascular necrosis B Bacterial septic arthritis C Crystal-induced arthritis D Fungal arthritis

A Avascular necrosis is associated with systemic lupus erythematosus and with long-term use of high doses of corticosteroids and may manifest as chronic monoarticular arthritis in the absence of signs and symptoms of inflammation. This patient most likely has avascular necrosis. This patient's clinical presentation of chronic monoarticular arthritis with pain on weight bearing and during the night in the absence of morning stiffness, warmth, erythema, or an inflammatory synovial fluid leukocyte count is consistent with this condition. Furthermore, avascular necrosis is associated with systemic lupus erythematosus and with long-term use of high doses of corticosteroids; a patient who has both of these risk factors is particularly likely to develop this condition. Both bacterial septic arthritis and crystal-induced arthritis (gout or calcium pyrophosphate deposition disease) may have monoarticular patterns of involvement but typically manifest acutely and would be associated with signs and symptoms of inflammation. In addition, the synovial fluid leukocyte count in a patient with either of these conditions would most likely be elevated and may exceed 50,000/µL in those with bacterial septic arthritis. In addition, crystal-induced arthritis would be associated with crystals in the synovial fluid and would be exceedingly uncommon in a premenopausal woman. Fungal arthritis commonly manifests as chronic monoarticular arthritis and often affects patients who are immunosuppressed, have a history of joint trauma, or travel to an endemic area. This condition may manifest as a subacute or chronic process and may be suspected when an organism is not identified in an inflammatory synovial fluid or when empiric antibiotic therapy does not cause a response. Suspicion for viral, fungal, or mycobacterial infection requires further specialized laboratory investigations to establish the diagnosis. However, fungal arthritis is unlikely in the absence of signs and symptoms of inflammation. In addition, the synovial fluid in a patient with fungal arthritis would most likely be inflammatory and have a predominance of neutrophils.

Rheuma 58 A 73-year-old man is evaluated during a follow-up visit for dermatomyositis. His condition was diagnosed 6 months ago. His serum creatine kinase level at that time was 3000 U/L. His disease responded well to prednisone, 60 mg/d, which was gradually tapered to 20 mg/d. He also takes azathioprine, 150 mg/d; alendronate; and calcium and vitamin D supplements. He mentions that his muscle weakness has increased over the past month but denies myalgia, tenderness, or side effects related to azathioprine. On physical examination, there are cushingoid facial changes. There is no rash. Muscle strength in the hip flexors and upper arms is 4/5. The muscles of the neck, back, upper arms, and legs are not tender to palpation. Laboratory studies reveal a leukocyte count of 9800/µL (9.8 × 109/L) and a serum creatine kinase level of 170 U/L. Which of the following is the most appropriate next step in this patient's management? A Decrease prednisone dosage B Increase azathioprine dosage C Substitute cyclosporine for azathioprine D Substitute methotrexate for azathioprine

A Corticosteroid-induced myopathy should be suspected in patients with polymyositis or dermatomyositis treated with corticosteroids who develop progressive weakness despite significant improvement in muscle enzyme levels. This patient most likely has corticosteroid-induced myopathy, and the most appropriate next step in his management is to decrease the prednisone dosage. He has progressive muscle weakness, but the significant decrease in his creatine kinase level suggests that his dermatomyositis is well controlled. Improvement in muscle strength typically is preceded by improvement in the creatine kinase level, but the discrepancy between this patient's findings on laboratory studies and symptoms is highly suggestive of a secondary cause for his weakness. Corticosteroid-induced myopathy should be suspected in patients with polymyositis or dermatomyositis treated with corticosteroids who develop progressive weakness despite significant improvement in muscle enzyme levels. This patient's cushingoid features also are consistent with this condition. The most appropriate management of a patient whose clinical presentation raises concern for corticosteroid-induced myopathy is to decrease the corticosteroid dosage and closely monitor the creatine kinase level for elevations. If corticosteroid-induced myopathy is causing this patient's symptoms, his muscle weakness should begin to resolve 3 to 4 weeks after his prednisone dosage is decreased. Increasing the azathioprine dosage would not be warranted in a patient with dermatomyositis whose creatine kinase level is not significantly elevated. Furthermore, this intervention would not directly address this patient's underlying corticosteroid-induced myopathy. Cyclosporine is an adjunct or substitute for methotrexate and azathioprine in patients in whom these agents are ineffective in the treatment of myositis or in those who cannot tolerate these agents. This patient is tolerating azathioprine, and there is therefore no need to add or substitute cyclosporine. Methotrexate is effective in the treatment of inflammatory myositis and is an alternative steroid-sparing drug to azathioprine. However, the patient's condition is responsive to his current medication regimen of corticosteroids and azathioprine, and substituting methotrexate is not likely to offer any advantage to a patient with corticosteroid-induced myopathy.

Rheuma 20 A 24-year-old woman is evaluated for a 1-year history of scaly plaques on her face and scalp. Results of a skin biopsy 1 week ago are consistent with chronic cutaneous lupus. She has ongoing mild fatigue, a 10-day history of lower back pain, and occasional migraine headaches for which she takes sumatriptan as needed. She also has a history of partial-onset seizures that began at age 13 years for which she currently takes phenytoin. On physical examination, temperature is 37.0 °C (98.6 °F), blood pressure is 125/73 mm Hg, pulse rate is 72/min, and respiration rate is 16/min. Cutaneous examination reveals scattered circular plaques on the cheeks, nose, scalp, and ear canals. Within the plaques, there is follicular plugging and atrophic scarring. She also has patches of alopecia where the rash is present on the scalp. Cardiopulmonary examination is normal. There is no lymphadenopathy or oral ulcers. Abdominal examination is unremarkable. Musculoskeletal examination reveals no synovitis, and neurologic examination is normal. Laboratory studies: Hemoglobin 14 g/dL (140 g/L) Leukocyte count 6300/µL (6.3 × 109/L) Erythrocyte sedimentation rate 16 mm/h Serum creatinine 0.8 mg/dL (61.0 µmol/L) Serum complement (C3 and C4) Normal Antinuclear antibodies Titer of 1:160 Anti-double-stranded DNA antibodies Negative Anti-Ro/SSA antibodies Negative Anti-La/SSB antibodies Negative Anti-Smith antibodies Negative Antiribonucleoprotein antibodies Negative Antihistone antibodies Negative Urinalysis Normal Which of the following is the most likely diagnosis? A Discoid lupus B Drug-induced lupus C Mixed connective tissue disease D Systemic lupus erythematosus

A Discoid lupus is characterized by the presence of red-to-pink round, oval, or polycyclic papules and plaques involving the face, scalp, and ear canals accompanied by follicular plugging and eventual development of atrophic scarring and hypopigmentation. This patient has discoid lupus, which is a form of chronic cutaneous lupus. Rash in this condition is characterized by sharply marginated, red-to-pink indurated papules and plaques. These lesions are round, oval, or polycyclic and have an adherent scale involving the face, scalp, and ear canals accompanied by follicular plugging. These papules and plaques expand peripherally with central clearing; with time, central atrophic scarring and hypopigmentation develop. Scarring on the scalp may cause patchy alopecia, which is irreversible. These lesions can last for years and may be slightly pruritic but are generally asymptomatic. Discoid lupus can exist as a primary condition in the absence of associated systemic disease; approximately 5% of patients with systemic lupus erythematosus (SLE) have discoid lupus, but only 10% of patients with discoid lupus develop SLE. Certain agents, including phenytoin, can cause drug-induced lupus. This condition usually manifests as fever, arthralgia, and serositis; antihistone antibodies also are universally present. This patient does not have any of these features. In addition, rash in patients with drug-induced lupus typically manifests as nonscarring scaly circular erythematous plaques that affect the neck, trunk, and extensor surfaces of the arms, which is not consistent with this patient's cutaneous findings. Mixed connective tissue disease is an overlap syndrome in which patients present with features of systemic sclerosis, myositis, and SLE. Diagnosis of this condition is established in patients with at least three of the following features: antiribonucleoprotein antibodies, Raynaud phenomenon, myositis, swelling of the hands, synovitis, and acrosclerosis. All of these features are absent in this patient. The presence of antinuclear antibodies is consistent with SLE, but patients with this condition typically have high titers of these antibodies. This patient also does not have anti-double-stranded DNA, anti-La/SSB, anti-Ro/SSA, anti-Smith, and antiribonucleoprotein antibodies, which are more specific for SLE than antinuclear antibodies. Fatigue, headache, and musculoskeletal involvement may occur in patients with SLE. However, fatigue is a nonspecific condition, migraine headaches are common in the general population, and lower back pain is not a musculoskeletal manifestation of SLE. Furthermore, this patient has no other features suggestive of SLE, such as oral ulcers, arthritis, serositis, renal disease, or hematologic abnormalities. Seizures may be associated with SLE but are more often caused by metabolic abnormalities, stroke, or infection. Furthermore, seizures due to SLE usually occur only in patients with acute, severely active disease, whereas this patient has a long history of seizures in the absence of serious systemic illness.

Rheuma 67 A 19-year-old female college student is evaluated in the emergency department for a 10-day history of fever and a 9-day history of migratory arthralgia and swelling of the joints. She initially developed pain and swelling in the right third and fourth proximal interphalangeal joints and the left second and third metacarpophalangeal joints, which resolved within 3 days. These symptoms then manifested in the right knee and also resolved within 3 days. She now has pain and swelling of the right wrist of 3 days' duration. Her last menstrual period was 12 days ago. Her only medication is an oral contraceptive pill. On physical examination, temperature is 38.0 °C (100.4 °F), blood pressure is 124/88 mm Hg, and pulse rate is 90/min. The oropharynx is clear. There is tenderness and puffiness of the dorsum of the right hand that extends to the distal forearm. The appearance of the right forearm is shown . The right wrist is tender, swollen, and has a decreased range of motion. The remainder of the physical examination is normal. Arthrocentesis is performed. The synovial fluid leukocyte count is 40,000/µL (90% polymorphonuclear cells). Polarized light microscopy of the fluid is negative for crystals, and Gram stain is negative. Culture results are pending. Which of the following is the most likely diagnosis? A Disseminated gonococcal infection B Nongonococcal septic arthritis C Parvovirus B19 infection D Rheumatic fever

A Disseminated gonococcal infection typically manifests as subacute migratory polyarthralgia and arthritis; tenosynovitis; and pustular, vesicopustular, or papular lesions. This patient most likely has disseminated gonococcal infection (DGI). Bacterial arthritis is classified as gonococcal or nongonococcal; gonococcal infection is associated with a better outcome. Gonococcal arthritis is the most common form of bacterial arthritis in young sexually active adults and is most likely to develop after menstruation in young sexually active women. DGI typically manifests as moderate fever and chills, subacute migratory polyarthralgia and arthritis, and tenosynovitis. Patients with this condition also may have tender pustular or vesiculopustular lesions on an erythematous base that usually develop on the distal extremities. DGI frequently involves the knees, hips, and wrists but not the spine. Synovial fluid analysis in patients with DGI usually reveals inflammatory synovial fluid, and microorganisms are rarely seen on Gram stain. Genitourinary symptoms associated with DGI usually are absent in women, and genital infection in women may have occurred long before systemic dissemination. Rectal and pharyngeal colonization of Neisseria gonorrhoeae in the setting of DGI is commonly asymptomatic. In all patients with clinical suspicion for DGI, routine culture of the rectum, cervix, and pharynx, as well as the blood and the joints, is indicated. More than 40% of patients with nongonococcal septic arthritis present without fever, and rigors infrequently occur in this setting. Nongonococcal septic arthritis usually manifests as acute monoarthritis and is not associated with pustular or vesiculopustular dermatitis. Gram staining of the synovial fluid is approximately 60% sensitive for bacteria in patients with nongonococcal septic arthritis. Synovial fluid culture is the gold standard for diagnosing nongonococcal joint infection in patients who have not undergone previous antibiotic therapy. Parvovirus B19 infection is a viral condition that commonly develops in children, and adults often acquire this condition from infected children. Affected adults usually present with nonspecific symptoms such as malaise, myalgia, and fever. Parvovirus B19 infection may manifest as acute symmetric polyarthritis involving the hands, wrists, knees, and feet that is indistinguishable from rheumatoid arthritis or systemic lupus erythematosus. This patient's migratory arthritis is not compatible with this diagnosis. Rheumatic fever is usually a disease of childhood but may present in adults as a migratory polyarthritis and fever. The knees, ankles, elbows, and wrists are affected most commonly. Arthritis might be the only initial symptom, but other manifestations may include carditis and chorea. Most importantly, patients with rheumatic fever do not have pustular skin lesions. Cutaneous findings in patients with rheumatic fever manifest as erythema marginatum, an evanescent, pink macular rash that affects the trunk and sometimes the limbs but not the face. This rash tends to expand centrifugally with clearing in the center.

Rheuma 48 A 50-year-old woman is evaluated for an 8-week history of diarrhea and abdominal pain. Stools are semiformed and greasy, and bowel movements seem to be stimulated by eating. She has not had nocturnal diarrhea. She also has lost 2.0 kg (4.4 lb) over the past month. She has an 8-year history of limited cutaneous systemic sclerosis associated with gastroesophageal reflux disease and Raynaud phenomenon. She has not traveled recently, used antibiotics, ingested raw or undercooked foods, or been exposed to anyone with a diarrheal illness. Current medications are amlodipine and omeprazole. On physical examination, temperature is 37.0 °C (98.6 °F), blood pressure is 120/80 mm Hg, pulse rate is 84/min, and respiration rate is 16/min. BMI is 22. Cardiopulmonary examination is normal. The abdomen is slightly distended and has mild diffuse tenderness to palpation; there is no organomegaly. Laboratory studies: Erythrocyte sedimentation rate 21 mm/h Complete blood count Normal Albumin 2.8 g/dL (28 g/L) Serum thyroid-stimulating hormone 4.1 µU/mL (4.1 mU/L) An abdominal radiographic series shows no free air under the diaphragm and a nonspecific bowel gas pattern. Which of the following is the most likely diagnosis? A Bacterial overgrowth B Clostridium difficile-associated colitis C Irritable bowel syndrome D Microscopic colitis

A Gastrointestinal involvement is common in both limited and diffuse cutaneous systemic sclerosis and may manifest as bacterial overgrowth. This patient most likely has small-bowel bacterial overgrowth. Gastrointestinal involvement is common in both limited and diffuse cutaneous systemic sclerosis, and dysfunctional motility in this setting may cause bacterial overgrowth. This patient's chronic diarrhea, abdominal pain, tender and distended abdomen, and hypoalbuminemia consistent with malabsorption raise suspicion for this condition. Although the gold standard is aspiration of duodenal luminal contents for quantitative culture at the time of upper endoscopy, this study is available at only a limited number of centers. A less invasive and more readily available screening test is the hydrogen breath test. Many clinicians simply recommend an empiric trial of antibiotics to evaluate whether the patient's symptoms improve. Clostridium difficile-associated colitis manifests as loose, watery stools; lower abdominal cramping; low-grade fever; and leukocytosis. Affected patients often have a history of antibiotic use or nosocomial exposure. Clostridium difficile-associated colitis is unlikely in a patient with symptoms suggestive of malabsorption and no recent hospitalization or antibiotic exposure. Irritable bowel syndrome is characterized by pain relieved with defecation. This condition initially manifests as a change in stool frequency or in the consistency of stool in the absence of hematochezia, unintentional weight loss, fever, or anemia. Irritable bowel syndrome could be responsible for this patient's chronic diarrhea and abdominal distention but would not explain the presence of greasy stools, weight loss, and hypoalbuminemia. Microscopic colitis is characterized by chronic diarrhea that is often accompanied by abdominal pain and mild weight loss. This condition most often affects otherwise well middle-aged to elderly patients. Microscopic colitis typically manifests as chronic or intermittent nonbloody watery stool. A syndrome of formed or semiformed greasy stools with evidence of malabsorption is not compatible with microscopic colitis.

Rheuma 64 A 55-year-old woman is evaluated for a 3-month history of fatigue, morning stiffness lasting for 1 hour, and decreased grip strength. She drinks two glasses of wine daily and is unwilling to stop. Her only medication is over-the-counter ibuprofen, 400 mg three times daily, which has helped to relieve her joint stiffness. On physical examination, vital signs are normal. Musculoskeletal examination reveals swelling of the metacarpophalangeal and proximal interphalangeal joints of the hands and decreased grip strength. There are effusions on both knees. The remainder of the physical examination is normal. Laboratory studies: Erythrocyte sedimentation rate 35 mm/h C-reactive protein Normal Rheumatoid factor Positive Antinuclear antibodies Positive Anti-cyclic citrullinated peptide antibodies Positive Alanine aminotransferase 25 U/L Aspartate aminotransferase 28 U/L Radiographs of the hands show soft-tissue swelling but no erosions or joint-space narrowing. Radiographs of the feet are normal. Which of the following is the most appropriate treatment for this patient? A Add hydroxychloroquine B Add methotrexate C Add subcutaneous etanercept D Increase ibuprofen dosage

A In patients with rheumatoid arthritis, disease-modifying antirheumatic drug therapy should be initiated as soon as the diagnosis is established. This patient has rheumatoid arthritis, and the most appropriate treatment for this patient is the addition of hydroxychloroquine, 400 mg/d. Prominent morning stiffness that usually lasts for more than 1 hour and fatigue are consistent with early presentations of rheumatoid arthritis. This condition most often involves the small joints of the hands and feet in a symmetric pattern, but involvement of the large joints also may occur. The presence of both rheumatoid factor and anti-cyclic citrullinated peptide antibodies is highly specific for rheumatoid arthritis, and radiographic manifestations of affected patients include periarticular osteopenia and, eventually, articular erosions. In patients with rheumatoid arthritis, early, aggressive disease control is critical and should be instituted as soon as the diagnosis is established. Experts recommend that affected patients begin disease-modifying antirheumatic drug (DMARD) therapy within 3 months of the onset of this condition. Hydroxychloroquine is warranted in a patient with early, mild, and nonerosive rheumatoid arthritis and is well tolerated. Methotrexate is often used as an initial DMARD in the treatment of rheumatoid arthritis. However, this agent is associated with hepatotoxicity, and risk for this condition is increased in patients who regularly consume alcoholic beverages; therefore, methotrexate is not indicated for these patients. The amount of alcohol that can safely be consumed in patients who use methotrexate has not yet been determined, but daily consumption of alcoholic beverages while using this agent is not recommended and most experts advise against the use of methotrexate for patients who regularly consume alcohol. The biologic DMARD etanercept would be an appropriate adjunct medication in a patient with rheumatoid arthritis in whom oral DMARD therapy has not provided adequate disease control. Etanercept and other tumor necrosis factor α inhibitors have greater efficacy when used in combination with methotrexate. However, there is currently insufficient evidence showing that single-agent use of a biologic DMARD is an appropriate initial treatment for this condition. Combination therapy with an NSAID and a DMARD has been shown to reduce joint pain and swelling in patients with rheumatoid arthritis. However, increasing this patient's ibuprofen dosage in the absence of DMARD therapy would not help to control her disease progression or prevent radiographic damage.

Rheuma 78 A 25-year-old man is evaluated in the emergency department for acute abdominal pain that began earlier today. For the past week, he has had progressive diffuse, crampy abdominal pain that is worse in the umbilical area and is exacerbated by food. Three weeks ago, he developed persistent low-grade fever, fatigue, malaise, nasal congestion, cough, and sore throat. For the past 2 weeks, he has had a rash on his legs. He also has a 1-week history of arthralgia. On physical examination, temperature is 37.8 °C (100.1 °F), blood pressure is 130/80 mm Hg, pulse rate is 96/min, and respiration rate is 18/min. Cardiopulmonary examination is normal. Palpable purpura is present on the legs. There is diffuse abdominal tenderness without rebound. No synovitis is present. A stool specimen is positive for occult blood. Laboratory studies: Hemoglobin 10.5 g/dL (105 g/L) Leukocyte count 12,400/µL (12.4 × 109/L) Platelet count 472,000/µL (472 × 109/L) Erythrocyte sedimentation rate 54 mm/h Serum creatinine 1.1 mg/dL (83.9 µmol/L) Blood urea nitrogen 25 mg/dL (8.9 mmol/L) Urinalysis 2+ protein; 3+ blood; 2-3 leukocytes, 5-10 erythrocytes/hpf; no casts Radiographs of the chest and abdomen are normal. Skin biopsy reveals a leukocytoclastic vasculitis. Immunofluorescence testing shows perivascular IgA deposition. Which of the following is the most appropriate initial treatment for this patient? A Prednisone B Prednisone plus cyclophosphamide C Prednisone plus cyclophosphamide and plasmapheresis D Prednisone plus infliximab E Prednisone plus rituximab

A Management of Henoch-Schönlein purpura may include short-term corticosteroids, which provide symptomatic relief but do not alter the course of the disease. This patient most likely has Henoch-Schönlein purpura (HSP), a typically self-limited condition that is characterized by purpuric rash, arthralgia, abdominal pain, and renal involvement that may manifest as hematuria and proteinuria. His skin biopsy shows leukocytoclastic vasculitis accompanied by perivascular IgA deposition, which is characteristic of HSP. The most appropriate management for this patient is short-term treatment with a corticosteroid such as prednisone in order to provide symptomatic relief. There is no evidence showing that infliximab, rituximab, cyclophosphamide, or plasmapheresis is beneficial in patients with HSP. Furthermore, no agent has yet been found that modifies the natural history of renal disease in HSP.

Rheuma 46 A 62-year-old man is evaluated in the emergency department for a 1-month history of nonproductive cough, progressive dyspnea on exertion, fever, arthritis, and weakness. He has had increasing difficulty rising from a chair, climbing stairs, and holding his arms up to comb his hair. For the past week, he has noticed pain and color changes of his digits when exposed to the cold. His medical history is otherwise unremarkable, and he takes no medications. On physical examination, temperature is 38.7 °C (101.6 °F), blood pressure is 148/88 mm Hg, pulse rate is 100/min, and respiration rate is 34/min. Pulse oximetry shows 92% oxygen saturation with the patient breathing 2 L/min of supplemental oxygen. There is no jugular venous distension. Cardiac examination reveals normal heart sounds without extra sounds, murmurs, or rubs. On pulmonary examination, late bilateral fine crackles are noted over the lower half of the lung fields. The sides of the fingers appear rough and cracked. He has tenderness and synovial thickening involving the wrists and the second and third metacarpophalangeal joints. Grip strength is intact, but he has evidence of weakness in the muscles of the upper arms and legs and neck flexors. Laboratory studies: Hemoglobin 13.8 g/dL (138 g/L) Leukocyte count 10,600/µL (10.6 × 109/L) Creatine kinase 5400 U/L Antinuclear antibodies Titer of 1:640 Anti-double-stranded DNA antibodies Negative Anti-Smith antibodies Negative Anti-Scl-70 antibodies Negative Anti-Jo-1 antibodies Positive Chest radiograph is shown . Which of the following is the most likely diagnosis? A Antisynthetase syndrome B Idiopathic pulmonary fibrosis C Systemic lupus erythematosus D Systemic sclerosis

A Nearly one third of patients with inflammatory myopathy have the antisynthetase syndrome, which is characterized by an acute or subacute onset, fever, fatigue, Raynaud phenomenon, "mechanic's hands," synovitis, and interstitial lung disease. This patient most likely has the antisynthetase syndrome. Polymyositis and dermatomyositis are inflammatory myopathies that typically present with the gradual onset of symmetric proximal weakness over weeks to months. This weakness causes progressive difficulty with rising from a chair, climbing stairs, or hair combing. Nearly one third of patients with an inflammatory myopathy have the antisynthetase syndrome. This condition can have an acute or subacute onset and is characterized by fever; fatigue; Raynaud phenomenon; synovitis; interstitial lung disease; and scaly, rough, dry, darkened, cracked horizontal lines that develop on the palmar and lateral aspects of the fingers that are known as "mechanic's hands." Patients with antisynthetase syndrome have antisynthetase antibodies that are specific for an inflammatory myopathy. The most common antisynthetase antibody is the anti-Jo-1 antibody. Not all patients with anti-Jo-1 antibodies develop the antisynthetase syndrome and not all patients with this syndrome have all of its manifestations, but patients who have anti-Jo-1 antibodies typically have several features of the antisynthetase syndrome in addition to their muscle disease. The greatest concern in patients with this syndrome is the risk for developing interstitial lung disease. Idiopathic pulmonary fibrosis (IPF), the most common of the idiopathic interstitial pneumonias, is a fibrosing interstitial pneumonia. This condition manifests as slowly progressive dyspnea and a chronic, nonproductive cough. Pulmonary examination in patients with IPF reveals end-inspiratory crackles, and chest radiograph performed on presentation nearly always reveals decreased lung volumes and basal reticular opacities. However, IPF is not associated with extrapulmonary involvement. Patients with systemic lupus erythematosus (SLE) may have fever, arthralgia or arthritis, Raynaud phenomenon, and very rarely, an acute interstitial pneumonia (lupus pneumonitis). However, SLE usually affects women in their 20s and 30s and is often associated with additional manifestations such as serositis, aphthous ulcers, malar or photosensitive rash, and specific autoantibodies including anti-double-stranded DNA and anti-Smith antibodies. SLE is an unlikely diagnosis based upon the extent of muscle weakness, degree of elevation of the creatine kinase level, absence of specific antibodies, and presence of anti-Jo-1 antibodies. Systemic sclerosis most frequently develops in patients between 30 and 50 years of age and has a 3:1 female predominance. Diagnosis may be established in patients who present with two of the following features: sclerodactyly, digital pitting, and basilar fibrosis visible on chest radiography. Antinuclear antibodies are present in more than 95% of patients with systemic sclerosis. Patients with anti-Scl-70 antibodies are at increased risk for developing diffuse cutaneous disease and interstitial lung disease. Systemic sclerosis is unlikely in the absence of sclerodactyly or digital pitting. Furthermore, this condition would not explain this patient's proximal muscle weakness.

Rheuma 11 A 45-year-old woman is hospitalized for shortness of breath of 2 days' duration. For the past 2 weeks, she has had ankle edema and a low-grade fever and cough productive of white sputum for which she began taking azithromycin 1 week ago. She has a 15-year history of allergic rhinitis and asthma treated with albuterol and fluticasone-salmeterol inhalers. Two months ago, her asthma worsened despite an increased fluticasone-salmeterol dosage and the addition of montelukast to her regimen. She also takes mometasone furoate aqueous nasal spray. On physical examination, temperature is 37.7 °C (99.9 °F), blood pressure is 120/65 mm Hg, pulse rate is 112/min, and respiration rate is 32/min. Cardiopulmonary examination reveals tachycardia, an S3 gallop, diffuse crackles, and wheezes. There is no rash, lymphadenopathy, splenomegaly, or synovitis, but there is 2+ edema of the ankles. Laboratory studies: Hemoglobin 10.3 g/dL (103 g/L) Leukocyte count 16,000/µL (16 × 109/L) (21% eosinophils; no immature forms) Platelet count 605,000/µL (605 × 109/L) Serum creatinine 1.4 mg/dL (106.8 µmol/L) Liver enzyme studies Normal Antinuclear antibodies Negative p-ANCA Positive Antimyeloperoxidase antibodies Positive Urinalysis 2+ protein; 3+ blood; 20-30 leukocytes, 15-20 erythrocytes/hpf; rare erythrocyte casts Urine protein-creatinine ratio 1.3 mg/mg Chest radiograph reveals cardiomegaly; small bilateral effusions; and pulmonary infiltrates in the right lower lobe, right middle lobe, and left lower lobe. Echocardiography shows an ejection fraction of 35%, diffuse hypokinesis, and no valvular or wall motion abnormalities. Bone marrow biopsy reveals 8% eosinophils and no abnormal cells. In addition to methylprednisolone, which of the following is the most appropriate treatment for this patient? A Cyclophosphamide B Hydroxyurea C Imatinib D No additional therapy

A Patients with Churg-Strauss syndrome who have cardiac, gastrointestinal, renal, or central nervous system involvement should be treated with cyclophosphamide in addition to high-dose corticosteroids. This patient has a long history of allergic rhinitis and asthma that has worsened over the past 2 months. She now presents with a multisystem illness characterized by fever, eosinophilia, pulmonary infiltrates, cardiomyopathy, and positive antimyeloperoxidase antibody titers. In addition, the presence of erythrocyte casts on urinalysis and her hypertension, hematuria, proteinuria, and elevated serum creatinine level are consistent with glomerulonephritis. This clinical presentation is strongly suggestive of Churg-Strauss syndrome, and the most appropriate management for this patient is methylprednisolone plus cyclophosphamide. The Churg-Strauss syndrome is a multisystem disorder characterized by the presence of allergic rhinitis, asthma, and eosinophilia. This condition has a mean age of diagnosis of 48 years, and the most commonly involved organ in affected patients is the lung followed by the skin. However, involvement of any organ may occur in patients with this condition, including the cardiovascular, gastrointestinal, renal, and central nervous systems. High-dose corticosteroids are indicated to treat this condition; affected patients with cardiac, gastrointestinal, renal, or central nervous system involvement also should receive cyclophosphamide. Hydroxyurea, imatinib, and bone marrow transplantation are used to manage hypereosinophilic syndromes, and imatinib is particularly effective in patients with platelet-derived growth factor receptor-β rearrangements. Hypereosinophilic syndromes can present with eosinophilia and multisystem disease but would not cause positive ANCA titers. The absence of immature cells on peripheral blood smear or splenomegaly and her relatively low levels of bone marrow hypereosinophilia also argue against a clonal hypereosinophilic syndrome.

Rheuma 79 A 63-year-old woman is evaluated during a follow-up visit for a 4-week history of fatigue; pain in the proximal interphalangeal joints, knees, and hips; and low-grade fever. She has not had joint swelling, chest pain, or shortness of breath. Over the past 4 years, she has had progressive dryness of the eyes and mouth. She has a 5-month history of Raynaud phenomenon, which has been less symptomatic since beginning nifedipine 4 months ago. On physical examination, temperature is 38.2 °C (100.8 °F), blood pressure is 125/72 mm Hg, pulse rate is 74/min, and respiration rate is 18/min. Cardiac examination is normal, and the lungs are clear. She has bilateral parotid gland enlargement, a firm 4-cm left axillary lymph node, and a shotty 0.3-cm left anterior cervical lymph node. Musculoskeletal examination reveals bilateral crepitus of the knees. There is no joint swelling. Laboratory studies: Hemoglobin 11.6 g/dL (116 g/L) Leukocyte count 3400/µL (3.4 × 109/L) Platelet count 120,000/µL (120 × 109/L) Rheumatoid factor 76 U/mL (76 kU/L) Antinuclear antibodies Positive Anti-Ro/SSA antibodies Positive Anti-La/SSB antibodies Positive Urinalysis Normal Blood cultures No growth A chest radiograph and mammogram are normal. Which of the following is the next best step in this patient's management? A Excisional axillary lymph node biopsy B Minor salivary gland biopsy C Prednisone D Transthoracic echocardiography

A Patients with primary Sjögren syndrome have up to a 44-fold increased incidence of lymphoma and should be monitored closely for lymphadenopathy. This patient's symptoms of keratoconjunctivitis sicca (dry eyes and dry mouth); the presence of antinuclear, anti-Ro/SSA, and anti-La/SSB antibodies; and an elevated rheumatoid factor level are characteristic of primary Sjögren syndrome. Her fatigue, arthralgia, Raynaud phenomenon, and low-grade fever also are consistent with this condition. Sjögren syndrome also may occur secondary to another autoimmune disease, such as rheumatoid arthritis or systemic lupus erythematosus. Patients with primary Sjögren syndrome have up to a 44-fold increased incidence of lymphoma, particularly non-Hodgkin lymphoma, and should be monitored closely for lymphadenopathy. This patient's firm enlarged left axillary lymph node and pancytopenia particularly raise suspicion of lymphoma, and an excisional axillary lymph node biopsy is the most appropriate next step in this patient's management. A minor (labial) salivary gland biopsy would help to confirm the diagnosis of Sjögren syndrome and would reveal increased lymphocytic infiltration. Because this patient has clinical evidence of Sjögren syndrome, salivary gland biopsy is not necessary; furthermore, this study would not help to evaluate this patient's enlarged axillary lymph node. Prednisone may help to treat constitutional symptoms associated with Sjögren syndrome but would not address this patient's more urgent enlarged left axillary lymph node. In addition, her constitutional symptoms may be manifestations of malignancy and not Sjögren syndrome. Infective endocarditis may manifest as fever, fatigue, and lymphadenopathy, and transthoracic echocardiography can help to diagnose this condition. However, infective endocarditis is unlikely in a patient with no cardiac abnormalities; negative blood culture results; and an absence of associated vascular phenomena or risk factors for this condition. Therefore, transthoracic echocardiography would not be warranted in this patient.

Rheuma 73 A 48-year-old woman is evaluated for a 4-month history of pain in the shoulders, neck, and lower back. She also has fatigue and difficulty sleeping. She does not have rash, chest pain, joint pain, anorexia, weight loss, fever, or depressed mood. On physical examination, vital signs are normal. Thyroid examination is normal. Musculoskeletal examination reveals widespread soft-tissue pain. The joints are not swollen. Muscle strength testing is limited because of pain. Deep-tendon reflexes and sensation are intact. Laboratory studies: Complete blood count Normal Erythrocyte sedimentation rate 18 mm/h Serum creatine kinase 164 U/L Serum thyroid-stimulating hormone 3.0 µU/mL (3.0 mU/L) Which of the following is the most appropriate next step in this patient's management? A Aerobic exercise program B Antinuclear antibody assay C Electromyography D Ibuprofen E Prednisone

A Pregabalin and duloxetine are approved for the management of fibromyalgia. This patient most likely has fibromyalgia, which typically affects women and most commonly occurs in patients between 40 and 60 years of age. The most appropriate next step in this patient's management is participation in a graded low-impact aerobic exercise program. Fibromyalgia is characterized by chronic widespread musculoskeletal pain above and below the waist of at least 3 months' duration. Additional manifestations of fibromyalgia include fatigue and difficulty sleeping. This patient does not have constitutional symptoms or inflamed joints, which would suggest a systemic inflammatory condition. The most appropriate initial management for fibromyalgia is regular participation in a graded low-impact aerobic exercise program. This intervention has been shown to benefit physical capacity in affected patients and to control the symptoms of this condition. Aerobic exercise is safe for patients with fibromyalgia but should be increased slowly, aiming for moderate intensity (at least 20 minutes per day 2 to 3 days weekly). If symptoms worsen, patients should decrease exercise until symptoms improve. The only approved agents for fibromyalgia are pregabalin and duloxetine, which have been shown to help alleviate pain and fatigue and improve sleep quality. Off-label use of antidepressant drugs even in the absence of depressive symptoms may be helpful for mood, pain, and sleep symptoms associated with this condition but have limited effect on overall health outcomes. An antinuclear antibody assay would be appropriate in a patient whose clinical presentation is suggestive of systemic lupus erythematosus. This condition is associated with polyarthralgias and fatigue but is unlikely in a patient who has no additional features associated with lupus, such as serositis, rash, and mucosal ulcerations. Electromyography would be warranted in a patient whose clinical presentation raises suspicion for polymyositis. This condition affects the proximal and truncal muscles and may involve the neck but usually manifests as muscle weakness; if present, pain in this setting is usually mild. Patients with polymyositis also usually have elevated muscle enzyme levels, which are absent in this patient. There are no data suggesting that off-label use of NSAIDs alone is effective in the treatment of fibromyalgia, but these agents may be beneficial in treating fibromyalgia when combined with amitriptyline (also off-label use). Prednisone would be indicated for a patient with polymyalgia rheumatica, which may manifest as chronic pain involving the neck and shoulders and may be associated with fatigue. However, polymyalgia rheumatica exclusively affects patients over 50 years of age and is almost always associated with an elevated erythrocyte sedimentation rate. In addition, polymyalgia rheumatica would not explain this patient's lower back symptoms. Corticosteroids have not been shown to be effective in fibromyalgia.

Rheuma 36 A 24-year-old woman is evaluated in the emergency department for nausea, vomiting, and fatigue of 4 days' duration. She has an 18-month history of diffuse cutaneous systemic sclerosis complicated by gastroesophageal reflux disease and Raynaud phenomenon. She also has a 12-year history of asthma. Two weeks ago, she was evaluated in the emergency department for an asthma flare; at that time, her blood pressure was 100/70 mm Hg and her complete blood count, renal function studies, and urinalysis were normal. She began a course of prednisone, 40 mg/d, for 10 days. Additional medications are inhaled fluticasone-salmeterol, amlodipine, omeprazole, and inhaled albuterol as needed. On physical examination, temperature is 37.0 °C (98.6 °F), blood pressure is 130/75 mm Hg, pulse rate is 78/min, and respiration rate is 16/min. Cardiopulmonary examination is normal except for several diffuse expiratory wheezes. There is skin thickening over the face, chest, arms, hands, and legs. There is 1+ edema of the lower extremities bilaterally. Laboratory studies: Hemoglobin 9.8 g/dL (98 g/L) Platelet count 100,000/µL (100 × 109/L) Blood urea nitrogen 92 mg/dL (32.8 mmol/L) Serum creatinine 5.7 mg/dL (434.9 µmol/L) Urinalysis 2+ protein; 3-5 erythrocytes/hpf; no casts A peripheral blood smear shows schistocytes. The addition of which of the following is the most appropriate management option for this patient? A Captopril B Cyclosporine C Plasma exchange D Prednisone

A Prompt initiation of therapy with an angiotensin-converting enzyme inhibitor such as captopril, regardless of the serum creatinine concentration, is the treatment of choice for scleroderma renal crisis. This patient most likely has scleroderma renal crisis (SRC), and the most appropriate management for this patient is to begin captopril. SRC is a complication of systemic sclerosis that most commonly affects patients with diffuse cutaneous disease within the first 48 months after diagnosis. The use of corticosteroids (usually more than 15 mg/d) in the preceding 6 months increases the risk for SRC, and particularly normotensive renal crisis, in patients with diffuse cutaneous systemic sclerosis. SRC classically is characterized by the acute onset of severe hypertension, renal failure, and microangiopathic hemolytic anemia. Patients who develop this condition as a result of corticosteroid use often are normotensive but may have blood pressures higher than their normal baseline levels. Prompt initiation of therapy with an angiotensin-converting enzyme inhibitor such as captopril is the treatment of choice for SRC. This therapy should be continued even in patients with significant renal insufficiency, as renal function has been shown to improve even after months of dialysis. Before the advent of these agents, mortality was an almost universal outcome in patients with SRC. Case reports suggest that cyclosporine may worsen renal disease in patients with systemic sclerosis. Therefore, cyclosporine, a renal vasoconstrictor, is not the most appropriate therapeutic agent for SRC, which is characterized by narrowing and obliteration of the vascular lumen of the renal arteries. Thrombotic thrombocytopenic purpura and the hemolytic uremic syndrome (TTP-HUS) are thrombotic disorders characterized by microangiopathic hemolytic anemia and thrombocytopenia. Other manifestations may include neurologic abnormalities (headache, mental status changes, seizures), renal dysfunction, and fever. Distinguishing TTP-HUS from SRC can be difficult but is critical. Plasma exchange often benefits patients with TTP-HUS but not those with SRC and may delay appropriate therapy. However, this patient developed the abrupt onset of renal failure and has a recent diagnosis of diffuse sclerosis and exposure to high-dose corticosteroid therapy; this clinical presentation is therefore more characteristic of SRC, and plasma exchange is not indicated. Corticosteroid use is associated with an increase in risk for SRC and, when used, should be administered at the lowest possible effective dose for the shortest period of time compatible with resolution of the treated condition. Corticosteroid use is not indicated as a treatment for normotensive SRC.

Rheuma 7 An 82-year-old woman with a 2-year history of osteoarthritis of the knees is evaluated for persistent swelling and pain in the right knee of 3 months' duration. She now uses a cane for ambulation and is unable to go grocery shopping. Medications are naproxen and hydrocodone-acetaminophen as needed. On physical examination, vital signs are normal. The right knee has a large effusion and a valgus deformity. There is decreased flexion of the right knee secondary to pain and stiffness, and she is unable to fully extend this joint. Range of motion of both knees elicits coarse crepitus. Laboratory studies reveal a serum creatinine level of 1.1 mg/dL (83.9 µmol/L) and a serum uric acid level of 8.2 mg/dL (0.48 mmol/L). Radiograph of the right knee reveals a large effusion and changes consistent with end-stage osteoarthritis. Aspiration of the right knee is performed. Synovial fluid leukocyte count is 3200/µL. Polarized light microscopy of the fluid demonstrates rhomboid-shaped weakly positively birefringent crystals. Results of Gram stain and cultures are pending. Which of the following is the most likely diagnosis? A Calcium pyrophosphate dihydrate deposition disease B Chronic apatite deposition disease C Gout D Septic arthritis

A Pseudogout is associated with acute or subacute attacks of warmth and swelling in one to two joints that resemble acute gouty arthropathy and weakly positive birefringent crystals that are rhomboid in shape seen on polarized light microscopy of synovial fluid. This patient has calcium pyrophosphate dihydrate (CPPD) deposition disease presenting as pseudogout. Pseudogout manifests as acute or subacute attacks of warmth and swelling in one to two joints that resemble acute gouty arthropathy. Pseudogout is associated with inflammatory synovial fluid and the presence of CPPD crystals that are weakly positively birefringent and rhomboid in shape seen on polarized light microscopy. Treatment of an acute pseudogout attack primarily involves NSAIDs, but a corticosteroid or colchicine would be appropriate alternative choices. This patient's radiographic and physical examination findings also are suggestive of osteoarthritis. Osteoarthritis that manifests in patients with CPPD deposition or the presence of chondrocalcinosis on radiography is known as pseudo-osteoarthritis. This degenerative condition mimics osteoarthritis except that it may affect joints not typically involved in osteoarthritis, such as the wrists, metacarpophalangeal joints, shoulders, and ankles. The synovial fluid in patients with pseudo-osteoarthritis is noninflammatory. Both pseudo-osteoarthritis and pseudogout may be present in the same patient. The treatment of pseudo-osteoarthritis is no different than the treatment of osteoarthritis and includes adequate analgesia, physical and occupational therapy, and arthroplasty for symptomatic disease unresponsive to more conservative therapy. Characteristic features of chronic apatite deposition disease include large, minimally inflammatory effusions that usually develop in the shoulder or knee, destruction of associated tendon structures, and chronic pain. Calcium apatite crystals may only appear as amorphous nonbirefringent crystalline clumps on synovial fluid analysis and therefore are not identified on routine examination. Identification of these crystals requires special staining or crystal analysis that is not routinely available. The absence of these crystals in this patient excludes this condition. Gout is caused by the deposition of monosodium urate crystals in the tissues of and around the joints. Early attacks of gout are monoarticular and most commonly involve the first metatarsophalangeal joint, whereas chronic gout may manifest as symmetric involvement of the small joints of the hands and feet accompanied by tophi and subcortical erosions on radiography. Definitive diagnosis of gout is established by the presence of strongly negatively birefringent needle-shaped crystals on polarized light microscopy of synovial fluid or fluid from a tophus, which is not consistent with this patient's findings. The diagnosis of septic arthritis should be considered in all patients with acute monoarthritis and a sudden increase in pain in a chronically damaged joint. This patient's joint fluid is inflammatory, but the leukocyte count is not sufficiently elevated to suggest septic arthritis.

Rheuma 60 A 23-year-old man is evaluated in the emergency department for a 5-day history of headache, blurred vision, and right eye pain. His eye pain increases when he attempts to read or when exposed to light. He also has a 3-year history of back stiffness that is worse in the morning and tends to improve as he becomes more active. He does not have arthralgia, arthritis, or rash. He takes no medications and is monogamous. On physical examination, temperature is 36.8 °C (98.2 °F), blood pressure is 130/76 mm Hg, pulse rate is 85/min, and respiration rate is 14/min. There are no skin lesions. The appearance of the right eye is shown . Photophobia is present during the penlight examination of the pupil. Both pupils react to light. An emergency referral is made to an ophthalmologist. Following resolution of the eye problem, this patient should be evaluated for which of the following systemic diseases? A Ankylosing spondylitis B Sarcoidosis C Sjögren syndrome D Systemic lupus erythematosus

A The most commonly diagnosed systemic illnesses in patients with anterior uveitis are reactive arthritis, ankylosing spondylitis, and sarcoidosis. The patient has anterior uveitis with a hypopyon, and the associated systemic disease is most likely ankylosing spondylitis. The classic triad for acute anterior uveitis is pain, sensitivity to light, and blurred vision; headache, tenderness, and tearing may also occur. Photophobia during penlight examination has a positive predictive value of 60% for severe eye disease and a negative predictive value of 90%. Prospective studies have documented systemic illness in 53% of patients with anterior uveitis. Patients with uveitis associated with systemic disease usually have a history or physical examination findings that suggest an underlying disorder. The most commonly diagnosed systemic illnesses in this setting are reactive arthritis, ankylosing spondylitis, and sarcoidosis. Acute anterior uveitis, particularly unilateral presentations that fluctuate between both eyes over time, is strongly associated with the HLA-B27-related arthropathies, including ankylosing spondylitis. In addition, this patient's chronic back stiffness is highly suggestive of ankylosing spondylitis. Furthermore, in up to 65% of patients with uveitis, spondyloarthropathy remains undiagnosed until these patients present with uveitis. Posterior uveitis may be related to sarcoidosis or vasculitis but is not typically associated with pain or redness of the eye. Patients with posterior uveitis also often have decreased visual acuity and floaters, which is not consistent with this patient's presentation. Furthermore, sarcoidosis is an unlikely cause of this patient's chronic low back pain. Sicca syndrome manifests as dryness of the mouth, eyes, and vagina and variable enlargement of the parotid glands in association with concomitant redness and gritty irritation of the eyes. This condition is suggestive of primary or secondary Sjögren syndrome. However, Sjögren syndrome would not cause anterior uveitis and also would not explain the presence of chronic low back pain in a young man. Anterior uveitis is associated with psoriasis and, in rare cases, Whipple disease, systemic lupus erythematosus, and the systemic vasculitides. However, the patient's long history of back pain in the absence of cutaneous and other manifestations of systemic lupus erythematosus makes this diagnosis unlikely.

Rheuma 15 A 68-year-old woman is admitted to the hospital for an 8-month history of progressive proximal muscle weakness. Muscle biopsy of the right quadriceps performed when symptoms first began was consistent with dermatomyositis. At the time of diagnosis, chest radiograph, breast examination, mammography, pelvic examination, Pap smear, and colonoscopy were normal. High-dose prednisone accompanied by alendronate and calcium and vitamin D supplements was begun 6 months ago but has not helped to alleviate her symptoms. She is now unable to bathe or dress herself, requires assistance rising from a chair, and uses a wheelchair because of frequent falls. She also occasionally chokes when eating. Since diagnosis of her condition, the serum creatine kinase level has been between 1000 and 2000 U/L. She has no additional symptoms. On physical examination, she appears frail with facial fullness. Vital signs are normal. BMI is 21. Cardiopulmonary examination is unremarkable. Cutaneous examination reveals a V-shaped erythematous macular eruption on the anterior chest. On musculoskeletal examination, she is unable to raise the upper arms or thighs against gravity, and the distal muscles are weak against resistance. Laboratory studies: Hemoglobin 10.8 g/dL (108 g/L) Erythrocyte sedimentation rate 120 mm/h Creatine kinase 3845 U/L Which of the following should be done next? A CT of the chest, abdomen, and pelvis B Discontinue prednisone C MRI of the thigh muscles D Repeat muscle biopsy

A The risk for malignant disease is increased in patients with dermatomyositis, and minimum evaluation for these patients should include age- and sex-appropriate screening tests. Despite high-dose corticosteroid therapy, this patient has active dermatomyositis manifested by progressive weakness, a persistently elevated creatine kinase level, and a characteristic V sign on the anterior chest on cutaneous examination. CT of the chest, abdomen, and pelvis is indicated for this patient. Evaluation for an occult malignancy is warranted in patients with dermatomyositis who are refractory to treatment, particularly those who are older than 60 years of age. The types of malignancies associated with the inflammatory myopathies have been shown to be similar to those in an age-matched population except that women with these conditions have a higher rate of ovarian cancer. Malignancy may predate or develop after myositis, but these conditions most often occur concomitantly. In some patients, symptoms of myositis are decreased or remit when the malignancy is resolved. This patient received the minimally appropriate evaluation for malignancy at the time of diagnosis, which consisted of a chest radiograph and sex- and age-appropriate screening. Although many experts recommend more extensive screening at the time of diagnosis, there are no data from controlled studies to support this practice. Therefore, decisions regarding screening must be guided by the patient's risk factors and symptoms and should be determined on an individual basis. MRI of the thigh muscles would most likely reveal ongoing active muscle inflammation consistent with this patient's persistently elevated creatine kinase level but would not further explain her refractory disease or help in the management. Repeat muscle biopsy to diagnose inclusion body myositis is not indicated for this patient. Inclusion body myositis is known to respond poorly to corticosteroid therapy, but this condition characteristically manifests as slowly progressive asymmetric proximal and distal muscle weakness that develops over many years and is not associated with the classic V-neck rash seen in this patient. Furthermore, serum creatine kinase levels in patients with inclusion body myositis are typically less than 1000 U/L. Worsening proximal muscle weakness despite corticosteroid therapy also may be caused by corticosteroid-induced myopathy. However, patients with this condition experience a decrease in the creatine kinase level; this patient's persistently elevated creatine kinase level excludes this diagnosis, and stopping the corticosteroid therapy is therefore not indicated.

Rheuma 54 A 27-year-old woman in the twelfth week of pregnancy is admitted to the hospital for hypertension and proteinuria. She has a 2-year history of diffuse cutaneous systemic sclerosis associated with gastroesophageal reflux disease and Raynaud phenomenon. She had been taking omeprazole but discontinued this agent when she became pregnant. Her only current medications are over-the-counter calcium carbonate and prenatal vitamins. On physical examination, temperature is 37.0 °C (98.6 °F), blood pressure is 194/104 mm Hg, pulse rate is 84/min, and respiration rate is 17/min. Cardiopulmonary examination is normal except for an S4 gallop. There is skin thickening over the face, chest, abdomen, arms, hands, and feet. Funduscopic examination is normal. Abdominal examination reveals a normal for pregnancy gravid uterus. Tendon friction rubs are present over the anterior lower extremities. Laboratory studies: Hemoglobin g/dL (74 g/L) Leukocyte count 8000/µL (8 × 109/L) Platelet count 66,000/µL (66 × 109/L) Serum creatinine 2.4 mg/dL (183.1 µmol/L) Urinalysis 3+ protein; no leukocytes or erythrocytes/hpf Alanine aminotransferase 35 U/L Aspartate aminotransferase 32 U/L Lactate dehydrogenase 300 U/L A peripheral blood smear shows schistocytes. Which of the following is the most appropriate management for this patient? A Captopril B Immediate delivery of the fetus C Methyldopa D Plasma exchange

A Therapy with an angiotensin-converting enzyme inhibitor is indicated for scleroderma renal crisis in pregnant patients despite being associated with an increased risk to the fetus. The most likely diagnosis is scleroderma renal crisis (SRC), and the most appropriate management for this patient is initiation of captopril. Patients with early diffuse systemic sclerosis have an increased risk for developing SRC. The differential diagnosis of a pregnant patient with diffuse cutaneous systemic sclerosis who presents with acute kidney injury, proteinuria, thrombocytopenia, and a microangiopathic hemolytic anemia includes SRC and preeclampsia, eclampsia (preeclampsia with seizures), HELLP (hemolysis, elevated liver function tests, low platelets) syndrome, and thrombotic thrombocytopenic purpura (TTP). Differentiating among these conditions is often extremely difficult but critical. SRC occurs almost exclusively in patients with diffuse cutaneous systemic sclerosis, particularly those with skin fibrosis that develops within 3 years of disease onset. In this patient, the abrupt onset of the signs and symptoms of hypertension and renal failure in the presence of diffuse systemic sclerosis during early gestation supports the diagnosis of SRC and helps to exclude the other entities in the differential diagnosis. Angiotensin-converting enzyme inhibitor therapy with aggressive upward titration is indicated in patients with SRC to reduce mortality. Therefore, captopril must be used in this patient to preserve renal function, control blood pressure, and reduce mortality despite being associated with an increased risk to the fetus. Preeclampsia is most commonly characterized by the gradual onset of hypertension, proteinuria, and edema. Symptoms typically develop in the late third trimester and rarely as early as the late second trimester. Some patients experience the onset of symptoms after delivery, but the presence of signs and symptoms of this condition before 20 weeks of gestation is unusual and argues strongly against this diagnosis. HELLP syndrome develops only in women whose pregnancies are complicated by preeclampsia. Delivery of the fetus is curative for patients with preeclampsia, eclampsia, or HELLP syndrome. Because this patient is symptomatic during the twelfth week of pregnancy, preeclampsia, eclampsia, and the HELLP syndrome are unlikely and delivery of the fetus is not appropriate. Methyldopa is an appropriate treatment for chronic hypertension in a pregnant patient, but chronic hypertension would not explain this patient's acute kidney injury and microangiopathic anemia. Methyldopa also does not alter outcome in patients with SRC and is therefore not indicated for this patient. TTP is characterized by thrombocytopenia, microangiopathic hemolytic anemia, acute kidney injury, fever, and neurologic symptoms; however, hematologic and renal manifestations are usually predominant. TTP is a much more rare complication of pregnancy compared with preeclampsia and tends to present at midpregnancy or later. Like preeclampsia, the occurrence of TTP before 20 weeks would be very unusual. In addition, the lactate dehydrogenase level is often markedly elevated in TTP secondary to both hemolysis and widespread tissue ischemia. The treatment of choice for TTP is plasma exchange. Because TTP is an unlikely diagnosis in this patient based upon the timing of the symptoms, treatment with plasma exchange would not be warranted.

Rheuma 71 A 32-year-old man is evaluated for a 10-year history of low back pain and stiffness that are alleviated with exercise and hot showers. He does not have a history of skin, eye, or bowel disease. He has not had previous infections of the gastrointestinal or genitourinary systems. On physical examination, vital signs are normal. The sacroiliac joints and lumbar spine are tender to palpation. There is complete loss of forward flexion in the lower spine. When standing upright against a wall, he is unable to touch the occiput to the wall. Radiographs of the spine reveal complete fusion of the sacroiliac joints bilaterally and squaring of the vertebral bodies throughout the lumbar and thoracic spine. In addition to starting an NSAID and physical therapy, which of the following is the most appropriate treatment for this patient? A Etanercept B Low-dose prednisone C Methotrexate D ASulfasalazine

A Tumor necrosis factor α inhibitors are indicated as first-line therapy for ankylosing spondylitis and usually are associated with a degree of symptomatic relief and improvement in inflammatory changes visible on MRI. This patient most likely has ankylosing spondylitis, and the most appropriate treatment is therapy with an anti-tumor necrosis factor α agent such as etanercept. This condition usually affects patients in the teenage years or 20s and manifests as chronic low back pain and stiffness that are alleviated with exercise. This patient's clinical presentation is consistent with severe ankylosing spondylitis. As this condition progresses, spinal fusion and a resulting loss of spinal mobility may occur. An inability to touch a wall with the occiput when standing upright against the wall indicates a flexion deformity, and the distance between the wall and the occiput helps to measure the level of a patient's deformity. Radiographs of the spine in patients whose disease has progressed for several years typically reveal fusion of the sacroiliac joints as well as squaring of the vertebral bodies, which is caused by erosion of the corners of the vertebral bodies due to inflammation of the ligamentous attachments. Conventional therapies such as NSAIDs are useful in relieving symptoms and in helping to maintain function in patients with ankylosing spondylitis but do not prevent progressive joint damage, bony ankylosis, physical deformity, or disability in patients with severe disease. Most patients treated with NSAIDs show significant relief of back pain within 48 hours of therapy with an optimal anti-inflammatory dose of NSAIDs and a prompt return of symptoms (within 48 hours) after discontinuation of the drug. Physical therapy and a regular exercise program have been shown to provide symptomatic relief, improve function, and increase the likelihood of a more functional posture as spinal fusion progresses. This intervention is therefore recommended in all patients with ankylosing spondylitis. Until the advent of tumor necrosis factor α inhibitors, no therapy had been shown to significantly and potentially affect progressive spinal fusion in patients with ankylosing spondylitis. Use of these agents usually is associated with a degree of symptomatic relief and improvement in inflammatory changes visible on MRI. To date, however, these agents have not been shown to impact progressive spinal changes. Many of the agents used in rheumatoid arthritis, including methotrexate, sulfasalazine, and low-dose prednisone, are beneficial in the treatment of peripheral inflammatory arthritis associated with ankylosing spondylitis. However, these agents do not significantly affect spinal involvement in the spondyloarthropathies.

Rheuma 12 A 41-year-old woman is evaluated for intermittent pain and cyanosis of the fingers that is usually associated with exposure to cold temperatures or stress. She does not smoke, and her efforts to keep room temperatures warm and to wear gloves and layers of clothing to maintain her core temperature have not been successful in managing her symptoms. She was diagnosed with limited cutaneous systemic sclerosis 1 year ago. She also has gastroesophageal reflux disease. Her only medication is omeprazole. On physical examination, temperature is 37.0 °C (98.6 °F), blood pressure is 128/72 mm Hg, and pulse rate is 88/min. Cutaneous examination of the hands shows sclerodactyly. Radial and ulnar pulses are 2+ and equal bilaterally. Which of the following is the most appropriate additional treatment for this patient? A Amlodipine B Isosorbide dinitrate C Prednisone D Propranolol

A Use of a dihydropyridine calcium channel blocker is warranted in patients with Raynaud phenomenon in whom cold avoidance does not provide sufficient relief. This patient has Raynaud phenomenon, which is present in more than 95% of patients with systemic sclerosis and is particularly likely to develop in patients with limited cutaneous disease. The most appropriate treatment for this patient is amlodipine. Systemic sclerosis is classified according to the degree of skin involvement. Systemic sclerosis with limited cutaneous involvement, or CREST syndrome (calcinosis, Raynaud phenomenon, esophageal dysmotility, sclerodactyly, and telangiectasia), manifests as skin thickening distal to the elbows and knees. Conversely, systemic sclerosis with diffuse cutaneous involvement is associated with skin thickening proximal to the elbows and knees. Diffuse and limited cutaneous systemic sclerosis may affect the face. Episodes of Raynaud phenomenon are often precipitated by cold exposure or stress and usually involve the extremities. In patients with Raynaud phenomenon, cigarette smoking is contraindicated and avoidance of cold is recommended; pharmacologic therapy is warranted for patients in whom these interventions do not provide sufficient relief. Dihydropyridine calcium channel blockers such as amlodipine have been shown to reduce the frequency and severity of attacks in patients with both primary and secondary Raynaud phenomenon, and these agents are frequently used as first-line treatment in this condition. Other agents used to manage Raynaud phenomenon include peripherally acting α-1 blockers, phosphodiesterase inhibitors, and endothelin receptor antagonists. Topical nitrates applied to the finger webs are often used in the treatment of Raynaud phenomenon but are usually used as second-line therapy. Oral therapy with nitroglycerin is less effective and less well tolerated than amlodipine and is not indicated as a first-line drug for this condition. Raynaud phenomenon is caused by microvascular involvement in patients with systemic sclerosis and is characterized by intimal proliferation and progressive luminal obliteration, as well as digital spasm. This process does not respond to anti-inflammatory agents; therefore, prednisone is not indicated in the treatment of Raynaud phenomenon. β-Blockers such as propranolol are not indicated in the treatment of Raynaud phenomenon and may actually worsen symptoms by preventing β-adrenergic-mediated vasodilation.

Rheuma 16 A 33-year-old woman is evaluated during a follow-up visit. She was diagnosed with rheumatoid arthritis 3 months ago; at that time, she began methotrexate therapy and a folic acid supplement. She also takes ibuprofen and acetaminophen. Despite this treatment, she still has 2 to 3 hours of morning stiffness daily and wakes frequently during the night with pain and stiffness. She also has persistent pain in the hands and feet. On physical examination, vital signs are normal. The neck and shoulders are stiff but have full range of motion. Small nodules are present on the elbows. The right elbow has a small effusion and has 15 degrees of flexion contracture. The wrists and metacarpophalangeal joints are tender bilaterally, and there is synovitis of the wrists. The left knee has a small effusion. The metatarsophalangeal joints also are tender bilaterally. Laboratory studies: Hemoglobin 12.2 g/dL (122 g/L) Platelet count 460,000/µL (460 × 109/L) Erythrocyte sedimentation rate 45 mm/h Radiographs of the hands show periarticular osteopenia and erosions of the right ulnar styloid and the base of the left fifth metacarpal bone. Which of the following is the most appropriate next step in this patient's treatment? A Add etanercept B Add hydroxychloroquine C Add cyclophosphamide D Discontinue methotrexate; begin sulfasalazine

A When adequate control of rheumatoid arthritis is not achieved with one or more oral disease-modifying antirheumatic drugs, the addition of biologic therapy with a tumor necrosis factor α inhibitor is usually indicated. This patient has early, aggressive rheumatoid arthritis, and the addition of etanercept is indicated. Methotrexate is the most commonly used and safest among the more effective disease-modifying antirheumatic drugs (DMARDs), has the greatest potential for modifying disease compared with hydroxychloroquine and sulfasalazine, and is central to most treatments for rheumatoid arthritis. Despite treatment with methotrexate, this patient has persistent morning stiffness, numerous tender and swollen joints, and elevated erythrocyte sedimentation rate. When adequate disease control is not achieved with one or more oral DMARDs, biologic therapy is indicated. The preferred initial biologic agent in this setting is a tumor necrosis factor α inhibitor such as etanercept, which is usually added to baseline methotrexate therapy. Use of a tumor necrosis factor α inhibitor in addition to methotrexate is significantly more effective in controlling joint damage and improving function compared with single-agent therapy with either medication. Screening for tuberculosis is indicated before beginning therapy with any biologic agent, and patients who test positive for latent tuberculosis should be treated with isoniazid before beginning this therapy. Hydroxychloroquine is an effective agent in the treatment of early, mild, and nonerosive rheumatoid arthritis but most likely would not be beneficial in a patient with aggressive disease, functional limitations, and radiographic evidence of joint erosion. Even the addition of hydroxychloroquine to methotrexate is unlikely to be as helpful for erosive rheumatoid arthritis as is the combination of methotrexate and a tumor necrosis factor α inhibitor. Cyclophosphamide is not indicated for the treatment of active rheumatoid arthritis except in patients with rheumatoid vasculitis in whom major organ function is compromised. However, this patient does not have manifestations consistent with rheumatoid vasculitis, such as cutaneous ulcers and mononeuritis multiplex. Sulfasalazine is often administered in combination with methotrexate and/or hydroxychloroquine in the treatment of early rheumatoid arthritis. Single-agent therapy with sulfasalazine also may be used when there are contraindications to the use of methotrexate. However, sulfasalazine is less effective than methotrexate in the treatment of rheumatoid arthritis and is not likely to benefit this patient if substituted for methotrexate.

Rheuma 23 A 28-year-old woman is evaluated for a 3-week history of pain and swelling of the right knee and ankle. For the past 6 weeks, she has had diffuse, crampy abdominal pain. For the past week, the pain has been accompanied by four to six daily episodes of bloody diarrhea and fecal urgency. She has lost approximately 1.5 kg (3.3 lb) since the onset of her symptoms. She has not noticed a rash or other joint or soft-tissue involvement. She has not traveled outside of her hometown and has a monogamous sexual relationship with her husband. She has no other medical problems and does not take any medications. On physical examination, temperature is 37.7 °C (99.9 °F), blood pressure is 128/72 mm Hg, pulse rate is 98/min, and respiration rate is 18/min. The abdomen is soft and diffusely tender to palpation. Bowel sounds are normal, and there is no organomegaly. Rectal examination reveals tenderness of the rectal canal and stool associated with bright red blood. The right ankle and knee are swollen and slightly warm to the touch, and range of motion of these joints elicits pain. The remainder of the physical examination is normal. Plain radiographs of the ankle and knee are normal. Arthrocentesis is performed. Synovial fluid analysis reveals a leukocyte count of 14,000/µL (92% polymorphonuclear cells, 8% macrophages). Which of the following is the most likely cause of this patient's joint symptoms? A Crystal-induced arthritis B Enteropathic arthritis C Gonococcal arthritis D Whipple disease

B The presence of acute oligoarticular arthritis involving the lower extremities in a patient with inflammatory bowel disease is suggestive of enteropathic arthritis. This patient's joint symptoms are most likely caused by enteropathic arthritis. She has a 6-week history of crampy abdominal pain and the recent onset of bloody diarrhea and rectal urgency. She also has had weight loss. This clinical presentation raises suspicion for inflammatory bowel disease. For the past 3 weeks, this patient also has had acute arthritis of the right knee and ankle accompanied by inflammatory features such as tenderness and swelling; her synovial fluid findings confirm the presence of an inflammatory process. The presence of acute oligoarticular arthritis involving the lower extremities in a patient with an inflammatory diarrheal illness is suggestive of enteropathic arthritis; enteropathic arthritis also may manifest as axial arthritis, such as a spondyloarthropathy. Crystal-induced arthritis typically manifests as acute monoarticular arthritis and would be unlikely in a premenopausal woman. Gonococcal arthritis may be associated with oligoarticular arthritis, and joint manifestations in this condition may be migratory. However, patients with gonococcal arthritis commonly have tenosynovitis and cutaneous involvement, which are not present in this patient. Furthermore, neither gonococcal nor crystal-induced arthritis would explain this patient's diarrhea and abdominal pain. Whipple disease is an extremely rare infectious syndrome caused by Tropheryma whippelii. The most common presenting symptom in affected patients is arthritis; other symptoms include diarrhea, malabsorption, and central nervous system and constitutional symptoms. Joint involvement is usually migratory and follows a chronic course.

Rheuma 77 A 44-year-old woman is evaluated for a 6-week history of progressive fatigue. She was diagnosed with limited cutaneous systemic sclerosis 6 years ago. She also has gastroesophageal reflux disease well controlled with omeprazole and Raynaud phenomenon for which she takes amlodipine. She has no other medical problems and does not drink alcoholic beverages or take NSAIDs. On physical examination, temperature is 37.0 °C (98.6 °F), blood pressure is 98/68 mm Hg, pulse rate is 96/min, and respiration rate is 18/min. Physical examination is normal except for sclerodactyly and widespread telangiectasias. Abdominal examination is unremarkable with no evidence of hepatosplenomegaly. Laboratory studies: Hemoglobin 8.4 g/dL (84 g/L) Mean corpuscular volume 78 fL Ferritin 10 ng/mL (10 mg/L) Upper endoscopy is performed and reveals longitudinal rows of flat, reddish stripes radiating from the pylorus into the antrum consisting of ectatic and sacculated mucosal blood vessels. Which of the following is the most likely diagnosis? A Erosive gastropathy B Gastric antral vascular ectasia (GAVE) C Gastric lymphoma D Portal hypertensive gastropathy

B The presence of fatigue and iron deficiency anemia in a patient with systemic sclerosis should raise suspicion for gastric antral vascular ectasia. This patient most likely has gastric antral vascular ectasia (GAVE), also known as watermelon stomach. Patients with systemic sclerosis may develop cutaneous telangiectasias as well as mucosal telangiectasias that involve the upper and lower gastrointestinal tract; the development of telangiectasias in the stomach is known as GAVE and is associated with chronic blood loss. Most cases of GAVE are idiopathic and occur in elderly women, but this condition may occur secondary to systemic sclerosis or cirrhosis. The presence of fatigue and iron deficiency anemia in a patient with systemic sclerosis should raise suspicion for GAVE. A diagnosis of GAVE is established if upper endoscopy shows telangiectasias in the antrum. In patients with this condition, endoscopic laser coagulation and obliteration of vascular ectasias is then indicated to decrease the risk of gastrointestinal bleeding. The hallmark of erosive gastropathy is the development of erosive lesions or superficial ulcers due to a direct action of NSAIDs or alcohol, mucosal hypoxia, or a combination of factors. Erosions in this condition tend to be multiple and shallow and are usually located in the corpus and fundus. This patient has no apparent risk factors for erosive gastropathy, and the endoscopic appearance of her gastric mucosa is not consistent with this diagnosis. Patients with gastric lymphoma may present with pain, anorexia, weight loss, and occult gastrointestinal bleeding. Most cases occur in individuals who are 50 to 60 years of age. On endoscopy, the appearance is varied and may include a gastric mass with or without ulceration, an ulcer, or nodular thickened gastric folds. This patient's endoscopic findings are incompatible with gastric lymphoma. A condition similar to GAVE known as portal hypertensive gastropathy may develop in patients with cirrhosis and portal hypertension. In this condition, the severity of the gastropathy is related to the degree of portal hypertension, and there are diffuse antral angiomas rather than the classic linear pattern seen in GAVE. Portal hypertensive gastropathy is unlikely in a patient with no history of liver disease, hepatosplenomegaly, or ascites. In addition, the appearance of the gastric mucosa on endoscopy in this patient is not characteristic of this condition.

Rheuma 22 A 58-year-old man is evaluated for a 3-year history of recurrent episodes of pain and swelling of the joints of the lower extremities. These episodes have an acute onset, are monoarticular, reach maximum intensity within 1 day, and last between 2 and 7 days. The right great toe is most often affected, but the knee also is involved. During the past 6 months, he has had three episodes. On physical examination, vital signs are normal. Musculoskeletal examination is normal except for bony enlargement of the right first metatarsophalangeal joint with valgus deformity. There are no tophi. Which of the following diagnostic studies will be most helpful in establishing this patient's diagnosis? A 24-Hour urine uric acid B Radiography of the right foot C Serum uric acid D Synovial fluid leukocyte count

B A presumptive diagnosis of gout can be established based solely on a patient's history and a supportive radiograph. Gout typically manifests as acute intermittent attacks of joint and tenosynovial inflammation associated with redness, swelling, and intense pain. Early attacks of gout are typically monoarticular and involve joints in the lower extremities, particularly the first metatarsophalangeal joint. These episodes usually last between several days and 2 weeks. As the disease progresses, these attacks become more frequent and increasingly polyarticular and involve the upper extremities. Because this patient's presentation is classic for gout, a presumptive diagnosis can be established based solely on his history and a supportive radiograph. In patients with gout, radiographs often reveal bony asymmetric erosions with overhanging edges that usually involve the feet. These changes are supportive of the diagnosis but are not diagnostic. Gout is often associated with hyperuricemia, but an elevated serum uric acid level is not diagnostic of this condition. More than 20% of patients with gout have normal or low uric acid levels during an acute attack; similarly, approximately 5% of the adult general population has elevated uric acid levels, but most of these patients remain asymptomatic. A 24-hour urine measurement of uric acid excretion would help to determine whether this patient is an underexcretor or overproducer of uric acid. Results of this study would therefore help to guide treatment but would not confirm the diagnosis. Synovial fluid analysis during an acute attack of gout may not reveal monosodium urate crystals in up to 25% of affected patients. Therefore, aspiration of an affected joint during a subsequent attack may be appropriate if crystals are not initially seen on synovial fluid analysis in suspected gout. However, a synovial fluid analysis documenting a synovial fluid leukocyte count higher than 15,000/µL is compatible with gout but does not help to establish the diagnosis.

Rheuma 24 A 42-year-old woman with systemic lupus erythematosus is admitted to the hospital for a 1-week history of worsening headache and a 2-day history of confusion, personality change, and emotional lability. She was diagnosed with lupus 1 year ago, and her condition has been well controlled with prednisone, hydroxychloroquine, and ibuprofen. Four months ago, prednisone was discontinued. Two months ago, she developed fatigue, intermittent low-grade fevers, and occasional headache. Two weeks ago, she was evaluated in the office for a 1-month history of purpuric lesions on her legs, palms, and soles. Biopsy specimen of a lesion showed leukocytoclastic vasculitis. At that time, prednisone, 40 mg/d, was initiated, and the purpuric lesions began to fade. On physical examination today, temperature is 37.9 °C (100.2 °F), blood pressure is 150/80 mm Hg, pulse rate is 102/min, and respiration rate is 20/min. She is confused and combative. She is oriented to name only, intermittently laughs and cries, and has paranoid ideation. Cardiopulmonary examination is normal. She does not have photophobia or nuchal rigidity. Abdominal examination is unremarkable. She has livedo reticularis involving the arms and legs, and there are purpuric lesions on the palms, fingertips, legs, soles, and toes. There is no synovitis. Neurologic examination is nonfocal. Laboratory studies: Hemoglobin 11.1 g/dL (111 g/L) Leukocyte count 3500/µL (3.5 × 109/L) Platelet count 200,000/µL (200 × 109/L) Erythrocyte sedimentation rate 83 mm/h Serum complement (C3 and C4) Decreased Antinuclear antibodies Titer of 1:2560 Cerebrospinal fluid findings: Leukocyte count 55/µL (100% lymphocytes) Erythrocyte count 1/µL Protein 72 mg/dL (720 mg/L) Gram stain Negative VDRL Negative Culture results are pending. T2-weighted MRI of the brain reveals scattered punctate areas of increased signal in the periventricular and subcortical white matter. Which of the following is the most appropriate next step in this patient's treatment? A Begin ampicillin and gentamicin B Begin methylprednisolone and cyclophosphamide C Discontinue ibuprofen D Schedule plasmapheresis

B Aggressive treatment with corticosteroids accompanied by monthly pulse cyclophosphamide is indicated to treat patients with central nervous system lupus. This patient has a history of systemic lupus erythematosus (SLE) and presents with headache, psychosis, and lymphocytic pleocytosis. In addition, this patient has cutaneous vasculitis, an elevated erythrocyte sedimentation rate, and hypocomplementemia. This clinical presentation strongly suggests a diagnosis of central nervous system (CNS) lupus. The most appropriate next step in her management is the addition of methylprednisolone and cyclophosphamide. CNS lupus can be caused by vasculitis of the small vessels of the brain but also has been associated with the presence of antineuronal antibodies. MRI of the brain in affected patients may be normal or may reveal small areas of ischemia in the periventricular and subcortical white matter. Seizure also may develop in patients with this condition. CNS lupus is a severe manifestation of SLE that is generally treated aggressively. Although few controlled trials on the treatment of CNS lupus have been performed to date, one small study demonstrated better outcomes in those treated with monthly pulse cyclophosphamide compared with monthly pulse methylprednisolone, with both of these agents used in conjunction with prednisone, 1 mg/kg/d. Treatment with ampicillin and gentamicin would be warranted in a patient with Listeria monocytogenes meningitis, which is rare but occurs more frequently in patients with immunosuppression and collagen vascular disorders. However, L. monocytogenes meningitis usually is associated with nuchal rigidity and would not explain this patient's psychotic symptoms. Furthermore, lymphocytic pleocytosis on cerebrospinal fluid analysis and a negative Gram stain argue strongly against this condition. Ibuprofen is an unlikely cause of this patient's clinical presentation. NSAIDs may be associated with aseptic meningitis, particularly in patients with SLE, but generally do not cause mental status changes or cutaneous vasculitis. Plasmapheresis is an extracorporeal technique that results in the removal of large-molecular-weight substances from the plasma, such as autoantibodies, immune complexes, cryoglobulins, and myeloma light chains. This intervention is used to treat thrombotic thrombocytopenic purpura but has not been shown to be beneficial in CNS lupus and may be associated with an increased risk of infection, including catheter-related infection.

Rheuma 8 A 76-year-old man comes for a preoperative evaluation before total joint arthroplasty of the right knee. He has a 24-year history of rheumatoid arthritis. His disease has been stable, but he has had progressive pain and loss of range of motion of the right knee. He has no other medical problems and has never been admitted to the hospital. Medications are methotrexate, a folic acid supplement, hydroxychloroquine, and prednisone. On physical examination, temperature is 37.2 °C (99.0 °F), blood pressure is 136/80 mm Hg, pulse rate is 90/min, and respiration rate 18/min. BMI is 23. Cardiopulmonary examination is normal. There is mild puffiness of the metacarpophalangeal joints bilaterally. He also has bilateral ulnar deviation and swan neck deformities involving the third digit of the right hand and the fourth digit of the left hand. Extension of the cervical spine is painful and decreased. There is a bony deformity of the right knee. Extension of the right knee is decreased by 10 degrees and flexion is limited to 110 degrees. Neurologic examination is unremarkable. Laboratory studies are normal, including the complete blood count and serum creatinine level. Chest radiograph and electrocardiogram are normal. Which of the following preoperative diagnostic studies should be performed in this patient? A B-type natriuretic peptide hormone B Cervical spine radiograph C Spirometry D Urinalysis

B Cervical spine radiographs with flexion and extension views are indicated for any patient with aggressive or long-standing rheumatoid arthritis to evaluate for cervical instability at the atlantoaxial articulation and are particularly important in patients who may undergo intubation. Patients with long-standing rheumatoid arthritis have a significant risk of cervical instability at the atlantoaxial articulation, and this patient's painful, decreased extension of the cervical spine raises suspicion for this condition. Additional manifestations of cervical instability at this site include occipital headaches, loss of coordination, paresthesias of the hands and feet, and urinary retention or incontinence, although this condition may be relatively asymptomatic until late in the disease course. Cervical spine radiographs with flexion and extension views are indicated for any patient with aggressive or long-standing rheumatoid arthritis to evaluate for cervical instability at this site. Evaluation for this condition is particularly important in the perioperative setting, when extension of the neck for intubation may lead to spinal cord compromise and resultant paraplegia. Patients with compensated, asymptomatic heart failure can generally undergo surgery if other risk factors are acceptable. Because this patient has no history of heart failure and a normal physical examination and electrocardiogram, cardiac risk is low and further preoperative evaluation is unnecessary. The role of B-type natriuretic peptide hormone in preoperative risk assessment is currently undefined and is not recommended for asymptomatic individuals. Rheumatoid arthritis may be associated with pulmonary involvement, but routine spirometry is not indicated for patients without pulmonary disease before surgery. Preoperative spirometry does not improve risk assessment over that of clinical evaluation alone. In the presence of a normal serum creatinine level, routine urinalysis is unlikely to detect the presence of significant kidney disease. Moreover, current evidence does not support a relationship between asymptomatic urinary tract infection and surgical infection. Therefore, routine preoperative urinalysis is not recommended for most surgical procedures.

Rheuma 61 A 25-year-old woman is evaluated during a routine follow-up visit. Four months ago, she was diagnosed with systemic lupus erythematosus that manifested as fatigue, malar rash, oral ulcers, pleuritis, and arthralgia. At that time, she began treatment with hydroxychloroquine and a 1-month course of low-dose prednisone. On physical examination today, she states that her symptoms have resolved somewhat but that she still has slight fatigue and mild arthralgia in her hands, feet, and knees. Temperature is 36.4 °C (97.6 °F), blood pressure is 130/92 mm Hg, pulse rate is 84/min, and respiration rate is 18/min. She has a mild malar flush, a painless ulcer on the hard palate, and trace bilateral ankle edema. The remainder of the examination is normal. Laboratory studies: Hemoglobin 10 g/dL (100 g/L) Leukocyte count 2300/µL (2.3 × 109/L) Platelet count 132,000/µL (132 × 109/L) Erythrocyte sedimentation rate 45 mm/h Serum creatinine 1.0 mg/dL (76.3 µmol/L) Albumin 3.1 g/dL (31 g/L) Serum complement (C3 and C4) Decreased Urinalysis 2+ protein; 3+ blood; 5-10 leukocytes, 15-20 erythrocytes, and 1 erythrocyte cast/hpf Which of the following is the next best step in this patient's treatment? A Amlodipine B High-dose prednisone C Ibuprofen D Low-dose prednisone

B Early treatment with high-dose corticosteroids is indicated in patients whose condition raises strong suspicion for lupus nephritis. This patient's hypertension, ankle edema, hematuria, proteinuria, hypoalbuminemia, and erythrocyte casts on urinalysis are highly suggestive of lupus nephritis despite the absence of renal insufficiency. To prevent irreversible renal damage, early treatment with a high-dose corticosteroid such as prednisone is indicated for patients whose condition raises strong suspicion for lupus nephritis. Whether renal biopsy is necessary in this clinical situation in order to establish a diagnosis remains uncertain, and treatment with high-dose corticosteroids would not significantly alter subsequent biopsy results. Initiation of antihypertensive therapy would benefit this patient but is not the most appropriate next step in the management of her condition; treatment of her nephritis takes precedence and may itself help to control her hypertension. Instead of a calcium channel blocker such as amlodipine, angiotensin-converting enzyme inhibitors are the antihypertensive drugs of choice in patients with lupus nephritis because these agents help to control proteinuria. Ibuprofen may help to control this patient's arthralgia. However, NSAIDs can significantly worsen renal function in patients with lupus nephritis and are therefore contraindicated in this patient population. Low-dose prednisone may help to alleviate this patient's arthralgia and rash but would not treat her lupus nephritis.

Rheuma 41 An 83-year-old woman is evaluated for persistent pain related to osteoarthritis of the left knee, which was diagnosed 3 years ago. She continues to ambulate with a cane as instructed by a physical therapist but is unable to walk more than 4 blocks without sitting down because of pain. She currently takes acetaminophen, but her pain remains severe. Previous treatment with acetaminophen with codeine was discontinued because of severe constipation, and tramadol was discontinued because of dizziness. Medical history is significant for hypertension, heart failure, and chronic kidney disease. She also takes lisinopril, metoprolol, and furosemide. She currently prefers medical therapy to surgical intervention, if possible. On physical examination, the left knee has slight valgus angulation and a medium-sized effusion without warmth or erythema. There is no tenderness along the joint line. On laboratory studies, erythrocyte sedimentation rate is 10 mm/h, C-reactive protein is normal. A radiograph of the left knee shows moderate lateral joint-space narrowing and osteophyte formation. Which of the following is the most appropriate next step in this patient's management? A Celecoxib B Hyaluronan injections C Hydrocodone with acetaminophen D Referral for total knee arthroplasty

B Hyaluronan injections may be considered in patients with osteoarthritis of the knee in whom NSAIDs are either contraindicated or do not provide adequate pain relief. In patients with osteoarthritis of the knee, intra-articular hyaluronan injections have been shown to be superior to placebo and comparable to other active interventions, such as physical therapy, NSAIDs, and intra-articular corticosteroids. This therapy may be considered in patients with osteoarthritis of the knee in whom NSAIDs are either contraindicated or do not provide adequate pain relief. The treatment consists of three or more hyaluronan injections administered 1 week apart, depending on the preparation. Hyaluronan injections can provide pain relief for at least 6 months but may not become effective until several weeks after an injection is administered. This intervention is effective in only a small number of patients, but those in whom this therapy is effective experience significant benefits. NSAIDs have been shown to be more effective than acetaminophen in relieving pain associated with osteoarthritis. However, neither selective nor nonselective NSAIDs would be indicated for an elderly patient with chronic kidney disease. These agents also should be used with caution in patients with heart failure and hypertension. Like hyaluronan injections, narcotic analgesics may benefit patients in whom NSAIDs are contraindicated or for whom NSAIDs do not sufficiently relieve pain. However, a narcotic analgesic is not appropriate in a patient with a history of severe constipation caused by previous narcotic treatment. Total knee arthroplasty is warranted only when no further medical therapy is available and the patient decides that the impairment caused by his or her condition warrants this intervention. Because of this patient's comorbidities, her functional status is most likely not sufficiently impaired to warrant the risk of surgery, and most importantly, the patient has indicated a preference for medical therapy.

Rheuma 57 A 42-year-old woman is evaluated during a follow-up visit for a 3-year history of polyarthralgia involving the metacarpophalangeal and proximal interphalangeal joints of the hands, wrists, elbows, shoulders, knees, and ankles accompanied by occasional swelling of the wrists and hands. Over the past 3 months, her joint symptoms have worsened, and she has had intermittent mouth ulcers, redness of the cheeks, and pain on inspiration. She was started on naproxen 1 week ago, and she states today that her joint and chest pain has decreased by approximately 80%. She takes no other medications. On physical examination, temperature is 37.1 °C (98.7 °F), blood pressure is 134/82 mm Hg, pulse rate is 84/min, and respiration rate is 16/min. Cardiopulmonary examination is normal. There is mild malar erythema, and there is one ulcer on the palate. Shotty cervical and axillary lymphadenopathy is present. Abdominal examination is unremarkable. Examination of her hands reveals ulnar deviation and metacarpal subluxation. The deformities in her hands are reducible, and she has full range of motion of all joints. The wrists and ankles are mildly tender. Laboratory studies: Hemoglobin 12.9 g/dL (129 g/L) Leukocyte count 3900/µL (3.9 × 109/L) Erythrocyte sedimentation rate 42 mm/h Rheumatoid factor 50 U/mL (50 kU/L) C3 40 mg/dL (400 mg/L) C4 10 mg/dL (100 mg/L) (normal range 13-38 mg/dL [130-380 mg/L]) Antinuclear antibodies Titer of 1:640 Anti-Ro/SSA antibodies Positive Radiograph of the hands shows no erosions. Which of the following is the most appropriate treatment for this patient? A Etanercept B Hydroxychloroquine C Methotrexate D No additional treatment

B Hydroxychloroquine is usually effective in managing mild manifestations of systemic lupus erythematosus and helps to prevent disease flares. This patient's arthralgia and arthritis, malar rash, oral ulcers, symptoms of pleuritis, positive titers of antinuclear and anti-Ro/SSA antibodies, and hypocomplementemia are consistent with systemic lupus erythematosus (SLE). This condition also may be associated with low titers of rheumatoid factor and tendon laxity that results in reducible joint deformities that mimic those in patients with rheumatoid arthritis but are not erosive; the development of these deformities is known as Jaccoud arthropathy. NSAIDs help to relieve pain in patients with SLE who have serositis or joint manifestations but do not alter disease progression or prevent disease flares. Therefore, these agents are not typically used as monotherapy for SLE. Hydroxychloroquine is usually effective in managing mild manifestations of SLE, including cutaneous and joint involvement. This medication also is an extremely effective disease-modifying agent in SLE and helps to prevent disease flares. Rarely, hydroxychloroquine causes an irreversible retinopathy, and regular ophthalmologic screening is indicated in patients using this agent. The tumor necrosis factor α inhibitor etanercept may worsen disease activity in patients with SLE and is generally not used in this patient population. Methotrexate is indicated for patients with active rheumatoid arthritis. This patient has positive titers of rheumatoid factor, but this finding alone is not diagnostic of rheumatoid arthritis. In addition, although this patient's joint deformities are suggestive of erosive joint disease associated with rheumatoid arthritis, they are in reality caused by tendon laxity. Methotrexate can be used in patients with SLE whose arthritis is unresponsive to hydroxychloroquine and is indicated for rare patients with SLE who have erosive joint disease.

Rheuma 13 A 75-year-old woman is evaluated for a sudden loss of vision in the left eye that began 30 minutes ago. She has a 2-week history of fatigue; malaise; and pain in the shoulders, neck, hips, and lower back. She also has a 5-day history of mild bitemporal headache. On physical examination, temperature is 37.3 °C (99.1 °F), blood pressure is 140/85 mm Hg, pulse rate is 72/min, and respiration rate is 16/min. BMI is 31. The left temporal artery is tender. Funduscopic examination reveals a pale, swollen optic disc. Range of motion of the shoulders and hips elicits moderate pain. Laboratory studies: Hemoglobin 9.9 g/dL (99 g/L) Leukocyte count 7300/µL (7.3 × 109/L) Platelet count 456,000/µL (456 × 109/L) Erythrocyte sedimentation rate 116 mm/h Which of the following is the most appropriate next step in this patient's management? A Brain MRI B High-dose intravenous methylprednisolone C Low-dose oral prednisone D Temporal artery biopsy

B In patients whose clinical presentation is suspicious for giant cell arteritis, corticosteroid therapy should be instituted immediately, before diagnostic testing is performed. This patient's headache, temporal artery tenderness, acute visual loss, fever, and mild anemia are strongly suggestive of giant cell arteritis (GCA). Immediate high-dose intravenous methylprednisolone is indicated for this patient. Pain in the shoulder and hip girdle accompanied by a significant elevation in the erythrocyte sedimentation rate is consistent with polymyalgia rheumatica, which is present in approximately 33% of patients with GCA. Anterior ischemic optic neuropathy usually causes acute and complete visual loss in patients with GCA, and funduscopic examination of these patients typically reveals a pale, swollen optic nerve. Rarely, patients with GCA regain vision if treated immediately with high doses of an intravenous corticosteroid such as methylprednisolone (1 g/d or 100 mg every 8 hours for 3 days) followed by oral prednisone (1 to 2 mg/kg/d). More importantly, this aggressive regimen helps to prevent blindness in the contralateral eye. Therefore, although temporal artery biopsy is the gold standard for diagnosing GCA, diagnostic testing should not precede treatment in patients whose clinical presentation is suspicious for this condition. Even in the absence of visual loss, GCA is a medical emergency. In a patient whose condition is suspicious for GCA but who does not have visual loss, immediate initiation of high-dose oral prednisone before diagnostic testing is performed also is indicated. Whether intravenous corticosteroid therapy is more effective than oral administration of prednisone for patients with GCA and visual loss remains uncertain. Nevertheless, intravenous therapy seems reasonable in this circumstance and is recommended by many experts, even though rigorous studies have not validated this approach. However, it is clear that low-dose oral prednisone, which is an adequate treatment for isolated polymyalgia rheumatica, does not sufficiently treat GCA. A process in the brain is unlikely to cause monocular visual loss, and patients with GCA typically have normal findings on brain MRI. Therefore, this study would most likely be unhelpful in this patient. In patients whose condition raises a strong suspicion of GCA, temporal artery biopsy should be performed after corticosteroid therapy is begun. Corticosteroid therapy will not affect the results of temporal artery biopsy as long as biopsy is performed within 2 weeks of initiating this therapy; positive biopsy results have been seen as late as 6 weeks after institution of high-dose corticosteroid therapy, but the yield of biopsy is higher when this study is performed sooner.

Rheuma 55 A 46-year-old woman is evaluated during a follow-up visit. Six months ago, she was diagnosed with dermatomyositis that manifested as proximal muscle weakness, Gottron papules, and a serum creatine kinase level of 2300 U/L. Her disease responded well to prednisone, 60 mg/d, and this dosage was gradually tapered. One month ago, her creatine kinase level was 340 U/L, and her prednisone dosage was tapered to 20 mg/d. Her symptoms have been well controlled on this decreased dosage. Additional medications are risedronate and calcium and vitamin D supplements. On physical examination, vital signs are normal. Proximal and distal muscle strength testing is 5/5. Laboratory studies: Glucose (fasting) 183 mg/dL (10.2 mmol/L) Creatine kinase 1250 U/L Alanine aminotransferase 87 U/L Aspartate aminotransferase 68 U/L Which of the following is the most appropriate management for this patient? A Add cyclosporine B Add methotrexate C Add rituximab D Increase prednisone dosage

B In patients with myositis undergoing long-term corticosteroid therapy, the addition of methotrexate helps to control muscle disease and allow for further tapering of the corticosteroid dosage over time. This patient has dermatomyositis, which initially responded well to high-dose prednisone therapy. However, now that her prednisone dosage has been decreased, her elevated creatine kinase and aspartate and alanine aminotransferase levels indicate that she again has active disease. She also is now experiencing hyperglycemia, a complication of high-dose corticosteroid therapy. The addition of methotrexate is indicated for this patient. Because an increase in creatine kinase levels often precedes a decrease in muscle strength, this patient's normal results on muscle strength testing do not preclude active myositis. Unless additional immunosuppressive therapy is instituted in this patient, she most likely will develop muscle weakness. Methotrexate is a commonly used adjunctive immunosuppressive agent in patients undergoing long-term corticosteroid therapy and will help to treat this patient's myositis and allow for further tapering of her corticosteroid dosage over time. This patient's elevated alanine and aspartate aminotransferase levels most likely reflect muscle injury rather than a liver abnormality. When using methotrexate in patients with myositis, liver toxicity due to use of this agent should only be suspected when these enzyme levels increase in the presence of a stable or decreasing creatine kinase level. This agent can be used safely in this setting with appropriate monitoring. Azathioprine, individually and in combination with methotrexate, also has been shown to be an effective steroid-sparing agent in the treatment of inflammatory myositis. Further decreasing this patient's prednisone dosage without adding additional therapy would place her at risk for worsening of her myositis and therefore would not be appropriate. The initiation of cyclosporine may be considered as an adjunct or substitute for methotrexate and azathioprine if these agents are ineffective in the treatment of myositis or cannot be tolerated. This agent may be effective but in some patients is associated with an increase in blood pressure and a reduction in renal function, which has limited its use as a first-line steroid-sparing agent. Uncontrolled studies support the effectiveness of rituximab in the treatment of myositis, but this agent is considered a third-line treatment after failure of methotrexate or azathioprine.

Rheuma 62 A 31-year-old woman is evaluated during a follow-up visit. Five years ago, she was diagnosed with systemic lupus erythematosus that manifested as malar rash, arthralgia, mild hemolytic anemia, and thrombocytopenia; until now, her condition had been well controlled with hydroxychloroquine and low-dose prednisone. She became pregnant 7 months ago but 1 month ago developed new-onset hypertension and proteinuria and experienced a late second-trimester fetal loss. At that time, laboratory studies showed a serum creatinine level of 1.5 mg/dL (114.5 µmol/L), hypocomplementemia, anti-double-stranded DNA antibodies, and hematuria; renal biopsy revealed World Health Organization class IV lupus nephritis. Treatment with high-dose prednisone and lisinopril was begun, and she received her first dose of monthly intravenous cyclophosphamide. She also experienced a first-trimester fetal loss 1 year ago. Additional medications are calcium and vitamin D supplements. On physical examination today, temperature is 36.4 °C (97.6 °F), blood pressure is 140/85 mm Hg, pulse rate is 88/min, and respiration rate is 16/min. Cardiopulmonary examination is normal. There is no rash or lymphadenopathy. Abdominal examination is unremarkable. There is no synovitis. There is bilateral 1+ peripheral edema. Laboratory studies: Serum creatinine 1.2 mg/dL (91.6 µmol/L) Serum complement (C3 and C4) Decreased Antinuclear antibodies Titer of 1:2560 Anti-double-stranded DNA antibodies Positive IgG-specific anticardiolipin antibodies Elevated Urinalysis 2+ protein; 2+ blood; 5-10 leukocytes, 10-15 erythrocytes/hpf She would like to attempt pregnancy again after successful management of the lupus nephritis and seeks advice on how to maintain her fertility until that time. Which of the following is the most appropriate next step in the management of this patient's reproductive health? A Oral dehydroepiandrosterone B Leuprolide acetate injections C Oral contraceptives D Placement of an intrauterine device

B In women treated with cyclophosphamide, coadministration of leuprolide acetate helps to maintain fertility and provide contraception. This patient has active systemic lupus erythematosus (SLE), lupus nephritis, antiphospholipid antibodies, and a history of recurrent fetal loss. The most appropriate management option for this patient is to start monthly leuprolide acetate injections. Pregnancy outcomes in pregnant women with recently or currently active SLE at the time of conception tend to be poor, and pregnancy in this setting is associated with an increased risk of intrauterine growth restriction and premature birth. Furthermore, pregnant patients with SLE, particularly those with renal manifestations, have an increased risk of preeclampsia. Active SLE is therefore a relative contraindication to pregnancy. Cyclophosphamide therapy, when combined with high-dose corticosteroids, is the treatment of choice in patients with lupus nephritis. However, this drug is a U.S. Food and Drug Administration category D agent in pregnancy, and conception should be avoided during its use. In addition, a course of cyclophosphamide is likely to cause infertility in a 31-year-old patient. Therefore, effective management of this patient's reproductive health consists of trying to avoid pregnancy during cyclophosphamide therapy and while SLE is active as well as trying to maintain fertility. In patients with mildly to moderately severe SLE, dehydroepiandrosterone has been shown to lower disease activity. However, this therapy has not been shown to be beneficial in patients with lupus nephritis. This therapy also does not preserve fertility or provide contraception. Monthly leuprolide acetate injections administered 2 weeks before administration of each cyclophosphamide dose will induce a temporary menopause and have been shown to protect the ovaries from the effects of cyclophosphamide. This treatment will maintain fertility and provide contraception, although use of a nonhormonal form of contraception, such as a barrier method or placement of an intrauterine device, is recommended to further ensure that pregnancy does not occur. Leuprolide acetate therapy can be discontinued once remission of this patient's renal disease has been achieved and she is switched from cyclophosphamide to mycophenolate mofetil (generally after 3 to 6 months). Use of a nonhormonal form of contraception alone would not prevent cyclophosphamide-induced infertility in this patient. Mycophenolate mofetil therapy does not have a detrimental effect on fertility and is an appropriate alternative to cyclophosphamide to induce and maintain remission of lupus nephritis. However, mycophenolate mofetil has been associated with birth defects and is therefore contraindicated during pregnancy. Women of child-bearing age should use effective contraception while taking this agent because of the risk of congenital malformation and pregnancy loss. Oral contraceptives increase the risk of thrombosis and are generally avoided in patients with antiphospholipid antibodies. These agents also may exacerbate activity of SLE and are therefore contraindicated in patients with SLE who have active and/or unstable disease. However, recent studies have shown that oral contraceptives can be used in patients with mild, stable SLE without inducing a flare.

Rheuma 21 A 56-year-old woman is evaluated during a follow-up visit for persistent rheumatoid arthritis. She was diagnosed with this condition 1 year ago and began treatment with methotrexate, 20 mg weekly, and hydroxychloroquine, 400 mg/d; etanercept was added 6 months ago. For the past 2 months, she has had persistent pain and swelling in the hands and knees as well as increased morning stiffness that lasts up to 3 hours daily. These symptoms interfere with her daily activities. Prednisone, 5 mg/d, was added 4 weeks ago and has provided only modest benefits. Medical history is significant for osteopenia. She also takes alendronate and calcium, vitamin D, and folic acid supplements. On physical examination, vital signs are normal. Range of motion of the shoulders is decreased. There are nodules on the elbows bilaterally and bilateral synovitis of the wrists. Grip strength is decreased. There are small effusions on the knees and squeeze tenderness of the metatarsophalangeal joints bilaterally. Laboratory studies: Hemoglobin 11 g/dL (110 g/L) Leukocyte count 8000/µL (8 × 109/L) Erythrocyte sedimentation rate 56 mm/h Rheumatoid factor Positive Anti-cyclic citrullinated peptide antibodies Positive Radiograph of the hands shows erosions of the right second metacarpophalangeal joint and at the base of the left fifth metacarpal bone. Diffuse periarticular osteopenia also is visible. Which of the following is the most appropriate next step in this patient's management? A Add infliximab B Discontinue etanercept; switch to infliximab C Discontinue methotrexate; switch to sulfasalazine D Increase prednisone

B Many patients with rheumatoid arthritis who do not respond to therapy with a tumor necrosis factor α inhibitor have a positive response to a different agent in this class. This patient has aggressive rheumatoid arthritis and should discontinue etanercept and switch to an alternative tumor necrosis factor α inhibitor. The goal of treatment in patients with rheumatoid arthritis is to maintain no evidence of disease. Frequent monitoring is therefore indicated to ensure a continued response to therapy, and medications often must be adjusted to achieve adequate disease control. Serial measurement of factors such as the number of swollen and tender joints and acute phase reactants can help to evaluate disease progression and guide treatment decisions. Periodic radiography also helps to confirm that erosive disease has not progressed despite therapy. Despite treatment with appropriate dosages of methotrexate and a tumor necrosis factor α inhibitor, this patient has numerous tender and swollen joints, positive rheumatoid factor, positive anti-cyclic citrullinated peptide antibodies, anemia, and an elevated erythrocyte sedimentation rate. She also has evidence of erosive disease on radiography. A change in therapy is therefore warranted. The three currently available tumor necrosis factor α inhibitors, etanercept, infliximab, and adalimumab, all have similar efficacy. However, many patients who do not respond to one of these agents have a positive response to a different agent in this class. In patients with rheumatoid arthritis, concomitant therapy with two or more biologic agents does not enhance their efficacy and poses a risk for toxicity due to infection. Therefore, the addition of infliximab to this patient's current therapeutic regimen would not be recommended. Sulfasalazine is often administered as combination therapy with methotrexate and/or hydroxychloroquine. This agent can be used as a single agent in patients with mild rheumatoid arthritis and often is added to methotrexate in the treatment of moderate to severe disease. However, sulfasalazine is unlikely to be effective as a single agent in a patient with progressive erosive disease. Increasing this patient's prednisone dosage would most likely help to relieve her signs and symptoms of active disease but would not prevent further disease progression. Furthermore, long-term corticosteroid therapy, particularly at an increased dosage, places this patient at higher risk for corticosteroid toxicity.

Rheuma 28 A 26-year-old woman is evaluated for a 2-month history of pain and swelling in the hands and daily morning stiffness that lasts for 3 to 4 hours. She is 4 months postpartum, and her pregnancy was without complications. She has no history of rash and is otherwise well. Her only medication is ibuprofen, which has not sufficiently relieved her symptoms. On physical examination, temperature is normal, blood pressure is 110/68 mm Hg, pulse rate is 82/min, and respiration rate is 16/min. The second and third proximal interphalangeal and metacarpophalangeal joints and the wrists are tender and swollen bilaterally. Laboratory studies show an erythrocyte sedimentation rate of 67 mm/h, and titers of IgM antibodies against parvovirus B19 are negative. Which of the following is the most likely diagnosis? A Gout B Osteoarthritis C Parvovirus B19 infection D Rheumatoid arthritis

D Rheumatoid arthritis is the most common cause of chronic, inflammatory polyarthritis in premenopausal women. This patient most likely has rheumatoid arthritis, which is the most common cause of chronic, inflammatory polyarthritis in premenopausal women. Rheumatoid arthritis commonly affects the metacarpophalangeal, proximal interphalangeal, and wrist joints. This patient's swelling, prolonged morning stiffness, and elevated erythrocyte sedimentation rate are consistent with this diagnosis. Furthermore, women are three times more likely to develop rheumatoid arthritis than men and have a slightly increased risk of developing this condition during the first 3 months postpartum. Gout may involve the hand and wrist and is associated with inflammatory features. However, gout usually has an asymmetric presentation and is unlikely to develop in a premenopausal woman. Osteoarthritis may manifest as chronic arthritis involving the proximal interphalangeal joints but would not affect the metacarpophalangeal joints or the wrists. Secondary osteoarthritis related to trauma or a metabolic condition such as hemochromatosis may explain this patient's pattern of joint involvement, but this condition would be unlikely in a 26-year-old woman. Osteoarthritis also would not have an inflammatory presentation. Viral arthritis usually is self limited except when associated with hepatitis B and C virus infection. Parvovirus B19 infection in adults may induce an acute rheumatoid factor-positive oligo- or polyarthritis. Most adult patients with parvovirus B19 infection also develop rash, but only rarely in adults does rash manifest as the classic rash seen in childhood erythema infectiosum, the "slapped cheek" rash. Diagnosis of acute parvovirus B19 infection may be established by detecting circulating IgM antibodies against parvovirus B19. Viral arthritis usually resolves within 3 weeks, although a minority of patients may develop persistent arthritis. The arthritis associated with acute parvovirus B19 infection does not cause joint destruction, and supportive analgesic therapy with NSAIDs is appropriate as tolerated. Parvovirus B19 infection is unlikely in this patient considering the duration of her symptoms, absence of rash, and negative titers of IgM antibodies against parvovirus B19.

Rheuma 49 An 82-year-old woman is evaluated for a flare of polymyalgia rheumatica manifested by aching in the shoulders and hips that began 2 weeks ago. She also has fatigue and malaise. She was diagnosed with polymyalgia rheumatica 8 months ago. At that time, she was prescribed prednisone, 20 mg/d; her symptoms promptly resolved; and her prednisone dosage was gradually tapered. Four months ago, her prednisone dosage was decreased from 7.5 mg/d to 5 mg/d, and her symptoms returned. Her prednisone dosage was then increased to 10 mg/d followed by a slow taper of this agent. Her prednisone dosage was most recently decreased from 7 mg/d to 6 mg/d, which is her current dosage. She also takes calcium and vitamin D supplements and a bisphosphonate. On physical examination, vital signs are normal. Range of motion of the shoulders, neck, and hips elicits mild pain. There is no temporal artery tenderness. Which of the following is the most appropriate treatment for this patient? A Increase prednisone to 20 mg/d B Increase prednisone to 7.5 mg/d; add methotrexate C Increase prednisone to 20 mg/d; add methotrexate D Increase prednisone to 7.5 mg/d; add infliximab

B Methotrexate is an effective steroid-sparing agent in the treatment of polymyalgia rheumatica. The most appropriate management in this patient is to increase the prednisone dosage to 7.5 mg/d and add methotrexate, 10 mg weekly. Typically, patients with polymyalgia achieve resolution of their symptoms with low-dose prednisone (10 to 20 mg/d); once these symptoms are controlled, the prednisone dosage can then be tapered. However, polymyalgia rheumatica commonly recurs when the prednisone dosage is being tapered. During flares, the prednisone dosage should be increased to the minimum amount needed to provide symptomatic relief; once symptoms subside, slower tapering of the dosage is warranted. Because two previous attempts to taper this patient's prednisone dosage below 7.5 mg/d have been unsuccessful, the addition of a steroid-sparing agent as well as an increase in her prednisone dosage is warranted. Methotrexate in particular has been shown to be an effective steroid-sparing agent in patients with polymyalgia rheumatica. This patient's prednisone dosage should be increased to the minimum dosage needed to control her symptoms, which in this individual has been shown to be between 7 and 7.5 mg/d. Increasing this patient's prednisone dosage to 20 mg/d would unnecessarily place her at greater risk for corticosteroid toxicity. Infliximab has not been shown to be an effective steroid-sparing agent in patients with polymyalgia rheumatica and therefore would not be indicated for this patient.

Rheuma 40 A 45-year-old woman is evaluated for a 2-week history of pleuritic chest pain. She has a 6-month history of arthralgia and a 2-month history of myalgia and mild proximal muscle weakness. She has difficulty climbing stairs, rising from a chair, and removing dishes from a high cabinet. She also has a 10-year history of Raynaud phenomenon. On physical examination, temperature is 36.4 °C (97.6 °F), blood pressure is 125/78 mm Hg, pulse rate is 90/min, and respiration rate is 18/min. Cardiopulmonary examination is normal. Abdominal examination is unremarkable. There are healed ulcerations on the second and third fingers of the right hand. There is no synovitis. Proximal upper- and lower-extremity muscle strength is 4/5 and is associated with mild muscle tenderness. Laboratory studies: Hemoglobin 12 g/dL (120 g/L) Erythrocyte sedimentation rate 63 mm/h Serum creatinine 0.9 mg/dL (68.7 µmol/L) Creatine kinase 896 U/L Alanine aminotransferase 98 U/L Aspartate aminotransferase 67 U/L Alkaline phosphatase 80 U/L Antinuclear antibodies Titer of 1:2560 Urinalysis Normal A chest radiograph shows blunting of the costophrenic angles. Which of the following antibody assays will confirm the most likely diagnosis? A Antimitochondrial B Antiribonucleoprotein C Anti-Ro/SSA D Anti-Smith E Antitopoisomerase I (anti-Scl-70)

B Mixed connective tissue disease is characterized by Raynaud phenomenon, arthralgia, and myositis accompanied by high titers of antiribonucleoprotein antibodies. This patient most likely has mixed connective tissue disease, which is a specific disorder that combines clinical findings of systemic lupus erythematosus, systemic sclerosis, and polymyositis and is characterized by high titers of anti-ribonucleoprotein antibodies. This patient's long-standing Raynaud phenomenon with associated digital ulcerations, arthralgia, myositis (characterized by proximal muscle weakness and elevated muscle enzyme levels, including alanine and aspartate aminotransferase and creatine kinase levels), and pleuritic chest pain compatible with serositis is consistent with mixed connective tissue disease. Antiribonucleoprotein antibodies may be present in patients with systemic lupus erythematosus (SLE), particularly those whose manifestations include Raynaud phenomenon and mild myositis. Furthermore, SLE may be associated with pleuritis and antinuclear antibodies. However, this patient does not have additional features associated with SLE, including malar or discoid rash, oral ulcers, or renal or neurologic involvement. This patient's prominent muscle weakness and Raynaud phenomenon also are more consistent with mixed connective tissue disease than SLE. Anti-Smith antibodies are highly specific for SLE but are associated with glomerulonephritis and central nervous system disease, which also are absent in this patient. Antimitochondrial antibodies are present in approximately 95% of patients with primary biliary cirrhosis, which occasionally may manifest as slightly elevated alanine and aspartate aminotransferase levels. However, elevated alkaline phosphatase levels are more typical of this condition, which also usually is associated with fatigue, pruritus, and hepatosplenomegaly. Patients with Sjögren syndrome often have anti-Ro/SSA antibodies, but this patient does not have the symptoms of keratoconjunctivitis sicca (dryness of the eyes and mouth) that characterize this condition. Anti-Ro/SSA antibodies also may be associated with SLE, particularly subacute cutaneous disease. Subacute cutaneous SLE manifests as a papulosquamous or annular rash that typically involves the neck, trunk, and extensor surfaces of the arms, which this patient does not have. Systemic sclerosis may manifest as Raynaud phenomenon associated with digital ulcerations. Antitopoisomerase I (anti-Scl-70) antibodies are present in approximately 30% of patients with systemic sclerosis and are particularly associated with diffuse skin disease. However, this patient has no additional signs of skin thickening characteristic of this condition or additional manifestations of systemic sclerosis, such as gastroesophageal involvement or basilar fibrosis visible on chest radiography. Furthermore, this patient's arthritis, serositis, and myositis are more suggestive of mixed connective tissue disease than systemic sclerosis.

Rheuma 26 A 70-year-old male dairy farmer is evaluated for a 1-year history of pain in the left knee that worsens with activity and is relieved with rest. On physical examination, vital signs are normal. A small effusion is present on the left knee, but there is no erythema or warmth. Range of motion of the left knee elicits pain and is slightly limited. Extension of this joint is limited to approximately 10 degrees, but flexion is nearly full. The remainder of the musculoskeletal examination is normal. The erythrocyte sedimentation rate is 15 mm/h. A standing radiograph of the left knee is shown . Which of the following is the most likely diagnosis? A Avascular necrosis B Osteoarthritis C Rheumatoid arthritis D Torn medial meniscus

B Physical examination findings consistent with osteoarthritis of the knee include crepitus, bony tenderness, bony enlargement, and an absence of palpable warmth. This patient most likely has osteoarthritis of the knee. He has two risk factors for this condition, advanced age and an occupation involving repetitive bending and physical labor. Osteoarthritis commonly affects weight-bearing joints such as the knees and is characterized by pain on activity that is relieved with rest. Swelling in patients with this condition is usually minimal, and range of motion may be limited. According to the American College of Rheumatology, osteoarthritis of the knee can be diagnosed if knee pain is accompanied by at least three of the following features: age greater than 50 years, morning stiffness lasting less than 30 minutes, crepitus, bony tenderness, bony enlargement, and an absence of palpable warmth. This patient's radiographic findings of osteophytes, joint-space narrowing, sclerosis, and cyst formation are typical of this condition. Arthrocentesis is not necessary to establish a diagnosis of osteoarthritis. Patients with avascular necrosis of the knee typically experience pain on weight bearing and may have a painful, limited range of motion. However, this condition also is associated with pain on rest and most commonly occurs in patients who use corticosteroids, have systemic lupus erythematosus, or consume excessive amounts of alcoholic beverages. Radiographs in patients with avascular necrosis usually reveal density changes; subchondral radiolucency; cysts; sclerosis; and, eventually, joint-space narrowing. Rheumatoid arthritis may be associated with a limited range of motion and joint-space narrowing visible on radiography. Patients with rheumatoid arthritis usually have symmetric arthritis that affects at least three joints as well as an elevated erythrocyte sedimentation rate and is associated with morning stiffness that persists for more than 30 minutes. In addition, rheumatoid arthritis also would not explain the presence of subchondral sclerosis and osteophytes on radiography. A torn medial meniscus would cause pain in the knee and can occur in the elderly in association with osteoarthritis. Patients with acute meniscal damage often describe a twisting injury with the foot in a weight-bearing position in which a popping or tearing sensation is felt, followed by severe pain; in addition, this condition is characterized by the sensation that the knee "locks" or "gives out."

Rheuma 59 A 54-year-old woman with rheumatoid arthritis is evaluated for a 3-month history of increased morning stiffness. She has not had joint swelling. She was diagnosed with rheumatoid arthritis 4 years ago and began treatment with methotrexate at that time; etanercept was added 1 year ago, and she began prednisone, 5 mg/d, 6 months ago. She also takes a folic acid supplement. Three weeks ago, she was diagnosed with superficial spreading melanoma on the leg. A wide excision was performed, and the surgical margins were clear. On physical examination, vital signs are normal. There is a small nodule on the left elbow. The right wrist has mild synovitis and a slightly decreased range of motion. Grip strength is normal, and the metacarpophalangeal and proximal interphalangeal joints are not swollen or tender to palpation. There is a surgical wound on the patient's right leg that is healing well. The remainder of the examination is normal. Which of the following is the most appropriate next step in this patient's treatment? A Begin leflunomide B Discontinue etanercept C Increase the prednisone dosage D No change in treatment

B Standard practice is to discontinue tumor necrosis factor α inhibitors in patients who develop cancers, including melanomas. There is evidence that tumor necrosis factor α (TNF-α) is active against some forms of cancer and that TNF-α inhibitors may increase the risk of cancer, particularly lymphoma, some solid cancers, and skin cancer. Patients with rheumatoid arthritis also have an increased risk of developing certain cancers, including leukemia and lymphoma, even in the absence of therapy with a TNF-α inhibitor. Therefore, establishing a causal relationship between cancer and TNF-α inhibitors is difficult. Studies attempting to demonstrate a relationship between TNF-α inhibitors and lymphoma and solid malignancies have yielded mixed results. In a study that used the National Data Bank for Rheumatic Diseases, the odds ratio for developing melanoma while taking a TNF-α inhibitor was approximately 2 but did not reach statistical significance. Nevertheless, standard practice based on expert opinion is to discontinue TNF-α inhibitor therapy in patients who develop cancers, including melanomas. The addition of leflunomide or an increase in the prednisone dosage would be warranted in a patient with rheumatoid arthritis who had signs or symptoms of disease activity, such as prolonged morning stiffness, fatigue, loss of function, swelling and tenderness of numerous joints, elevated levels of acute phase reactants, or progressive erosive changes visible on radiography. However, this patient's disease appears to be well controlled on her current medication regimen, and additional therapy would not be warranted.

Rheuma 69 A 35-year-old woman is evaluated in the office before the initiation of infliximab for rheumatoid arthritis. She was diagnosed with rheumatoid arthritis 5 years ago, and her disease is inadequately controlled on methotrexate and naproxen. She has no other complaints or medical problems and has no risk factors for tuberculosis. She has never been screened for tuberculosis. Her physical examination is unremarkable except for changes compatible with active rheumatoid arthritis involving her hands and feet. A chest radiograph is normal. Forty-eight hours after administering the PPD skin test, there is 5 mm of induration at the injection site. Which of the following is the most appropriate next step in this patient's management? A Infliximab B Isoniazid C Isoniazid and infliximab D Isoniazid, rifampin, pyrazinamide, and ethambutol

B The Centers for Disease Control and Prevention recommend treatment of latent tuberculosis infection for all patients planning to take a tumor necrosis factor α inhibitor who have a tuberculin skin test result of 5 mm or more of induration or a positive interferon-γ release assay. Adverse effects of tumor necrosis factor α (TNF-α) inhibitors include the risk for serious infection. Infliximab, adalimumab, and etanercept are associated with an increased incidence of reactivation tuberculosis, particularly extrapulmonary tuberculosis. Therefore, all patients being considered for such therapy should undergo screening for latent tuberculosis infection, which includes a full medical history, physical examination, tuberculin skin testing with purified protein derivative (PPD), or an interferon-γ release assay. If screening is positive, appropriate treatment for the patient's tuberculin status is indicated before beginning therapy with a TNF-α inhibitor. The Centers for Disease Control and Prevention recommends treatment of latent tuberculosis infection for all patients planning to take a TNF-α inhibitor who have a PPD result of 5 mm or more of induration or a positive interferon-γ release assay. Therefore, the most appropriate treatment for this patient is isoniazid for 9 months. Although the most appropriate duration of treatment with isoniazid before beginning infliximab is unknown, most experts recommend at least 2 months of isoniazid therapy before initiating a TNF-α inhibitor. The finding of a normal PPD reaction does not rule out the possibility of latent tuberculosis infection, as rheumatoid arthritis may be associated with false-negative results, particularly in patients taking immunosuppressive drugs. Whenever infliximab or another TNF-α inhibitor is initiated, a high index of suspicion for reactivation tuberculosis must be maintained even after negative screening for latent tuberculosis. Therefore, initiation of infliximab without isoniazid therapy or initiation of infliximab concomitantly with isoniazid therapy would not be recommended in a patient with positive results on PPD testing. Four-drug antituberculous therapy is indicated for active tuberculosis when a patient's drug resistance status is unknown but is not appropriate for this patient, who has no evidence of active tuberculosis.

Rheuma 18 A 55-year-old woman is evaluated for progressive polyarthralgia, photosensitive rash, and lower-extremity purpura of 7 weeks' duration. She also has daily low-grade fever and intermittent pleuritic chest pain. She has an 11-year history of rheumatoid arthritis treated with oral methotrexate and intravenous infliximab. Her disease has been mostly stable except for occasional flares treated with prednisone. On physical examination, vital signs are normal except for a temperature of 38.0 °C (100.4 °F). Malar rash is present. Cardiopulmonary examination reveals normal breath sounds, and no rubs are heard. She is unable to take a deep breath because of pain. Several small 1-cm maculopapular eruptions are visible on the lower extremities bilaterally. Musculoskeletal examination reveals synovitis of the metacarpophalangeal and proximal interphalangeal joints and the wrists bilaterally. The left elbow has a nodule. Range of motion of the right wrist is decreased. Laboratory studies: Rheumatoid factor Positive Antinuclear antibodies Titer of 1:640 Anti-double-stranded DNA antibodies Positive Chest radiograph reveals small bilateral pleural effusions. Which of the following is the most appropriate next step in this patient's treatment? A Add sulfasalazine B Discontinue infliximab; begin prednisone C Discontinue methotrexate; begin hydroxychloroquine D Discontinue methotrexate; begin sulfasalazine

B The most appropriate management of a patient with drug-induced lupus caused by a tumor necrosis factor α inhibitor is discontinuation of the offending agent. This patient most likely has drug-induced lupus caused by the tumor necrosis factor α inhibitor infliximab. The most appropriate next step in this patient's management is to discontinue infliximab and begin prednisone. Many patients who use tumor necrosis factor α inhibitors develop autoantibodies, including antinuclear, anti-double-stranded DNA, and anti-Smith antibodies; rarely, these patients develop drug-induced lupus. Patients with this condition may present with typical manifestations of systemic lupus erythematosus but are particularly likely to have cutaneous and pleuropericardial involvement. Renal and neurologic manifestations are extremely rare. The most appropriate management of a patient with drug-induced lupus caused by a tumor necrosis factor α inhibitor is discontinuation of the offending agent, which usually resolves this condition. Prednisone also should be added to this patient's medication regimen to control pleuritis and synovitis associated with drug-induced lupus. Although this patient's worsening joint symptoms may be related to her underlying rheumatoid arthritis, her rheumatoid arthritis had been well controlled on her current medication regimen. If her flare were related to active rheumatoid arthritis, her symptoms would most likely be alleviated by initiation of sulfasalazine or an increase in her infliximab dosage. However, her musculoskeletal features, fever, malar rash, photosensitivity, purpura, symptoms of pleuritis, antinuclear and anti-double-stranded DNA antibody positivity, and findings on chest radiography also raise strong suspicion for drug-induced lupus. Therefore, progressive rheumatoid arthritis is a less likely explanation for this patient's current symptoms than is drug-induced lupus, and initiation of sulfasalazine or an increase in her infliximab dosage would not be indicated. Hydroxychloroquine may be useful for the treatment of systemic lupus erythematosus and drug-induced lupus and could be added to this patient's existing medication regimen, but discontinuing methotrexate would not be appropriate.

Rheuma 39 A 44-year-old man is evaluated for a 2-year history of joint pain in the wrists, hands, knees, ankles, and feet. He also has a 12-year history of plaque psoriasis. His current medications include acetaminophen for joint pain, topical hydrocortisone and calcipotriene ointments, and a tar-based shampoo. He does not drink alcoholic beverages and has no history of liver disease. On physical examination, vital signs are normal. There are large erythematous plaques with an overlying silvery scale on the elbows, knees, sacrum, and scalp. The wrists are swollen and warm bilaterally, and there is bilateral synovitis of the metacarpophalangeal joints. There are effusions on both knees. The appearance of the toes is shown . Screening laboratory tests, including metabolic profile, renal function, and liver chemistry tests, are normal. After discussing medication options, the patient refuses to begin therapy with tumor necrosis factor α inhibitors because of concern about possible side effects. Which of the following is the most appropriate treatment for this patient? A Hydroxychloroquine B Ibuprofen C Methotrexate D Prednisone E Sulfasalazine

C In patients with psoriatic arthritis, methotrexate helps to control cutaneous and musculoskeletal manifestations, slows the progression of joint destruction, and is considered first-line therapy for this condition. This patient has psoriasis, dactylitis, and symmetric peripheral polyarthritis involving the small joints. This constellation of symptoms raises strong suspicion for psoriatic arthritis. Methotrexate is the most appropriate treatment for this patient. Psoriatic arthritis affects 20% to 40% of patients with psoriasis and is most likely to develop in patients with psoriasis who have extensive skin involvement. Patients with psoriatic arthritis also may have arthritis solely involving the distal interphalangeal joints; asymmetric oligoarthritis; arthritis mutilans; spondyloarthritis that is often associated with peripheral joint disease; spinal involvement that manifests as inflammatory pain and stiffness of the neck and lower back; and tendinitis of the Achilles tendon, flexor tendons of the fingers, and tendons around the ankles. Despite an absence of evidence obtained from clinical trials, methotrexate has dominated therapy in patients with psoriatic arthritis for many years and is effective in those who have low risk for bone marrow or liver toxicity. In patients with psoriatic arthritis, methotrexate helps to control cutaneous and musculoskeletal manifestations; long-term therapy with this agent also is well tolerated. Hydroxychloroquine would be helpful in a patient with rheumatoid arthritis, and this patient's pattern of musculoskeletal involvement is somewhat consistent with this condition. However, rheumatoid arthritis usually spares the distal interphalangeal joints and involves the proximal interphalangeal joints. This patient also has dactylitis, which is characterized by inflammation of the whole digit resulting from joint and tendon inflammation that is typical for psoriatic arthritis but not associated with rheumatoid arthritis. Furthermore, hydroxychloroquine has been associated with severe flares of psoriasis and should be avoided in patients with this condition. Prednisone therapy may help to relieve this patient's joint symptoms. However, tapering of corticosteroids in patients with psoriatic arthritis may be associated with dramatic flares in skin disease; these agents therefore should be used with caution in this population group. NSAIDs such as ibuprofen may help to alleviate this patient's pain and swelling involving the joints but are believed to similarly exacerbate psoriasis. Furthermore, neither prednisone nor ibuprofen would slow the progression of this patient's articular damage. Sulfasalazine has modest efficacy in the treatment of joint disease in patients with psoriatic arthritis, and a meta-analysis showed that it may help to relieve skin disease, as well. However, this agent is less effective than methotrexate for both skin and joint disease associated with psoriatic arthritis.

Rheuma 31 A 65-year-old man is evaluated for a 2-year history of increasing weakness of the hands and legs. Medical history is insignificant, and he takes no medications or vitamin supplements. On physical examination, vital signs are normal. Cardiopulmonary examination is unremarkable. There are red, flaking patches predominantly involving the scalp, eyebrows, nasolabial folds, ears, and chest. Strength in the left wrist flexor, left hip flexor, and right quadriceps muscles is decreased. The remainder of the physical examination is normal. Laboratory studies reveal a serum creatine kinase level of 320 U/L. Electromyography demonstrates a mixed pattern of myopathic and neurogenic changes. Muscle biopsy specimen shows endomysial inflammation and basophilic vacuoles. On electron microscopy, the vacuoles contain particles resembling myxovirus or paramyxovirus. Which of the following is the most likely diagnosis? A Dermatomyositis B Fibromyalgia C Inclusion body myositis D Mixed connective tissue disease E Polymyositis

C Inclusion body myositis typically manifests as the insidious onset of asymmetric proximal and distal muscle weakness associated with only moderately elevated creatine kinase levels. This patient most likely has inclusion body myositis (IBM), which is the most common form of myositis in patients older than 60 years. This condition typically manifests as muscle weakness that affects the proximal and distal muscles and often has an asymmetric pattern of involvement. The quadriceps, wrist, and finger flexor muscles are commonly involved. IBM characteristically has an insidious onset and is associated with only moderately elevated serum creatine kinase levels that are typically less than 1000 U/L. The presence of mixed myopathic and neurogenic findings on electromyography is characteristic of this condition. IBM responds poorly to immunosuppressive therapy that is usually effective in inflammatory myositis. Dermatomyositis and polymyositis also have an insidious onset and manifest as muscle weakness often in the absence of pain that typically causes progressive difficulty with routine activities, such as rising from a chair, climbing stairs, or hair combing. These conditions are characterized by symmetric proximal muscle weakness, whereas this patient has an asymmetric pattern of involvement that also prominently affects the distal muscles. Furthermore, although dermatomyositis and polymyositis may be associated with serum creatine kinase levels that range from normal to several thousand U/L, a creatine kinase level of 320 U/L would be more typical of IBM. This patient's cutaneous findings are most consistent with seborrheic dermatitis, which is not associated with dermatomyositis. Mixed connective tissue disease is characterized by features of systemic lupus erythematosus, inflammatory myopathy, and systemic sclerosis. This patient's absence of sclerodactyly, Raynaud phenomenon, digital pitting, serositis, and joint pain makes this an unlikely diagnosis. The central features of fibromyalgia are widespread musculoskeletal pain with localized tenderness unexplained by any other diagnosis as well as sleep disturbance and fatigue. IBM is not associated with fibromyalgia. Furthermore, fibromyalgia would not explain this patient's muscle weakness and elevated serum creatine kinase level. Finally, polymyositis, dermatomyositis, mixed connective tissue disease, or fibromyalgia would not explain this patient's electromyographic or biopsy findings.

Rheuma 75 A 24-year-old woman is evaluated for a 2-week history of persistent pain and swelling in the right foot and knee and the left heel. One month ago, she developed an episode of conjunctivitis that resolved spontaneously. She also had an episode of severe diarrhea 2 months ago while traveling to Central America that was successfully treated with a 3-day course of ciprofloxacin and loperamide. She has not had other infections of the gastrointestinal or genitourinary tract, rash, or oral ulcerations. Her weight has been stable, and she has not had abdominal pain, blood in the stool, or changes in her bowel habits. She has had only one sexual partner 6 years ago. She otherwise feels well, has no other medical problems, and takes no medications other than acetaminophen for joint pain. On physical examination, vital signs, including temperature, are normal. Cutaneous examination, including the nails and oral mucosa, is normal. There is no evidence of conjunctivitis or iritis. Musculoskeletal examination reveals swelling, warmth, and tenderness of the right knee and ankle. There is tenderness to palpation at the insertion site of the left Achilles tendon. Which of the following is the most likely diagnosis? A Enteropathic arthritis B Psoriatic arthritis C Reactive arthritis D Rheumatoid arthritis

C Reactive arthritis is characterized by the presence of inflammatory arthritis that manifests within 2 months of an episode of bacterial gastroenteritis or nongonococcal urethritis or cervicitis in a genetically predisposed patient. This patient most likely has reactive arthritis, which may be associated with plantar fasciitis and Achilles tendinitis. Reactive arthritis is characterized by the presence of inflammatory arthritis that manifests within 2 months of an episode of bacterial gastroenteritis or nongonococcal urethritis or cervicitis in a genetically predisposed patient. This patient's history of conjunctivitis preceding her musculoskeletal symptoms is also consistent with this condition. Historically, what is now called reactive arthritis was previously called Reiter syndrome, which referred to the co-incidence of arthritis, conjunctivitis, and urethritis (or cervicitis). However, only one third of affected patients have all three symptoms. Reactive arthritis usually has an asymmetric oligoarticular pattern and involves the lower extremities; inflammatory back pain also may be present. Reactive arthritis also may be associated with enthesitis, which commonly manifests as Achilles tendinitis (as seen in this patient), plantar fasciitis, and spondylitis. Reactive arthritis also is associated with numerous cutaneous manifestations, including keratoderma blenorrhagicum, oral ulcerations, and circinate balanitis. Up to 20% of patients with Crohn disease or ulcerative colitis develop inflammatory joint disease. Polyarthritis that resembles seronegative rheumatoid arthritis develops in 20% of these patients, whereas 10% to 15% of these patients develop spondylitis. Joint involvement also may manifest as chronic or intermittent asymmetric oligoarthritis. Risk for inflammatory joint disease associated with Crohn disease or ulcerative colitis increases in patients with more advanced colonic conditions and additional concomitant extraintestinal manifestations, including abscesses, erythema nodosum, uveitis, or pyoderma gangrenosum. The course of peripheral arthritis parallels intestinal involvement except in those with spinal disease. The absence of bowel symptoms except for the initial infection, stable weight, and lack of cutaneous manifestations argue against enteropathic arthritis as the cause of this patient's findings. Psoriatic arthritis is a systemic chronic inflammatory arthritis associated with numerous clinical manifestations. Asymmetric oligoarthritis of the large joints of the lower extremities is present in 40% of patients with this condition, whereas 25% of these patients develop small-joint polyarthritis similar to rheumatoid arthritis. Psoriatic arthritis also may manifest as uveitis, tendinitis, and enthesitis. Cutaneous involvement may be limited to nail pitting and commonly precedes joint inflammation, although 15% of affected patients develop joint inflammation first. However, in the absence of skin findings, this patient's preceding gastrointestinal infection most strongly suggests the diagnosis of reactive arthritis. Rheumatoid arthritis is a symmetric polyarthritis that involves the small joints of the hands and feet as well as other joints throughout the body. The subacute onset of asymmetric oligoarthritis following a gastrointestinal infection is not characteristic of this condition.

Rheuma 76 A 19-year-old woman is evaluated for a 5-day history of pain and swelling of the cartilage of the left ear. There is no discharge from the ear, and she has not had fever. Three months ago, she developed redness and pain 1 week after getting the cartilage of the right ear pierced; at that time, she was prescribed a 10-day course of amoxicillin-clavulanate, and her symptoms resolved. She pierced her left nostril and umbilicus 9 months ago but did not develop any unusual symptoms. On physical examination, temperature is 37.7 °C (99.8 °F), blood pressure is 110/72 mm Hg, pulse rate is 88/min, and respiration rate is 18/min. The left pinna is swollen, red, tender, and warm; the lobe of the left ear is spared. The conjunctivae and tympanic membranes are not injected, and the sclerae and oropharynx are clear. Cardiovascular examination reveals a normal S1 and S2 and no murmurs. The lungs are clear. Abdominal and musculoskeletal examinations are normal. There is no peripheral edema. Which of the following is the most likely diagnosis? A Churg-Strauss syndrome B Cogan syndrome C Relapsing polychondritis D Wegener granulomatosis

C Relapsing polychondritis is characterized by inflammation of the cartilage that commonly involves the nose, ears, trachea, and joints. This patient most likely has relapsing polychondritis. This condition is characterized by inflammation of the cartilage that commonly involves the nose, ears, trachea, and joints. Auricular involvement typically manifests as redness, swelling, and tenderness; rarely, hearing loss also may occur. Local trauma associated with cartilage piercing infrequently may trigger chondritis in these patients. Relapsing polychondritis may have a relapsing-remitting disease course with self-limited episodes that often resolve without specific therapy and are often misdiagnosed as infection. In this patient, corticosteroids would help to manage her auricular symptoms. Relapsing polychondritis also may manifest as large-airway disease, which has an acute, severe presentation and can cause airway collapse and respiratory failure. Therefore, patients with relapsing polychrondritis should undergo evaluation for end-organ involvement, including upper-airway, and valvular manifestations. Screening for upper-airway involvement should include spirometry and flow volume loop, whereas echocardiography would help to evaluate for valvular manifestations. Churg-Strauss syndrome and Wegener granulomatosis are small- to medium-vessel granulomatous vasculitides. Although Churg-Strauss syndrome may cause destruction of the nasal cartilage and Wegener granulomatosis frequently causes nasal cartilage inflammation resulting in saddle nose deformity, auricular cartilage involvement is not typical of either of these conditions. Cogan syndrome is an autoimmune inflammatory disorder characterized by nonsyphilitic keratitis and audiovestibular dysfunction that results in progressive hearing loss. However, this condition is not characterized by inflammation of the cartilage.

Rheuma 65 A 34-year-old man is evaluated in the emergency department for progressively worsening shortness of breath that began this morning; he is accompanied by his wife. He is unable to provide a history due to respiratory distress. His wife states that he developed redness of the eyes approximately 6 months ago that resolved with the use of prescription eye drops but has otherwise been healthy. He currently takes no medications. On physical examination, he is in marked respiratory distress and is using accessory muscles to breathe. Temperature is 37.1 °C (98.8 °F), blood pressure is 165/98 mm Hg, pulse rate is 135/min, and respiration rate is 50/min. Cardiac examination is normal. Tracheal stridor is heard, and tenderness is noted over the thyroid cartilage and anterior trachea. The lungs are clear to auscultation. There is no skin rash, conjunctival injection, or jaundice. The oropharynx is clear. The abdomen is soft and nontender, and there is no organomegaly. Musculoskeletal examination reveals large knee effusions bilaterally. Laboratory studies: Hemoglobin 14.8 g/dL (148 g/L) Leukocyte count 17,000/µL (17 × 109/L) Erythrocyte sedimentation rate 100 mm/h Urinalysis Normal Chest radiograph is normal, with no cardiomegaly and no pulmonary infiltrates. Anesthesia staff are standing by for intubation, if needed. Which of the following is the most likely diagnosis? A Goodpasture syndrome B Polymyositis C Relapsing polychondritis D Rheumatoid arthritis E Systemic sclerosis

C Relapsing polychondritis may manifest acutely as respiratory failure associated with tracheal and/or airway collapse. This patient most likely has relapsing polychondritis, an inflammatory disorder of the cartilage. Diagnosis of this condition is based on the presence of inflammation in at least two cartilaginous sites (such as the ears, nose, or trachea) or the presence of inflammation at one site accompanied by at least two of the following features that need not be present at the same time: ocular inflammation, audiovestibular manifestations, and/or inflammatory arthritis. Typical laboratory abnormalities in patients with relapsing polychondritis include an elevated erythrocyte sedimentation rate and C-reactive protein level. This patient's respiratory failure is consistent with one of the most severe forms of relapsing polychondritis, large-airway disease. Large-airway disease that includes the larynx, trachea, and bronchi may begin subtly and, if undiagnosed, may evolve into life-threatening disease. His tracheal tenderness and stridor are suggestive of tracheal inflammation and subglottic stenosis, which often cause tracheal and/or airway collapse in patients with severe manifestations of relapsing polychondritis. In addition, this patient's eye redness that resolved with eye drops is most likely a manifestation of inflammatory eye disease, which develops in up to 50% of patients with relapsing polychondritis. Goodpasture syndrome is a pulmonary-renal syndrome characterized by the presence of anti-glomerular basement membrane antibodies. Affected patients commonly develop pulmonary hemorrhage and a rapidly progressive glomerulonephritis. Goodpasture syndrome may manifest as acute respiratory failure and may be associated with an elevated erythrocyte sedimentation rate and leukocyte count. However, this condition typically causes cough, hemoptysis, and fever and is unlikely in a patient with a normal chest radiograph and an absence of renal involvement. Patients with rheumatoid arthritis can present with respiratory manifestations, and involvement of the cricoarytenoid joint in particular may cause hoarseness, stridor, and potential upper-airway respiratory distress. However, extra-articular disease is not usually the presenting manifestation of rheumatoid arthritis. Systemic sclerosis is characterized by visceral fibrosis and vascular manifestations such as Raynaud phenomenon, whereas polymyositis is an inflammatory muscle disease characterized by proximal muscle weakness. One of the leading causes of mortality in patients with systemic sclerosis is pulmonary involvement, which most commonly manifests as interstitial lung disease (ILD) or pulmonary artery hypertension; polymyositis also may be associated with ILD. However, this patient does not have features compatible with ILD, such as dry cough, crackles heard on pulmonary examination, or bibasilar infiltrates visible on chest radiographs. His presentation also is not consistent with pulmonary artery hypertension, which may manifest as fatigue, dyspnea, syncope, a loud P2 on cardiac auscultation, and central pulmonary artery enlargement seen on chest radiographs. In addition, pulmonary involvement in a patient with systemic sclerosis or polymyositis would have a more gradual onset and would not cause acute respiratory failure

Rheuma 32 A 62-year-old man is evaluated for a 2-year history of enlargement and discomfort of the metacarpophalangeal joints of both hands. He works as a bank manager and leads a sedentary lifestyle. He does not have morning stiffness. He was diagnosed with type 2 diabetes mellitus 3 months ago for which he takes metformin. On physical examination, vital signs are normal. Examination of the skin reveals generalized hyperpigmentation. There is bony enlargement of the metacarpophalangeal joints bilaterally but no evidence of synovial proliferation. Range of motion of the hands is full, and he can make a strong fist. Examination of the proximal and distal interphalangeal joints, knees, and hips is normal. Laboratory studies: Hemoglobin A1c 7.3% Erythrocyte sedimentation rate 13 mm/h Glucose (fasting) 100 mg/dL (5.6 mmol/L) Rheumatoid factor Negative Anti-cyclic citrullinated peptide antibodies Negative Hand radiographs show joint-space narrowing and hook-shaped osteophyte formation in the metacarpophalangeal joints. Radiographs of the hips and knees are normal. These imaging studies reveal no evidence of chondrocalcinosis. Which of the following is the most likely diagnosis? A Calcium pyrophosphate deposition disease B Diabetic cheiroarthropathy (stiff hand syndrome) C Hemochromatosis D Primary osteoarthritis E Rheumatoid arthritis

C Secondary osteoarthritis usually involves joints not affected by primary osteoarthritis and develops because of another condition, such as trauma, previous inflammatory arthritis, or metabolic disorders such as hemochromatosis or chondrocalcinosis. This patient most likely has secondary osteoarthritis associated with hemochromatosis. Approximately 40% to 60% of patients with hemochromatosis develop an arthropathy with a presentation similar to that of osteoarthritis. The presence of symmetric pain and bony enlargement of the joints accompanied by radiographic findings of joint-space narrowing and osteophytes is consistent with osteoarthritis. However, primary osteoarthritis does not typically involve the metacarpophalangeal joints; if this occurs, suspicion should be raised for secondary osteoarthritis. Similarly, radiographs of the metacarpophalangeal joints may reveal hook-shaped osteophytes that are significantly different from radiographs of patients with primary osteoarthritis. Hemochromatosis arthropathy also may involve the proximal interphalangeal joints and, less frequently, the shoulders, hips, knees, and ankles. Finally, primary osteoarthritis usually affects patients with advanced age or who have occupations involving repetitive bending or manual labor. Secondary osteoarthritis usually involves joints not affected by primary osteoarthritis. Secondary arthritis develops because of another condition, such as trauma, previous inflammatory arthritis, or metabolic disorders such as hemochromatosis or chondrocalcinosis. In this patient, the presence of skin hyperpigmentation and diabetes mellitus raises strong suspicion for hemochromatosis, which is particularly associated with involvement of the metacarpophalangeal joints in patients without primary osteoarthritis. Symptoms of osteoarthritis that involve the second and third metacarpophalangeal joints also may be caused by calcium pyrophosphate deposition disease. However, radiographs of patients with this condition would typically reveal chondrocalcinosis, which occurs most frequently in the knees, symphysis pubis, and triangular fibrocartilage of the wrist. Diabetic cheiroarthropathy (stiff hand syndrome) more commonly occurs in patients with long-standing diabetes. This condition manifests as joint stiffness, limited range of motion in the absence of pain, and skin thickening of the fingers, which is not compatible with this patient's presentation or radiographic findings. Rheumatoid arthritis may involve the metacarpophalangeal joints in a symmetric pattern and may be present in patients without rheumatoid factor. This condition also may manifest as rheumatoid nodules (subcutaneous nodules that develop over bony prominences at sites such as the extensor surfaces of the hand) that may resemble the bony enlargement associated with osteoarthritis. However, rheumatoid arthritis is unlikely in the absence of morning stiffness and joint swelling.

Rheuma 47 A 75-year-old woman is evaluated during a follow-up examination for severe pain associated with osteoarthritis of the knee. For the past 10 years, she has had gradually progressive right knee pain. Originally, her pain increased with activity and was relieved with rest; however, she now also has discomfort at rest, which disturbs her sleep. Acetaminophen, ibuprofen, naproxen, and celecoxib have been ineffective in controlling her pain; tramadol provided only modest pain relief. Three months ago, she received an intra-articular corticosteroid injection, which alleviated her pain for 1 week. Medical history is significant for hypertension and compensated heart failure. She also takes metoprolol, lisinopril, low-dose aspirin, and furosemide. Currently, the knee pain is the single condition that is impairing the quality of her life. On physical examination, there is a 15-degree varus angulation of the right knee in the standing position. There is no effusion or other signs of inflammation. Standing radiographs of the right knee show severe medial joint-space narrowing and mild lateral joint-space narrowing. Which of the following is the most appropriate next step in the management of this patient's knee pain? A Add diclofenac B Add hydrocodone-acetaminophen C Refer for total knee arthroplasty D Repeat intra-articular corticosteroid injection

C Total knee arthroplasty is warranted only when no further medical therapy is available and the patient decides that the impairment caused by his or her condition warrants this procedure. Trials of acetaminophen and two NSAIDs did not relieve this patient's pain, tramadol provided only modest pain relief, and an intra-articular corticosteroid injection only alleviated her pain for 1 week. Moreover, the presence of bone-on-bone contact on radiography and pain even on rest suggests that she is unlikely to achieve significant pain relief from additional nonsurgical therapy. The most appropriate next step in this patient's management is total knee arthroplasty, which is appropriate only when no further medical therapy is available and the patient decides that the impairment caused by his or her condition warrants this procedure. Diclofenac is unlikely to effectively manage this patient's pain. Previous trials of several NSAIDs have not been beneficial. Furthermore, this patient's age and comorbidities pose the potential for complications associated with NSAID use, including the risk for renal and gastrointestinal toxicity. These agents also may exacerbate heart failure and should be used with caution in patients with hypertension. The American College of Rheumatology guidelines recommend opioid analgesics as second-line agents in the treatment of osteoarthritis. These agents are particularly useful in patients with osteoarthritis of the hip or knee who are intolerant of or who do not achieve adequate pain control using acetaminophen and/or NSAIDs. However, tramadol was already tried in this patient and was not beneficial. Repeat administration of intra-articular corticosteroid injection would be unlikely to sufficiently benefit a patient with such severe osteoarthritis. In addition, a previous intra-articular corticosteroid injection provided only transient relief.

Pneumo 6 A 25-year-old woman is evaluated for a 2-week history of bilateral lower-extremity pain and skin lesions that she describes as "red knots." Ibuprofen has not alleviated her discomfort, and she has continued to develop new skin lesions. Six months ago, she developed vulvar ulcers that were negative for herpes simplex virus on a polymerase chain reaction assay; these lesions healed within 3 weeks. Two years ago, she developed uveitis that was treated with prednisolone drops. She also has a 7-year history of Raynaud phenomenon and a long-standing history of recurrent oral ulcers. She has had no recent infections and currently has no vulvar ulcers. On physical examination, vital signs and cardiopulmonary and abdominal examinations are normal. There is no conjunctival injection. There are two ulcers on her tongue. Cutaneous examination reveals several subcutaneous reddish-colored nodules that are tender to palpation located on the lower extremities bilaterally. There is no synovitis, and range of motion of all joints is full. Laboratory studies: Complete blood count Normal Erythrocyte sedimentation rate 95 mm/h Metabolic panel Normal Rheumatoid factor Negative Antinuclear antibodies Negative ANCA Negative Urinalysis Normal A chest radiograph is normal. Which of the following is the most appropriate treatment for this patient? A Leflunomide B Penicillin C Prednisone D Sulfasalazine

C Treatment in patients with Behçet disease is directed toward the involved organ system. This patient's lower-extremity lesions are consistent with erythema nodosum. She also has oral ulcers, an elevated erythrocyte sedimentation rate, and a history of genital mucosal ulcers and uveitis. This clinical presentation strongly suggests a diagnosis of Behçet disease, and the most appropriate management for this patient is prednisone. Diagnostic criteria for Behçet disease consist of the presence of oral ulcerations that recur at least three times in 1 year and at least two of the following manifestations: recurrent genital ulcerations, inflammatory eye disease, cutaneous lesions, and positive results on a pathergy test (characterized by a papule developing 48 hours after insertion of a 20-gauge needle intradermally). Patients with Behçet disease also may have central nervous system vasculitis; a nonerosive arthritis that involves the medium and large joints; and elevated markers of inflammation, such as the erythrocyte sedimentation rate. Other manifestations include arterial and venous thromboses, and patients with Behçet disease have a high mortality rate associated with arterial aneurysm rupture. Treatment of Behçet disease is directed toward the involved organ system. Although erythema nodosum can be treated with NSAIDs, use of these agents has not alleviated this patient's symptoms. Treatment with a corticosteroid such as prednisone is therefore reasonable. Leflunomide and sulfasalazine are indicated to treat rheumatoid arthritis but are not useful in patients with erythema nodosum or Behçet disease. Rheumatoid arthritis may present with Raynaud phenomenon and subcutaneous nodules. However, rheumatoid nodules typically develop on pressure points and are not tender or erythematous. Rheumatoid arthritis also is characterized by swelling and tenderness in and around the joints and may manifest as joint stiffness, synovial hypertrophy, synovitis, joint effusion, and loss of normal range of motion. These findings are absent in this patient. Sulfasalazine also is used to treat inflammatory bowel disease, which may be associated with erythema nodosum. However, this patient has no gastrointestinal manifestations.

Rheuma 42 A 53-year-old man is evaluated for a 6-week history of fatigue, fever, numbness and tingling in the hands and feet, mild abdominal pain, and a nodular rash on the hands, arms, and legs. He also has lost 4.1 kg (9.0 lb). On physical examination, temperature is 37.9 °C (100.3 °F), blood pressure is 150/82 mm Hg, pulse rate is 96/min, and respiration rate is 14/min. Cardiopulmonary examination is normal. Examination of the hands reveals numerous subcutaneous nodules. There is shotty cervical, axillary, and inguinal lymphadenopathy. Abdominal examination reveals hepatomegaly and mild diffuse abdominal tenderness without rebound. The testes are tender. There is no synovitis on musculoskeletal examination. Laboratory studies: Hemoglobin 8.6 g/dL (86 g/L) Leukocyte count 15,200/µL (15.2 × 109/L) Platelet count 523,000/µL (523 × 109/L) Erythrocyte sedimentation rate 113 mm/h Alanine aminotransferase 73 U/L Aspartate aminotransferase 85 U/L Antinuclear antibodies Negative ANCA Negative Hepatitis B surface antigen (HBsAg) Positive Antibody to hepatitis B surface antigen (anti-HBs) Negative Hepatitis B e antigen (HBeAg) Positive Antibody to hepatitis C virus (anti-HCV) Negative Serologic test for HIV antibodies Negative Skin biopsy specimen is consistent with necrotizing vasculitis. Which of the following is the most appropriate treatment for this patient? A Cyclophosphamide B Prednisone C Prednisone plus cyclophosphamide D Prednisone plus lamivudine

D Patients with polyarteritis nodosa in the acute setting who also have hepatitis B virus infection should undergo short-term high-dose corticosteroid therapy accompanied by long-term treatment with an antiviral agent. This patient's constitutional symptoms, abdominal pain, symptoms of peripheral neuropathy, and skin biopsy specimen showing necrotizing vasculitis are consistent with polyarteritis nodosa. Testicular tenderness also is a particularly specific finding for polyarteritis nodosa. The most appropriate treatment for this patient is prednisone, 60 mg/d tapered over 2 weeks, plus lamivudine, 100 mg/d. Additional features of polyarteritis nodosa may include renovascular hypertension, and cutaneous polyarteritis nodosa may manifest as nodules, ulcers, purpura, or livedo reticularis. Anemia, leukocytosis, and an elevated erythrocyte sedimentation rate are typical laboratory findings in patients with polyarteritis nodosa. Involvement of the renal arterioles, gastrointestinal tract, and heart may occur. This patient also has hepatitis B virus infection, which is present in approximately 50% of patients with polyarteritis nodosa; rarely, patients with infection with hepatitis C virus or HIV may have polyarteritis nodosa. In patients with polyarteritis nodosa who do not have active hepatitis B virus infection, high-dose corticosteroid therapy is indicated. The presence of systemic involvement is a poor prognostic sign and typically warrants treatment with cyclophosphamide, as well. In patients with polyarteritis nodosa who have concomitant active hepatitis B virus infection, however, the use of potent immunosuppressive agents can increase the viral load and worsen viral infection. Therefore, these agents should be avoided if possible. Patients with polyarteritis nodosa in the acute setting who also have hepatitis B virus infection should undergo only short-term (typically 2 weeks) high-dose corticosteroid therapy in order to help decrease inflammation and relieve symptoms. Because patients with both polyarteritis nodosa and hepatitis B virus infection typically achieve remission of their vasculitis when seroconversion from hepatitis B e antigen positivity to hepatitis B e antibody positivity occurs, short-term high-dose corticosteroid therapy should be accompanied by long-term treatment with an antiviral agent, such as lamivudine.

Rheuma 50 A 52-year-old woman is evaluated during a follow-up office visit. She has hypercholesterolemia that was treated with atorvastatin for 5 years. Six months ago, she developed achy thigh muscles. At that time, laboratory studies revealed a serum creatine kinase level of 430 U/L, and she was switched to cholestyramine. Since then, her creatine kinase levels have remained elevated. Her last evaluation 2 months ago revealed a creatine kinase level of 448 U/L. She has remained minimally symptomatic with myalgia during this time. She does not drink alcoholic beverages, and her only large muscle exercise includes walking approximately 1 mile daily. There is no family history of muscle disorders. On physical examination, vital signs are normal. BMI is 31. There is no rash. On musculoskeletal examination, proximal and distal muscle strength is intact. Neurologic examination is normal. Laboratory studies reveal a serum creatine kinase level of 550 U/L and a normal serum thyroid-stimulating hormone level. Which of the following diagnostic studies would be warranted for this patient? A Aldolase B Antinuclear antibody C Erythrocyte sedimentation rate D MRI of the thigh muscles

D MRI of the proximal muscles, particularly the thighs, is helpful in patients with persistently elevated creatine kinase levels to assess whether muscle inflammation is present. MRI of the thigh muscles is warranted for this patient. For more than 6 months, this patient has had minor myalgia and a mildly elevated creatine kinase level despite discontinuation of atorvastatin. Statin-related myopathy usually begins within weeks to months after starting the inciting medication and may manifest as myalgia, weakness, and an elevated creatine kinase level. Some affected patients develop myalgia without elevations of the creatine kinase level. In most cases, the symptoms and laboratory abnormality associated with this condition resolve within a few days or weeks of stopping the inciting medication. Approximately 95% of cases of this condition resolve within 6 months. This patient discontinued atorvastatin 6 months ago and does not have a history of excessive alcohol use, which may cause myopathy. Her normal thyroid-stimulating hormone level excludes hypothyroidism. In this setting, MRI of the proximal muscles, particularly the thighs, would help to assess whether muscle inflammation is present and to help localize a biopsy site. Unlike electromyography, MRI can screen only a limited number of muscle groups. Nevertheless, MRI of the involved muscle groups, particularly the thighs, is being increasingly employed to initially evaluate a suspected myositis and to monitor disease progression. On MRI, actively inflamed muscle groups demonstrate increased edema within the muscle; this finding is not specific for myositis but does suggest a myopathic process. MRI also is a noninvasive means of localizing a muscle for biopsy. However, the role of MRI in patients with an inflammatory myopathy is still being defined, and the results of this study should be interpreted within the clinical context and should be considered complementary to results of electromyography and nerve conduction velocity studies. Patients with active muscle disease may have an elevated aldolase level, but this enzyme is less specific for muscle damage than creatine kinase. Furthermore, the results of aldolase measurement would not influence the need to evaluate this patient for inflammatory muscle disease. Approximately 80% of patients with an inflammatory myopathy have positive titers of antinuclear antibodies. Therefore, the absence of these antibodies would not exclude myositis, and the presence of antinuclear antibodies is not diagnostic of active muscle disease. Similarly, patients with myositis may have an elevated erythrocyte sedimentation rate, but this finding is a nonspecific marker of inflammation and is not used to diagnose an inflammatory myopathy or to monitor disease activity.

Rheuma 37 A 52-year-old woman is evaluated for a 4-day history of swelling and pain of the left ankle. She has a 6-year history of Crohn disease associated with joint involvement of the knees and ankles. Her last disease flare was 2 years ago; at that time, she was treated with a 3-month course of tapering prednisone and infliximab. She has continued taking infliximab. She also has been on azathioprine for 3 years. On physical examination, temperature is 38.0 °C (100.5 °F), pulse rate is 88/min, and respiration rate is 18/min. The left ankle is warm and swollen, and passive range of motion of this joint elicits pain. The knees are mildly tender to palpation bilaterally but do not have effusions, warmth, or erythema. Range of motion of the knees elicits crepitus bilaterally. The remainder of the musculoskeletal examination is normal. Arthrocentesis of the left ankle is performed and yields 3 mL of cloudy yellow fluid. The synovial fluid leukocyte count is 75,000/µL (92% neutrophils). Polarized light microscopy of the fluid shows no crystals, and Gram stain is negative. Culture results are pending. Which of the following is the most likely diagnosis? A Avascular necrosis of the ankle B Crohn disease arthropathy C Crystal-induced arthritis D Septic arthritis

D A diagnosis of septic arthritis should be considered in patients who present with the sudden onset of monoarthritis or the acute worsening of chronic joint disease and an increase in the synovial fluid leukocyte count. This patient has acute monoarticular arthritis associated with warmth and swelling of the involved joint, an inflammatory synovial fluid, and fever. This clinical presentation raises strong suspicion for septic arthritis. A diagnosis of septic arthritis should be considered in any patient who presents with the sudden onset of monoarthritis or the acute worsening of chronic joint disease and an increase in the synovial fluid leukocyte count. The synovial fluid leukocyte count in patients with septic arthritis is usually higher than 50,000/µL. In adults presenting with acute mono- or oligoarthritis, the probability of septic arthritis increases directly with the synovial fluid leukocyte count. For example, the positive likelihood ratio for patients with a synovial fluid leukocyte count between 25,000/µL and 50,000/µL is 2.9, whereas this value for patients with a synovial fluid leukocyte count between 50,000/µL and 100,000/µL is 7.7. In patients with a synovial fluid leukocyte count higher than 100,000/µL, the positive likelihood ratio is 28. A negative Gram stain does not exclude a diagnosis of septic arthritis. In addition, patients who are immunocompromised due to use of agents such as infliximab also have an increased risk for infection, including septic arthritis. Because of this patient's recent flare of bowel disease, infection with gram-negative bacteria must particularly be considered. In this patient, treatment of this type of infection should include the immediate initiation of empiric antibiotics (for example, vancomycin and ceftriaxone), repeated joint fluid drainage via arthrocentesis or surgical joint débridement, and discontinuation of infliximab. Avascular necrosis may be associated with the use of corticosteroids but usually manifests as a subacute or chronic condition. Synovial fluid in patients with this condition is usually noninflammatory. This patient has Crohn disease associated with musculoskeletal involvement. Crohn disease arthropathy may manifest as spondylitis accompanied by sacroiliitis. This condition also may be associated with two types of peripheral arthritis. In type I arthropathy, the peripheral arthritis is acute, affects six or fewer joints, and is associated with joint inflammation that tends to correlate with the activity of the bowel disease. Type I arthropathy also occurs early in the course of the bowel disease and may precede the onset of bowel disease. Type II arthropathy is characterized by polyarticular or migratory arthritis that is chronic and recurrent and does not parallel the activity of the bowel disease. Onset of type II arthropathy rarely precedes the diagnosis of inflammatory bowel disease. However, neither type of Crohn disease arthropathy manifests as acute monoarthritis, and the synovial fluid in patients with Crohn disease arthropathy is rarely markedly inflammatory. This patient's musculoskeletal symptoms and elevated synovial fluid leukocyte count are consistent with crystal-induced arthritis, but this condition is unlikely in the absence of crystals in polarized light microscopy of the synovial fluid.

Rheuma 33 A 64-year-old man is evaluated during a 3-month follow-up visit for rheumatoid arthritis. He was diagnosed with this condition 15 years ago. He has approximately 20 minutes of morning stiffness daily. He has chronic pain in the right knee; however, this pain has not worsened since his last visit. His last disease flare was 1 year ago, and he does not have fatigue, difficulty sleeping, or fever. His weight and functional level have been stable. Medications are methotrexate, hydroxychloroquine, naproxen, and a folic acid supplement. On physical examination, vital signs are normal. BMI is 33. There is bony enlargement of the second and third metacarpophalangeal joints bilaterally but no tenderness. There are swan-neck deformities of the third and fourth digits on the right hand. Range of motion of the knees elicits crepitus bilaterally, and extension of the right knee is restricted by 5 degrees. There is subluxation of the right second and left third metatarsophalangeal joints. Laboratory studies reveal a hemoglobin level of 12.5 g/dL (125 g/L) and an erythrocyte sedimentation rate of 44 mm/h. Radiographs of the hands and feet show marginal joint erosions that are unchanged from those obtained 2 years ago. Radiograph of the right knee reveals significant uniform joint-space narrowing and prominent osteophytes. Which of the following is the most appropriate medical treatment for this patient? A Etanercept B Prednisone C Sulfasalazine D No additional therapy

D Active inflammatory disease in patients with rheumatoid arthritis manifests as morning stiffness lasting longer than 1 hour, fatigue, anorexia, weight loss, loss of function, tenderness and swelling of numerous joints, elevated acute phase reactant levels, and progressive joint damage visible on radiography. This patient has long-standing seropositive erosive rheumatoid arthritis associated with swan-neck deformities, subluxations of the metatarsophalangeal joints, and post-inflammatory osteoarthritis in the right knee that manifests as uniform joint-space narrowing and prominent osteophyte formation. However, he does not have evidence of active inflammatory disease, which usually manifests as morning stiffness that lasts longer than 1 hour, fatigue, progressive loss of function, weight loss, or tenderness and swelling of numerous joints. Furthermore, his symptoms have not worsened since his last visit 3 months ago, and radiography reveals no further signs of erosive joint damage. In addition, although this patient's erythrocyte sedimentation rate is elevated, treatment decisions in patients with rheumatoid arthritis should be guided by signs and symptoms of their condition, and this patient has no evidence of active inflammatory disease. Therefore, a change in this patient's medication regimen is not warranted. Evidence shows that rapidly escalating therapy with a multidrug regimen is the most effective means of controlling rheumatoid arthritis. The addition of prednisone or sulfasalazine would be warranted in a patient whose disease is not controlled. Similarly, etanercept would be helpful in the setting of an inadequate response to therapy with an oral disease-modifying antirheumatic drug. However, the addition of any of these agents would not be appropriate for this patient, whose disease is currently well controlled.

Rheuma 1 A 56-year-old woman is evaluated for a 6-week history of arthralgia, prolonged morning stiffness involving the hands and feet, and severe fatigue. She has a history of hypothyroidism well controlled with levothyroxine. She takes ibuprofen, which has not helped to relieve her joint pain. Her mother has osteoarthritis of the knees. On physical examination, temperature is 36.8 °C (98.2 °F), blood pressure is 135/78 mm Hg, pulse rate is 90/min, and respiration rate is 16/min. BMI is 32. Cardiopulmonary examination is normal. There is no rash. Musculoskeletal examination reveals tenderness and swelling of the second and third metacarpophalangeal joints bilaterally. The elbows are stiff but have a full range of motion and are without synovitis. There is squeeze tenderness of the metatarsophalangeal joints bilaterally. Laboratory studies: Complete blood count Normal Rheumatoid factor Negative Thyroid-stimulating hormone 1.8 µU/mL (1.8 mU/L) Anti-cyclic citrullinated peptide antibodies Positive IgG antibodies against parvovirus B19 Positive IgM antibodies against parvovirus B19 Negative Which of the following is the most likely diagnosis? A Hypothyroidism B Parvovirus B19 infection C Polymyalgia rheumatica D Rheumatoid arthritis E Systemic lupus erythematosus

D Anti-cyclic citrullinated peptide antibodies are a more specific marker for rheumatoid arthritis than rheumatoid factor and may be present in patients with early disease who do not have rheumatoid factor. This patient most likely has early rheumatoid arthritis. This condition has a peak age of onset in the mid 50s. Rheumatoid arthritis most often manifests as symmetric polyarthritis involving the small joints of the hands and feet, and patients with early disease may have fever and fatigue. Rheumatoid factor assays detect the presence of IgM reactive against IgG, and rheumatoid factor is present in approximately 50% of patients with early rheumatoid arthritis. However, rheumatoid factor is not specific for rheumatoid arthritis and is present in patients with systemic lupus erythematosus, Sjögren syndrome, sarcoidosis, and other autoimmune diseases, as well as chronic infections. Anti-cyclic citrullinated peptide (CCP) antibodies are a more specific marker for rheumatoid arthritis and may be present in patients with early disease who do not have rheumatoid factor. Anti-CCP antibody assays also help to predict more severe disease, radiologic progression, and poorer functional outcomes. Hypothyroidism may cause fatigue and is occasionally associated with polyarthralgia. However, this patient's normal thyroid-stimulating hormone level excludes a diagnosis of hypothyroidism. Parvovirus B19 infection may manifest as fatigue, fever, rheumatoid factor positivity, and joint manifestations that resemble rheumatoid arthritis. This condition is usually self limited and resolves after several months. Affected adults commonly have nonspecific rashes but only rarely develop the characteristic rash of erythema infectiosum that defines infection in children, which is known as the "slapped cheek" rash. The presence of anti-CCP antibodies and the absence of IgM antibodies against parvovirus B19 in this patient argue against active parvovirus B19 infection. This patient's IgG antibodies against parvovirus B19 may be related to a previous infection with this virus that has since resolved. Polymyalgia rheumatica is characterized by fatigue, aching, and morning stiffness that involve the shoulder and hip girdles and is sometimes associated with giant cell arteritis. Fever, frank joint swelling, and symmetric involvement of the small joints of the hands and feet occur in a minority of patients with polymyalgia rheumatica. Patients with this condition usually have a markedly elevated erythrocyte sedimentation rate and anemia. This patient's lack of shoulder and hip girdle symptoms argues against this diagnosis. Systemic lupus erythematosus develops more frequently in women of reproductive age compared with the general population. This condition may be associated with polyarthritis and constitutional symptoms but is unlikely in the absence of systemic manifestations, such as rash, pleuritis, and cytopenias.

Rheuma 44 A 43-year-old man comes for evaluation of joint pain. For the past 8 months, he has had morning stiffness that lasts approximately 30 minutes daily and polyarthralgia involving the wrists and fingers. His symptoms have not interfered with his daily activities. He has hepatitis C virus infection for which he is not currently receiving therapy. Ibuprofen has been ineffective in relieving his joint pain. His mother and younger sister have severe rheumatoid arthritis. On physical examination, temperature is 37.0 °C (98.6 °F), blood pressure is 128/62 mm Hg, pulse rate is 68/min, and respiration rate is 12/min. Musculoskeletal examination reveals tenderness of the metacarpophalangeal and proximal interphalangeal joints bilaterally without swelling. Range of motion of all joints is full. Laboratory studies: Alanine aminotransferase 55 U/L Aspartate aminotransferase 46 U/L Alkaline phosphatase 100 U/L Rheumatoid factor Positive Hepatitis C virus antibodies Positive Radiographs of the hands show periarticular osteopenia but no erosions. Which of the following is the most appropriate management for this patient? A Begin methotrexate B Begin prednisone C Obtain radiographs of the feet D Perform anti-cyclic citrullinated peptide antibody assay

D Anti-cyclic citrullinated peptide antibodies are a more specific marker for rheumatoid arthritis than rheumatoid factor. Despite the modest duration of his morning stiffness, this patient has chronic symmetric polyarthralgia, tenderness around the joints, positive rheumatoid factor, and the presence of periarticular osteopenia on radiography. This clinical presentation is compatible with a diagnosis of rheumatoid arthritis. However, patients with hepatitis C virus (HCV) infection may have arthritis that mimics rheumatoid arthritis. Furthermore, even in the absence of arthritis, patients with HCV infection frequently have positive titers of rheumatoid factor because of the presence of circulating cryoglobulins. An anti-cyclic citrullinated peptide (CCP) antibody assay is therefore warranted to determine whether this patient's symptoms are caused by rheumatoid arthritis; these antibodies have a 95% specificity for rheumatoid arthritis and are not associated with HCV infection. Concomitant rheumatoid factor and anti-CCP antibody positivity are highly specific for rheumatoid arthritis. Differentiating between HCV infection-associated arthritis and rheumatoid arthritis is essential, because treatment of these conditions differs greatly. Initiation of effective antiviral treatment is indicated for HCV infection-associated arthritis, whereas careful selection and early initiation of disease-modifying antirheumatic drug therapy is indicated for rheumatoid arthritis. Therapy with prednisone would most likely help to control this patient's joint symptoms but would not help to establish a diagnosis. More importantly, patients with HCV infection treated with corticosteroids typically experience an increase in HCV viral load. Short courses of corticosteroids probably do not greatly impact the natural history of HCV infection, but the effect of long-term treatment is unknown. Rheumatoid arthritis often affects the feet, and radiographs of this patient's feet may show evidence of joint inflammation even in early disease. Radiographic changes typical of rheumatoid arthritis include erosions or unequivocal bony decalcification adjacent to the involved joints. Although the presence of these radiographic findings may support the diagnosis of rheumatoid arthritis, these findings are not diagnostic nor does their absence exclude this diagnosis. In addition, these findings are not likely to be present in a patient without symptomatic involvement of the feet. Therefore, radiographs of the feet would be unlikely to support a diagnosis of rheumatoid arthritis. Because methotrexate can cause hepatotoxicity, fibrosis, and cirrhosis, usually after prolonged use, use of this agent in a patient with pre-existing liver disease is contraindicated.

Rheuma 56 A 75-year-old man is evaluated for a 2-year history of pain in the left groin that radiates down the medial thigh. His pain worsens with walking and is relieved with rest. Acetaminophen provides only minimal relief for his pain. He was diagnosed with hypertension 10 years ago and also has type 2 diabetes mellitus and chronic kidney disease due to hypertensive nephropathy. Medications are glyburide, pioglitazone, and lisinopril. On physical examination, his blood pressure is 130/80 mm Hg. The FABERE (Flexion, ABduction, External Rotation, Extension) maneuver performed on the left hip reproduces pain on external rotation. The femoral pulses are intact, and there is no femoral lymphadenopathy. Laboratory studies: Serum creatinine 1.9 mg/dL (145.0 µmol/L) Blood urea nitrogen 24 mg/dL (8.6 mmol/L) Urinalysis Normal Radiograph of the left hip shows superior joint-space narrowing, subchondral sclerosis, and osteophytes. Which of the following is the most appropriate treatment of this patient's groin pain? A Celecoxib B Ibuprofen C Sulindac D Tramadol

D Both selective and nonselective NSAIDs have been shown to cause acute kidney injury in patients with chronic kidney disease and should not be used in this setting. This patient's clinical presentation is consistent with osteoarthritis of the hip, and the most appropriate treatment for his groin pain is tramadol. Pain associated with osteoarthritis of the hip is often referred to the groin and medial thigh and, less frequently, the knee and/or buttocks. Patients with this condition also often experience pain on passive range of motion of the hip joint at the extremes of flexion, extension, or internal and external rotation. NSAIDs such as celecoxib, ibuprofen, or sulindac are indicated for patients with osteoarthritis who have moderate-to-severe disease or who have achieved inadequate pain relief with acetaminophen. However, this patient has chronic kidney disease. In patients with compromised renal function, prostaglandin inhibition associated with ibuprofen can lead to frank renal failure, particularly in patients already taking an angiotensin-converting enzyme inhibitor. The most appropriate therapy for this patient's pain, therefore, is tramadol, which would not be expected to have any effect on renal function. The American College of Rheumatology guidelines recommend tramadol as a second-line agent in the treatment of osteoarthritis. This medication is considered particularly useful in patients with osteoarthritis of the hip or knee who are intolerant of or who do not achieve adequate pain control using acetaminophen and/or NSAIDs. Tramadol is a centrally acting analgesic, and the most common adverse effects associated with this agent are nausea and dizziness.

Rheuma 27 A 31-year-old man with systemic lupus erythematosus is evaluated for new-onset hypertension, peripheral edema, hematuria, and proteinuria. He also has hypocomplementemia, elevated anti-double-stranded DNA antibodies, and a serum creatinine level of 1.8 mg/dL (137.3 µmol/L). He was diagnosed with lupus 1 year ago, and his disease had been well controlled with prednisone and hydroxychloroquine. Additional medications are calcium and vitamin D supplements. Renal biopsy reveals World Health Organization class IV proliferative glomerulonephritis. His prednisone dosage is increased from 5 mg/d to 60 mg/d, and he begins intravenous cyclophosphamide, 1 g monthly, and lisinopril. Hydroxychloroquine therapy is continued. At a follow-up evaluation 6 months later, he is asymptomatic. His prednisone has been tapered to 5 mg/d, and his most recent dose of cyclophosphamide was administered 3 weeks ago. On physical examination, temperature is 36.4 °C (97.6 °F), blood pressure is 130/80 mm Hg, pulse rate is 64/min, and respiration rate is 16/min. Physical examination is unremarkable. Laboratory studies: Hemoglobin 12.3 g/dL (123 g/L) Leukocyte count 4200/µL (4.2 × 109/L) Platelet count 155,000/µL (155 × 109/L) Serum creatinine 1.2 mg/dL (91.6 µmol/L) Serum complement (C3 and C4) Normal Anti-double-stranded DNA antibodies Positive (low titers) Urinalysis 1+ protein; 3-5 leukocytes, 3-5 erythrocytes/hpf Urine protein-creatinine ratio 0.5 mg/mg Which of the following is the most appropriate treatment for this patient? A Continue monthly cyclophosphamide B Discontinue cyclophosphamide C Discontinue cyclophosphamide; start methotrexate D Discontinue cyclophosphamide; start mycophenolate mofetil

D Once remission is achieved with cyclophosphamide therapy, patients with lupus nephritis can safely be switched to mycophenolate mofetil or azathioprine to avoid cyclophosphamide toxicity. This patient has proliferative lupus nephritis; treatment of this condition involves two phases, remission induction and remission maintenance. The most appropriate treatment for this patient is to discontinue cyclophosphamide and start mycophenolate mofetil. This patient has been treated with corticosteroids and cyclophosphamide for the past 6 months. He is currently asymptomatic and has normal complement levels and only low titers of anti-double-stranded DNA antibodies. He has proteinuria, but this finding alone is not indicative of active lupus nephritis; mild proteinuria commonly persists even after effective resolution of lupus nephritis and is most likely caused by damage to the basement membrane. However, immunosuppression should not be stopped at this early stage. Nearly 50% of patients with proliferative lupus nephritis relapse after reduction or cessation of cyclophosphamide or other immunosuppressant therapy. Therefore, once remission is achieved, immunosuppression should be continued to maintain remission and prevent end-stage renal disease. Because of the risk for toxicity, prolonged monthly cyclophosphamide use is not recommended to maintain remission in patients with lupus nephritis. Once remission has been induced with 3 to 6 months of cyclophosphamide therapy, patients with this condition can safely be switched to mycophenolate mofetil or azathioprine. Methotrexate has not been studied for this indication. Alternatively, after six monthly cyclophosphamide infusions have been administered, the frequency of cyclophosphamide infusions can be reduced to every 3 months for an additional 18 months.

Rheuma 9 A 48-year-old woman is evaluated for a rash on her legs, arthralgia and myalgia, low-grade fever, and fatigue of 4 weeks' duration. Over the past week, she has had numbness and burning pain in her left foot; she also feels that this foot is slightly "dragging," and she has tripped and fallen several times. Fifteen years ago, she used illicit injection drugs for a 3-year period. On physical examination, temperature is 37.7 °C (99.8 °F), blood pressure is 150/100 mm Hg, pulse rate is 88/min, and respiration rate is 18/min. Cardiopulmonary examination is normal. Skin examination findings are shown . Abdominal examination reveals hepatomegaly. She has a left foot drop, and there is no synovitis. Laboratory studies: Hemoglobin 9.9 g/dL (99 g/L) Leukocyte count 10,400/µL (10.4 × 109/L) Platelet count 323,000/µL (323 × 109/L) Total protein 7.9 g/dL (79 g/L) Albumin 2.9 g/dL (29 g/L) Serum creatinine 1.6 mg/dL (122.1 µmol/L) Total bilirubin 0.9 mg/dL (15.4 µmol/L) Alkaline phosphatase 84 U/L Alanine aminotransferase 95 U/L Aspartate aminotransferase 108 U/L Rheumatoid factor 232 U/mL (232 kU/L) C3 32 mg/dL (320 mg/L) C4 8 mg/dL (80 mg/L) (normal range, 13-38 mg/dL [130-380 mg/L]) Antinuclear antibodies Negative Urinalysis 3+ protein; 2+ blood; 15-20 erythrocytes, 5-10 leukocytes/hpf A chest radiograph is normal. Which of the following diagnostic studies should be performed next? A ANCA B Anti-cyclic citrullinated peptide antibodies C Anti-double-stranded DNA antibodies D Serum cryoglobulin

D Cryoglobulinemic vasculitis secondary to hepatitis C virus infection may be associated with hepatomegaly, abnormal findings on liver chemistry studies, hypergammaglobulinemia, hypocomplementemia, and the presence of rheumatoid factor. This patient has constitutional symptoms, palpable purpura, and a foot drop consistent with mononeuritis multiplex. In addition, her elevated blood pressure, elevated serum creatinine level, proteinuria, and hematuria are suggestive of glomerulonephritis. This clinical presentation is compatible with many forms of vasculitis, but cryoglobulinemic vasculitis is the most likely diagnosis in a patient with hepatomegaly, abnormal liver chemistry studies, hypergammaglobulinemia (suggested by the elevated total protein and low albumin level), profound hypocomplementemia, and the presence of rheumatoid factor. A serum cryoglobulin assay is the most appropriate next diagnostic step in this patient. Cryoglobulins are immunoglobulins that precipitate in the cold and have rheumatoid factor activity (that is, they bind to the Fc portion of immunoglobulins). Type II cryoglobulinemia is most commonly associated with vasculitis, and cryoglobulins in this setting are monoclonal and are bound to polyclonal IgG. Approximately 85% to 95% of cases of type II cryoglobulinemia are associated with hepatitis C virus infection, which this patient most likely has based on her constitutional symptoms, history of injection drug use, and liver chemistry study abnormalities. The ANCA-associated vasculitides (Wegener granulomatosis, microscopic polyangiitis, and Churg-Strauss syndrome) may be associated with purpura, mononeuritis multiplex, and glomerulonephritis. However, none of these conditions would explain this patient's hypocomplementemia, hypergammaglobulinemia, or high titers of rheumatoid factor. An anti-cyclic citrullinated peptide antibody assay would be warranted in a patient in whom rheumatoid arthritis is suspected. This patient is rheumatoid factor positive, but her clinical presentation is more consistent with vasculitis than rheumatoid arthritis. Palpable purpura is rarely associated with rheumatoid arthritis and usually develops only in patients with long-standing erosive rheumatoid arthritis, which is not compatible with this patient's history. Rheumatoid arthritis occasionally is associated with the presence of cryoglobulins, but cryoglobulins in patients with rheumatoid arthritis are typically polyclonal (type III) and are not associated with vasculitis. Anti-double-stranded DNA antibodies are highly specific for systemic lupus erythematosus (SLE). SLE may manifest as mononeuritis multiplex, purpura, glomerulonephritis, and hypocomplementemia but is not usually associated with abnormal results on liver chemistry studies. In addition, more than 99% of patients with SLE have antinuclear antibodies, which are absent in this patient. Because patients with hepatitis C virus infection commonly have coinfection with HIV and share the same risk factors for hepatitis B, screening for infection with hepatitis B and C virus as well as HIV also would be indicated in this patient.

Rheuma 29 A 45-year-old man is evaluated for an 8-week history of bilateral knee and ankle pain. There is no history of trauma. He also has daily morning stiffness that lasts up to 2 hours and fatigue that has caused him to limit his activities. He consumes approximately 12 beers on weekends and is unwilling to limit his alcohol consumption. He has been taking ibuprofen, but this has not relieved his joint symptoms. On physical examination, temperature is 37.6 °C (99.7 °F), blood pressure is 140/80 mm Hg, and pulse rate is 88/min. BMI is 30. There is no rash. The wrists and knees are swollen bilaterally. He is unable to fully extend the right knee. Laboratory studies: Hemoglobin 14 g/dL (140 g/L) Leukocyte count 10,500/µL (10.5 × 109/L) Erythrocyte sedimentation rate 28 mm/h Rheumatoid factor Positive Alanine aminotransferase 35 U/L Aspartate aminotransferase 35 U/L Anti-cyclic citrullinated peptide antibodies Negative Arthrocentesis of the right knee is performed. Synovial fluid analysis reveals a leukocyte count of 13,500/µL. Polarized light microscopy of the fluid is negative for crystals, and synovial fluid culture is negative. Radiographs of the hands are normal. Radiographs of the knees show moderate joint effusions. Which of the following is the most appropriate treatment for this patient? A Leflunomide B Methotrexate C Prednisone D Sulfasalazine

D Disease-modifying antirheumatic drug therapy should be instituted within 3 months of the diagnosis of rheumatoid arthritis. This patient has early nonerosive rheumatoid arthritis, which usually manifests as fatigue, low-grade fever, and symmetric arthritis. The most appropriate treatment for this patient is sulfasalazine. Rheumatoid arthritis most often involves the small joints of the hands and feet and usually has a symmetric pattern of involvement; the larger joints, such as the knees, also may be affected. Additional manifestations of rheumatoid arthritis may include morning stiffness that lasts for more than 1 hour, synovitis, joint effusions, and loss of normal range of motion. Rheumatoid factor is present in more than 75% of patients with established rheumatoid arthritis, but the prevalence of rheumatoid factor in early disease is lower. An antibody against filaggrin, anti-cyclic citrullinated peptide, is less sensitive than rheumatoid factor but highly specific (96%) for rheumatoid arthritis. Plain radiographs in patients with early rheumatoid arthritis are often normal but may reveal soft-tissue swelling, uniform joint-space narrowing, and juxta-articular demineralization. This patient's synovial fluid analysis also is compatible with an inflammatory process, which further supports the diagnosis of rheumatoid arthritis. In most patients, rheumatoid arthritis is associated with progressive, irreversible joint destruction that causes increasing functional limitation. Disease-modifying antirheumatic drug (DMARD) therapy should be instituted within 3 months of the diagnosis of rheumatoid arthritis to prevent irreversible joint damage. Hydroxychloroquine and sulfasalazine are DMARDs that may be useful in patients with mild disease, which is often defined as a presentation of 5 to 10 inflamed joints with mild functional limitation, a minimally elevated erythrocyte sedimentation rate and C-reactive protein levels, and no erosions or anemia. Either sulfasalazine or hydroxychloroquine would be appropriate as initial therapy to treat this patient who has early, mild, nonerosive disease. Methotrexate is a DMARD that is often used in patients with active rheumatoid arthritis but would be contraindicated in a patient with abnormal liver chemistry study results or underlying liver disease, or in those who regularly consume alcoholic beverages. Leflunomide is similarly associated with hepatotoxicity and should be avoided in patients who consume significant amounts of alcohol. Prednisone would most likely help relieve this patient's joint symptoms and is often used as part of combination therapy in patients with rheumatoid arthritis. However, this agent has no disease-modifying properties and as a single agent would not help to prevent disease progression.

Rheuma 66 A 29-year-old woman with systemic lupus erythematosus (SLE) is evaluated in the office after obtaining positive results on a home pregnancy test. She has a 1-month history of nausea but is otherwise asymptomatic. Her last menstrual period was 2 months ago. This is her first pregnancy. Her SLE is well controlled with hydroxychloroquine, and her last flare was 10 months ago. On physical examination, temperature is 36.2 °C (97.2 °F), blood pressure is 110/72 mm Hg, pulse rate is 76/min, and respiration rate is 16/min. Physical examination is normal. A repeat pregnancy test is positive. Laboratory studies: Hemoglobin 12.1 g/dL (121 g/L) Leukocyte count 5400/µL (5.4 × 109/L) Platelet count 342,000/µL (342 × 109/L) Serum creatinine 0.7 mg/dL (53.4 µmol/L) Serum complement (C3 and C4) Normal Antinuclear antibodies Titer of 1:2560 Anti-Ro/SSA antibodies Positive Anti-double-stranded DNA antibodies Negative Anti-Smith antibodies Positive Urinalysis Normal She seeks advice on how to manage her SLE during her pregnancy. Which of the following is the most appropriate management of this patient? A Discontinue hydroxychloroquine B Recommend termination of pregnancy C Start prednisone D No change in management

D Hydroxychloroquine is safe to use in pregnancy and is useful for preventing systemic lupus erythematosus flares. This patient has systemic lupus erythematosus (SLE) and is in her first trimester of pregnancy, but her disease is well controlled with hydroxychloroquine. No change in treatment is warranted. She has had no symptoms of SLE for 10 months and currently has no signs of active disease, such as anemia, leukopenia, thrombocytopenia, hypocomplementemia, or anti-doubled-stranded DNA antibodies. She does have antinuclear, anti-Smith, and anti-Ro/SSA antibodies, but the presence of these autoantibodies does not vary with disease activity. Anti-Ro/SSA antibodies may be associated with congenital heart block in the fetus, and pregnant patients with these antibodies should undergo fetal echocardiography starting at 16 weeks of pregnancy. Hydroxychloroquine is a U.S. Food and Drug Administration category C agent in pregnancy. However, this agent is useful for preventing SLE flares, and expert opinion considers use of this agent to be appropriate during pregnancy because the benefits outweigh the risks. Discontinuation of this agent is therefore not needed in this patient. Patients with SLE whose disease has been quiescent for at least 6 months, during which time they either did not use medications for SLE or used medications that can safely be continued during pregnancy, generally have positive pregnancy outcomes. Therefore, there is no need to recommend termination of this patient's pregnancy. Pregnancy may trigger SLE flares, and, if needed, prednisone can be used during pregnancy. However, the addition of prednisone would not be warranted in a patient with no signs of active SLE, and corticosteroids generally are not used prophylactically.

Rheuma 70 A 67-year-old man is evaluated in the emergency department for a 2-week history of pain involving the left hip. He has had no fever. Four years ago, he underwent total arthroplasty of the left hip joint to treat osteoarthritis. One month ago, he underwent tooth extraction for an abscessed tooth. On physical examination, temperature is 36.6 °C (98.0 °F), blood pressure is normal, and pulse rate is 90/min. Cardiopulmonary examination is normal. A well-healed surgical scar is present over the left hip, and there is no warmth or tenderness. External rotation of the left hip joint is markedly painful. Laboratory studies reveal an erythrocyte sedimentation rate of 88 mm/h. Radiograph of the left hip shows a normally seated left hip prosthesis. Fluoroscopic-guided arthrocentesis is performed. The synovial fluid leukocyte count is 38,000/µL (90% neutrophils). Polarized light microscopy of the fluid shows no crystals, and Gram stain is negative. Culture results are pending. Which of the following is the most likely diagnosis? A aseptic loosening B Gout C Pigmented villonodular synovitis D Prosthetic joint infection

D In patients with prosthetic joint infection, pain is the predominant or only symptom, and fever and leukocytosis are frequently absent. This patient most likely has prosthetic joint infection, which may occur at any time in the postoperative period. Prosthetic joint infections that occur after the first postoperative year are most frequently caused by hematogenous spread of organisms to the prosthetic joint. The source of infection in this setting is often obvious and includes skin or genitourinary tract infection or, as in this patient, an abscessed tooth. Pain is the predominant or only symptom in patients with prosthetic joint infection, and fever and leukocytosis are frequently absent. Patients with prosthetic joint infection usually have an elevated erythrocyte sedimentation rate. Radiography may reveal prosthetic loosening, but hardware loosening may occur in patients without infection, as well. The gold standard for diagnosing prosthetic joint infection is arthrocentesis or intraoperative tissue sampling with culture before antibiotic therapy is initiated. The synovial fluid leukocyte count in patients with prosthetic joint infection is usually lower compared with that in patients with other forms of septic arthritis. Aseptic loosening refers to loss of fixation of the arthroplasty components, which is a major long-term complication of hip arthroplasty. The most striking manifestation of this condition is pain in the proximal and medial aspect of the thigh that is worse with weight bearing. Osteolysis is typically seen on radiographs of affected patients, which this patient does not have. Aseptic loosening also would not explain this patient's inflammatory synovial fluid. This patient's elevated synovial fluid leukocyte count with a predominance of neutrophils is suggestive of gout, but this condition does not have a subacute onset and does not commonly affect the hips. An acute attack of gout also would be associated with crystals visible on polarized light microscopy of the synovial fluid. Pigmented villonodular synovitis is a rare proliferative synovitis that most commonly involves the hip or knee. Radiographs in patients with this condition may reveal bone erosions or may be normal. Pigmented villonodular synovitis typically develops in young patients and is not associated with prosthetic joint placement.

Rheuma 5 A 78-year-old man with a 15-year history of osteoarthritis is evaluated for severe pain and swelling of the left knee of 4 days' duration. He also has hypertension, type 2 diabetes mellitus, and chronic kidney disease. Medications are glyburide, lisinopril, and low-dose aspirin. On physical examination, vital signs are normal. He is unable to bear weight on the left leg because of pain. The left knee is swollen and warm, and range of motion of this joint is limited and elicits pain. There are no tophi. Laboratory studies: Leukocyte count 15,600/µL (15.6 × 109/L) (90% polymorphonuclear cells, 10% lymphocytes) Glucose (random) 210 mg/dL (11.7 mmol/L) Serum creatinine 2.2 mg/dL (167.9 µmol/L) Serum uric acid 10.7 mg/dL (0.63 mmol/L) Urinalysis Normal Arthrocentesis of the left knee is performed. Synovial fluid leukocyte count is 24,000/µL (90% polymorphonuclear cells, 10% lymphocytes). Polarized light microscopy reveals intra- and extracellular monosodium urate crystals. Gram stain is negative. Which of the following is the most appropriate treatment for this patient? A Allopurinol B Colchicine C Ibuprofen D Intra-articular methylprednisolone E Prednisone

D Intra-articular injections of corticosteroids have been shown to be effective in the treatment of acute gout attacks and are useful in patients who cannot take NSAIDs or oral and parenteral corticosteroids. This patient has gout, which manifests as acute, intermittent attacks of severe pain, redness, and swelling of a joint accompanied by intracellular urate crystals seen on polarized light microscopy of the synovial fluid. The most appropriate treatment for this patient is an intra-articular corticosteroid injection. Early attacks of gout are typically monoarticular and usually involve a joint in the lower extremities, particularly the first metatarsophalangeal joint. Affected joints may be exquisitely tender to the touch and have a painful range of motion. Gout also is associated with an inflammatory synovial fluid leukocyte count during an acute attack and an elevated serum uric acid level, although many patients with hyperuricemia do not develop gout. Intra-articular corticosteroid injection has been shown to rapidly treat acute gout. This therapy is particularly useful in patients in whom NSAIDs or oral or parenteral corticosteroids are contraindicated. Furthermore, local injection therapy with a corticosteroid only minimally affects glycemic control. However, infectious arthritis must be excluded before this treatment is administered. Allopurinol would help to decrease this patient's uric acid level. However, use of uric acid-lowering agents during an acute attack does not ameliorate the attack of gout and may prolong an attack or cause a more frequent attack rate. Furthermore, allopurinol may cause a rare but serious hypersensitivity syndrome in patients who have renal insufficiency. Therefore, this agent should be used with caution starting at a low dose in this population group. Effective treatment of acute attacks of gout involves high-dose therapy with NSAIDs, corticosteroids, or colchicine. Immediately initiating NSAID treatment at anti-inflammatory doses is most likely more important than the specific agent used. However, NSAIDs are contraindicated in this patient because of his chronic kidney disease. Colchicine is most effective for the treatment of acute gout when used within the first 24 hours of symptom onset but would be less efficacious in this patient, whose symptoms began 4 days ago. This agent is associated with potential bone marrow toxicity, particularly in patients with renal insufficiency. Corticosteroids may be given orally, intramuscularly, or by intra-articular injection. These agents have similar efficacy to NSAIDs, but a high-dose oral or parenteral corticosteroid may exacerbate this patient's diabetes mellitus and is therefore relatively contraindicated.

Rheuma 19 A 21-year-old man is evaluated for a 5-day history of pain and swelling in the right ankle. Seven days ago, he developed a nodule on the left leg. He has no history of trauma to either of these sites. He is otherwise asymptomatic. On physical examination, vital signs are normal. The right ankle is warm and swollen, and range of motion elicits pain. He also has a warm, firm 2-cm erythematous nodule over the anterior left lower extremity that is tender to palpation and has been present for 7 days. The remainder of the physical examination is normal. Chest radiograph is shown . A plain radiograph of the right ankle is normal. Arthrocentesis of the right ankle is performed, and synovial fluid analysis reveals a leukocyte count of 3000/µL (80% lymphocytes, 12% macrophages). There are no crystals. Which of the following is the most likely diagnosis? A Bacterial arthritis B Crystal-induced arthritis C Osteoarthritis D Sarcoidosis

D Löfgren syndrome is a variant of sarcoidosis that manifests as acute inflammatory arthritis and typically involves the ankles, erythema nodosum, and hilar lymphadenopathy. This patient most likely has Löfgren syndrome, a variant of sarcoidosis characterized by acute monoarticular arthritis typically involving the ankles, erythema nodosum, and hilar lymphadenopathy that usually has a good prognosis. This patient has acute monoarticular arthritis involving a lower extremity, and the presence of tenderness, warmth, and swelling of the involved joint suggests that his condition is inflammatory; a synovial fluid leukocyte count higher than 2000/µL confirms the inflammatory nature of his condition. In addition, the tender, erythematous, indurated subcutaneous lesion over the right lower extremity is consistent with erythema nodosum, and a chest radiograph reveals hilar lymphadenopathy. Bacterial and crystal-induced arthritis may manifest as acute monoarticular arthritis and are often accompanied by signs and symptoms of inflammation, including the presence of inflammatory synovial fluid. However, the synovial fluid leukocyte count in patients with bacterial arthritis would be significantly higher than 3000/µL and often exceeds 50,000/µL and would be associated with a predominance of neutrophils. In addition, crystal-induced arthritis is unlikely in the absence of crystals on polarized light microscopy of the synovial fluid. Osteoarthritis usually manifests as pain that occurs with activity and is relieved with rest. This condition also typically is chronic in nature at the time of presentation and would be unusual in a young patient in the absence of trauma. Furthermore, osteoarthritis is not commonly associated with signs and symptoms of inflammation and does not typically involve the ankle. Finally, bacterial or crystal-induced arthritis or osteoarthritis would not explain this patient's erythema nodosum or hilar lymphadenopathy.

Rheuma 43 A 25-year-old female preschool teacher is evaluated for a 2-week history of symmetric pain and stiffness in the small joints of the hands and feet. Three weeks ago, she developed a macular, nonpruritic, erythematous rash on the upper extremities that resolved within 3 days. At that time, she also developed low-grade fever, myalgia, and malaise that also resolved within a few days. She states that many children at her workplace have had fever and rash. She does not have a history of Raynaud phenomenon, photosensitivity, serositis, kidney disease, or anemia. On physical examination, vital signs are normal. Cardiopulmonary examination is unremarkable. Cutaneous examination reveals no rash or scarring, and there are no oral ulcerations. There is mild diffuse soft-tissue swelling of the hands and feet bilaterally. The metacarpophalangeal and metatarsophalangeal joints are tender to palpation. Laboratory studies: Complete blood count Normal Rheumatoid factor Negative Antinuclear antibodies Titer of 1:40 Urinalysis Normal Which of the following antibody assays will be most helpful in establishing this patient's diagnosis? A Anti-cyclic citrullinated peptide antibodies B Anti-double-stranded DNA antibodies C Anti-topoisomerase I (anti-Scl-70) antibodies D IgM antibodies against parvovirus B19

D Parvovirus B19 infection in adults typically initially manifests as constitutional symptoms and an erythematous rash followed by the onset of symmetric arthritis of the small joints that may mimic rheumatoid arthritis. This patient most likely has parvovirus B19 infection. This condition commonly develops in outbreaks in children that manifest as erythema infectiosum, and adults usually acquire this infection from infected children; parents of young children and childcare workers are therefore more likely to develop this condition than the general population. Parvovirus B19 infection in children characteristically manifests as fever and an erythematous malar rash known as the "slapped cheek" rash, whereas infected adults are more likely to develop joint involvement. In adults, parvovirus B19 infection typically initially manifests as constitutional symptoms such as fever, myalgia, fatigue, and nausea. Approximately 70% of patients also develop an erythematous rash that may involve the face, trunk, or extremities. These symptoms are followed by the onset of symmetric arthritis that usually involves the small joints of the wrists, hands, and feet as well as the knees that typically resolves within weeks to months. IgM antibodies against parvovirus B19 would confirm the presence of this condition; these antibodies would be detectable 7 to 10 days after exposure to this virus and can be measured for several months after this time. This patient's pattern of joint involvement is compatible with early rheumatoid arthritis and systemic lupus erythematosus (SLE). However, caution is warranted before diagnosing this patient with either of these conditions because of the short duration of her symptoms. Rheumatoid arthritis is associated with anti-cyclic citrullinated peptide antibodies and may be present in patients who do not have rheumatoid factor; conversely, some patients with parvovirus B19 infection have positive titers of rheumatoid factor and antinuclear antibodies. A diagnosis of rheumatoid arthritis would not explain this patient's rash. Anti-double-stranded DNA antibodies are an insensitive but highly specific marker for SLE, which usually affects women in their 20s to 30s and also may manifest as constitutional symptoms. However, this patient does not have additional signs or symptoms consistent with SLE, such as serositis, anemia, or kidney disease. In addition, SLE is associated with a photosensitive malar or discoid rash, which is not consistent with this patient's cutaneous manifestations. Antitopoisomerase I (anti-Scl-70) antibodies are a specific marker for diffuse cutaneous systemic sclerosis. This condition may manifest as acute polyarthritis in patients with early disease, but musculoskeletal involvement in patients with systemic sclerosis is almost always preceded by Raynaud phenomenon. Systemic sclerosis also would be associated with characteristic skin thickening and would not explain this patient's erythematous rash.

Rheuma 53 A 19-year-old woman is evaluated for a 6-month history of pain and swelling of the hands. Her hands develop blue and white color changes when exposed to cold temperatures, and she has small, painful, slowly healing lesions on the tips of her fingers. She also has progressive fatigue and weakness. She takes no medications. On physical examination, she rocks back and forth and reaches forward and uses her arms to push herself up when rising from her chair. She also has difficulty getting onto the examination table. Vital signs are normal. The appearance of a finger is shown . Proximal muscle strength is 3/5 in the upper and lower extremities. There are effusions of the metacarpophalangeal, wrist, and knee joints bilaterally. The remainder of the physical examination is normal. Laboratory studies: Hemoglobin 10.2 g/dL (102 g/L) Leukocyte count 1900/µL (1.9 × 109/L) (15% lymphocytes) Platelet count 85,000/µL (85 × 109/L) Erythrocyte sedimentation rate 85 mm/h Creatine kinase 5500 U/L Alanine aminotransferase 320 U/L Aspartate aminotransferase 270 U/L Total bilirubin 0.8 mg/dL (13.7 µmol/L) Antinuclear antibodies Titer of 1:2560 Antiribonucleoprotein antibodies Positive Which of the following is the most appropriate next step in this patient's management? A Bone marrow biopsy B CT of the chest, abdomen, and pelvis C Liver biopsy D Prednisone

D Patients with an inflammatory myopathy and features of systemic sclerosis, systemic lupus erythematosus, and rheumatoid arthritis accompanied by high titers of antiribonucleoprotein antibodies often have mixed connective tissue disease. This patient's elevated creatine kinase level and proximal muscle weakness are consistent with an inflammatory muscle disease. She also has arthritis, Raynaud phenomenon with digital pitting, pancytopenia, and high titers of antinuclear and antiribonucleoprotein antibodies. This clinical presentation is consistent with mixed connective tissue disease, and the most appropriate next step in this patient's management is initiation of prednisone therapy. Treatment in patients with mixed connective tissue disease involves managing the specific visceral manifestations. High-dose oral corticosteroid therapy is the standard first-line treatment for active inflammatory muscle disease and is warranted particularly in a patient with muscle weakness of this severity. This patient's pancytopenia is most likely associated with mixed connective tissue disease features consistent with systemic lupus erythematosus, and bone marrow biopsy is unlikely to provide additional information that would alter the diagnosis or treatment options. Although the complete blood count in this patient should be monitored closely, corticosteroid therapy initiated to treat the muscle disease is likely to also help to treat her hematologic abnormalities. The inflammatory myopathies are associated with malignancy, particularly in older patients. However, malignancy associated with a myopathy is rare in patients under 30 years of age and in those whose myositis is associated with a systemic connective tissue disease. Therefore, evaluation for malignancy, including screening CT of the chest, abdomen, and pelvis, is not indicated in this patient. Liver biopsy would be beneficial in a patient with elevated liver enzyme levels of unknown origin. However, active muscle disease is associated with an increase in aspartate and alanine aminotransferase levels, and this patient's liver enzyme study results are most likely related to her underlying muscle disease.

Rheuma 74 A 53-year-old man is evaluated in the emergency department for a 2-day history of acute swelling and pain in the right knee. He also has had fever up to 38.3 °C (101.0 °F). Three weeks ago, he was evaluated in the emergency department for cellulitis. Medical history is significant for chronic tophaceous gout and hypertension. Medications are allopurinol, atenolol, and enalapril. He has a monogamous sexual relationship with his wife of 30 years. On physical examination, temperature is 38.1 °C (100.5 °F), blood pressure is 124/50 mm Hg, and pulse rate is 88/min. Cardiopulmonary examination is normal. Tophi are present on both elbows. The right fourth proximal interphalangeal joint and left third metacarpophalangeal joint have soft-tissue swelling but no warmth or erythema. The right knee is markedly swollen and has overlying warmth and erythema. Palpation of this joint elicits pain. Laboratory studies reveal a leukocyte count of 15,000/µL (15 × 109/L). Arthrocentesis is performed. The synovial fluid leukocyte count is 110,000/µL (95% neutrophils). Polarized light microscopy of the fluid reveals negatively birefringent monosodium urate crystals. Gram stain of the aspirated fluid is negative. Culture results are pending. Which of the following is the most appropriate treatment for this patient? A Ciprofloxacin B Intra-articular corticosteroids C Prednisone D Vancomycin

D Pending culture results, empiric vancomycin is recommended for patients with suspected septic arthritis whose synovial fluid Gram stain reveals gram-positive organisms and for immunocompetent patients with a negative synovial fluid Gram stain. This patient may have septic arthritis and should begin treatment with intravenous vancomycin. He has a history of chronic tophaceous gout but also has a history of cellulitis and the new onset of acute monoarthritis of the knee, fever, peripheral leukocytosis, and a markedly elevated synovial fluid leukocyte count. This constellation of symptoms is consistent with an acute flare of gout but also raises strong suspicion for septic arthritis. A diagnosis of septic arthritis should be considered in any patient who presents with the sudden onset of monoarthritis or the acute worsening of chronic joint disease. In patients with gout or pseudogout, the presence of crystals on polarized microscopic analysis of the synovial fluid does not exclude a concomitant infection. Therefore, until culture results are available, empiric antibiotic treatment is indicated for this patient. Infection with gram-positive pathogens such as staphylococci or streptococci is the most common cause of septic arthritis in older patients. Therefore, vancomycin, which has broad gram-positive coverage, is the empiric therapy of choice in patients with suspected septic arthritis whose Gram stain reveals gram-positive organisms. Furthermore, many experts recommend empiric antibiotic therapy with vancomycin for immunocompetent patients with suspected septic arthritis and a negative Gram stain, whereas vancomycin and a third-generation cephalosporin such as ceftriaxone are recommended for patients who are immunocompromised or have trauma-associated infection. In addition, experts suggest careful joint management with repeated arthrocenteses or drainage until the inflammatory component of this condition is resolved. If disseminated gonococcal infection is included in the differential diagnosis, ceftriaxone may be added to vancomycin. Gram-negative organisms account for 9% to 20% of cases of septic arthritis, and certain pathogens occur more frequently in specific patient groups. For example, Salmonella infection has been reported in patients with systemic lupus erythematosus, AIDS, or sickle cell anemia, whereas patients who use illicit injection drugs often have infection with Pseudomonas aeruginosa. Comorbid medical conditions, a history of antibiotic use, and extra-articular infections (particularly urinary tract infections and decubitus ulcers) predispose patients to gram-negative septic arthritis. Neither prednisone nor intra-articular corticosteroids are effective for treating infections.

Rheuma 38 A 72-year-old woman is evaluated for a 1-year history of progressive pain in the right knee. The pain is most acute along the medial aspect of the joint, worsens with activity, and is relieved with rest. She has no stiffness in the morning and has had no swelling. She also has not experienced locking or giving away of this joint. On physical examination, vital signs are normal. There is bony enlargement of the proximal and distal interphalangeal joints. There is no evidence of a right knee effusion. Passive flexion and extension of the right knee are painful. Laboratory studies, including complete blood count, erythrocyte sedimentation rate, and C-reactive protein, are normal. Radiograph of the right knee also is normal. In addition to acetaminophen as needed, which of the following is the most appropriate next step in this patient's management? A Arthroscopy B Aspiration of the knee C MRI of the knee D Physical therapy

D Physical therapy is an appropriate first-line management option for patients with osteoarthritis of the knee, and quadriceps muscle training in particular has been shown to reduce pain in this setting. This patient has osteoarthritis of the knee. The most appropriate next step in her management is referral for physical therapy, which is an appropriate first-line management option for patients with this condition. Quadriceps muscle training in particular has been shown to reduce pain in this population group. Use of over-the-counter acetaminophen or an NSAID on an as-needed basis also may benefit this patient. Arthroscopy and MRI of the knee would most likely reveal abnormalities of the articular cartilage not visible on plain radiography but are not needed to establish the diagnosis of osteoarthritis. Similarly, aspiration of the knee joint would be warranted in patients with an effusion to obtain a synovial fluid leukocyte count but is not needed to establish a diagnosis; furthermore, this patient does not have an effusion.

Rheuma 45 A 60-year-old woman is evaluated for a 4-month history of progressive fatigue and dyspnea on exertion. She does not smoke cigarettes and denies chest pain, palpitations, dizziness, or syncope. She has a 12-year history of limited cutaneous systemic sclerosis. A screening cardiopulmonary evaluation 3 years ago was normal. She also has gastroesophageal reflux disease and Raynaud phenomenon and intermittently develops ulcers on the fingertips. Current medications are amlodipine, omeprazole, and nitroglycerin ointment. On physical examination, temperature is 37.0 °F (98.6 °F), blood pressure is 120/80 mm Hg, pulse rate is 84/min, and respiration rate is 16/min. Cardiac examination reveals a loud pulmonic component of S2 with fixed splitting and a 2/6 early systolic murmur at the lower left sternal border that increases with inspiration. The lungs are clear to auscultation. The abdominal examination is unremarkable. Sclerodactyly is present, and pitting scars are visible over several fingertips. There is no peripheral edema. Complete blood count and erythrocyte sedimentation rate are normal. Electrocardiogram shows evidence of right ventricular hypertrophy. Chest radiograph shows no infiltrates. Pulmonary function studies: FVC 84% of predicted FEV1/FVC 0.8 DLCO 44% of predicted Which of the following is the most likely diagnosis? A Atrial septal defect B Interstitial lung disease C Left ventricular failure D Pulmonary arterial hypertension

D Pulmonary arterial hypertension is among the most common manifestations of lung involvement in patients with systemic sclerosis. This patient most likely has pulmonary arterial hypertension (PAH) associated with collagen vascular disease related to systemic sclerosis. Pulmonary disease is the primary cause of morbidity in patients with systemic sclerosis; PAH is among the most common manifestations of lung involvement in these patients, particularly in those with limited cutaneous disease. This patient's worsening fatigue and dyspnea on exertion in the presence of clear lung fields are consistent with PAH. Physical signs of elevated pulmonary artery pressure include a loud P2, fixed split S2, pulmonic flow murmur, and tricuspid regurgitation. Chest radiographs are usually normal in early disease but may show enlarged pulmonary arteries, right atrium, and right ventricle. Electrocardiographs in these patients may show right ventricular strain or hypertrophy. Pulmonary function studies in patients with PAH usually reveal an isolated decreased DLCO in the setting of normal airflow and lung volumes (excluding restrictive lung disease). Echocardiography is an early diagnostic test for patients with signs and symptoms of PAH. This study is used to exclude congenital heart disease, such as atrial septal defect, and valvular heart disease, such as mitral stenosis, that may manifest as PAH. Echocardiography may be used to estimate peak right ventricular systolic pressure if tricuspid regurgitation is noted on examination. Atrial septal defects are the second most common form of congenital heart disease in adults. The characteristic physical examination finding in patients with an atrial septal defect is related to the development of PAH. This patient's limited systemic sclerosis and normal cardiopulmonary evaluation 3 years ago makes atrial septal defect a much less likely cause of her findings. Interstitial lung disease (ILD) is a common pulmonary manifestation in patients with systemic sclerosis who also have dyspnea and fatigue, and patients with ILD also usually have dry cough. The absence of late inspiratory crackles on pulmonary examination and the presence of normal lung volumes on pulmonary function testing further argue against this condition. In patients with ILD, the lung volumes are less than 80% of predicted. Left ventricular failure may manifest as dyspnea and fatigue and may be associated with cardiac murmurs. However, this condition is unlikely in the absence of additional abnormal cardiopulmonary examination findings, such as an S3 or S4 gallop and pulmonary crackles. In addition, chest radiographs in patients with left ventricular heart failure usually demonstrate pulmonary vascular congestion.

Rheuma 10 A 69-year-old woman is evaluated for a 5-day history of pain and swelling in the right knee that developed after a prolonged period of gardening in her backyard. Her pain worsens when she walks and is relieved with rest. She has no morning stiffness. On physical examination, temperature is normal, blood pressure is 128/72 mm Hg, pulse rate is 88/min, and respiration rate is 17/min. BMI is 32. The right knee has a moderate effusion but is not warm or erythematous. Range of motion of the right knee elicits pain. The remainder of the musculoskeletal examination is normal. Plain radiographs of the right knee show osteophytes and medial joint-space narrowing. Arthrocentesis is performed, and synovial fluid analysis reveals 250 leukocytes/µL (88% lymphocytes, 12% macrophages). Which of the following is the most likely diagnosis? A Bacterial arthritis B Crystal-induced arthritis C Fungal arthritis D Osteoarthritis

D The acute presentation of a noninflammatory monoarthritis involving a weight-bearing joint is suggestive of osteoarthritis. This patient most likely has osteoarthritis of the knee. Osteoarthritis may manifest as acute monoarticular arthritis and is usually noninflammatory. This condition is characterized by pain on activity that is relieved with rest and most commonly involves the proximal and distal interphalangeal and first carpometacarpal joints of the hands as well as the weight-bearing joints such as the knees and hips. Range of motion of the involved joint in patients with osteoarthritis is usually painful and may be limited. This patient's radiographic findings of joint-space narrowing and osteophytes are consistent with osteoarthritis. The synovial fluid leukocyte count in patients with osteoarthritis is typically between 200 and 2000/µL and may be associated with a predominance of lymphocytes. Bacterial, crystal-induced, and fungal arthritis also may manifest as monoarticular arthritis. However, these conditions typically are associated with inflammation and would typically cause morning stiffness, warmth, and erythema of the involved joint. Bacterial and fungal arthritis would most likely be associated with a synovial fluid leukocyte count of at least 10,000/µL with a predominance of neutrophils, and the synovial fluid leukocyte count in patients with bacterial arthritis often exceeds 100,000/µL. Finally, bacterial and crystal-induced arthritis often have an acute presentation, whereas fungal arthritis is usually chronic in nature at the time of presentation.

Rheuma 68 A 20-year-old woman is evaluated for a 5-month history of malaise, fatigue, myalgia, occasional headaches, and an unintentional 4.5-kg (10.0-lb) weight loss. Five weeks ago, she began to develop pain in her arms and legs when exercising at the gym; this pain resolves with rest. On physical examination, temperature is 37.3 °C (99.2 °F), blood pressure is 180/95 mm Hg in the right arm and 110/70 mm Hg in the left arm, pulse rate is 84/min, and respiration rate is 16/min. A bruit is heard over the left subclavian artery and left flank. The radial pulse is absent on the left side, and the dorsalis pedis pulses are absent bilaterally. Laboratory studies: Hemoglobin 9.2 g/dL (92 g/L) Leukocyte count 14,000/µL (14 × 109/L) Platelet count 575,000/µL (575 × 109/L) Erythrocyte sedimentation rate 125 mm/h Serum creatinine 1.1 mg/dL (83.9 µmol/L) Urinalysis Normal Chest radiograph is normal. Which of the following is the most likely cause of this patient's hypertension? A Glomerulonephritis B Pheochromocytoma C Polyarteritis nodosa D Renal artery stenosis

D The initial manifestations of Takayasu arteritis include fever, arthralgia, myalgia, fatigue, and weight loss and may precede the development of signs and symptoms of large-vessel narrowing by several months. This patient's hypertension is most likely caused by renal artery stenosis. Her clinical presentation is strongly suggestive of Takayasu arteritis, a disease of the large arteries that most commonly affects young women. Typical initial manifestations of Takayasu arteritis include fever, arthralgia, myalgia, fatigue, and weight loss. Affected patients also commonly have elevated acute phase reactants, such as the erythrocyte sedimentation rate, and a normocytic, normochromic anemia. After several weeks or months, patients with Takayasu arteritis develop symptoms of vascular insufficiency, such as arm and leg claudication, or hypertension that is usually caused by renal artery stenosis that is often bilateral. Affected patients also commonly have diminished, often asymmetric arterial pulses in the extremities. Takayasu arteritis commonly causes stenoses in the aorta and its branches. Significant stenosis of one of the subclavian arteries, which is suggested by this patient's left subclavian bruit, results in decreased blood pressure in the ipsilateral arm. In patients with a discrepancy in blood pressure between the arms, the higher measurement is considered the most accurate. Blood pressure also can be measured in the legs, but some patients may have artificially low blood pressure in all four limbs because of large-vessel stenoses. High-dose corticosteroids often help to resolve hypertension associated with Takayasu arteritis. However, renal artery bypass grafting or angioplasty may be needed if hypertension in patients treated with these agents persists. Glomerulonephritis is unlikely in the absence of significant hematuria, proteinuria, or erythrocyte casts on urinalysis. In addition, although Takayasu arteritis may cause inflammation of the renal arteries, this condition does not cause glomerulonephritis. Screening for pheochromocytoma is often performed in young patients with new-onset hypertension, but this patient's constitutional symptoms and elevated erythrocyte sedimentation rate are more consistent with an inflammatory process. Polyarteritis nodosa is a necrotizing vasculitis of the medium-sized arteries that commonly involves the intrarenal and renal arteries. Aneurysmal beading of these vessels can be seen on angiography, and hypertension is common in affected patients. Signs and symptoms of large-vessel stenosis, such as claudication, bruits, or absence of peripheral pulses, are not consistent with poly-arteritis nodosa.

Rheuma 25 A 34-year-old woman is evaluated for a 4-month history of heartburn and regurgitation that occurs several times daily. She has a 2-month history of swelling and aching in the hands. Six months ago, she also began to develop episodes of pain and cyanosis involving the fingers upon exposure to cold temperatures. She has no other medical problems and takes no medications. On physical examination, temperature is 37.0 °C (98.6 °F), blood pressure is 100/60 mm Hg, pulse rate is 78/min, and respiration rate is 18/min. There is skin thickening over the fingers. There is no evidence of joint swelling, synovitis, or tendinitis. The remainder of the physical examination is normal. Laboratory studies, including complete blood count, basic metabolic profile, and urinalysis, are normal except for an antinuclear antibody titer of 1:640 (speckled pattern). The patient received counseling about avoiding the cold and other environmental protection. Which of the following is the most appropriate additional treatment for this patient? A Cyclophosphamide B Enalapril C Methotrexate D Omeprazole E Prednisone

D Treatment of systemic sclerosis consists of symptomatic management of the disease manifestations. This patient's recent onset of skin thickening, Raynaud phenomenon, and symptoms consistent with gastroesophageal reflux disease is suggestive of systemic sclerosis. The most appropriate treatment for this patient is omeprazole. Systemic sclerosis most commonly affects women in their 30s and 40s and is strongly associated with the presence of antinuclear antibodies. This condition may involve the skin and visceral organs. Treatment of systemic sclerosis consists of symptomatic management of the disease manifestations. Therefore, treatment with a proton pump inhibitor such as omeprazole would be appropriate to treat this patient's gastroesophageal reflux disease. Avoidance of cold also would benefit her symptoms of Raynaud phenomenon; if this intervention is not effective, a dihydropyridine calcium channel blocker may be warranted. In patients with systemic sclerosis, cyclophosphamide has been shown to improve pulmonary symptoms and lung volumes and has been shown to modestly improve lung function, thickening of the skin, and quality of life. However, this agent has not been evaluated in randomized clinical trials for the treatment of skin disease in the absence of pulmonary involvement. Angiotensin-converting enzyme inhibitors such as enalapril are the treatment of choice for scleroderma renal crisis but have no role in prophylactic treatment of this condition. Randomized placebo-controlled clinical trials of methotrexate for the treatment of scleroderma skin disease have been promising, but the group differences were small and not statistically significant. These findings suggest that methotrexate is not effective in the treatment of early systemic sclerosis. Prednisone has not been shown to alter the disease course in early systemic sclerosis. Moreover, use of this agent is a risk factor for the development of scleroderma renal crisis in patients with systemic sclerosis

Rheuma 17 A 57-year-old man is evaluated in the emergency department for the acute onset of rapidly worsening dyspnea. For the past 10 weeks, he has had pain and swelling in the small joints of the hands and in the knees; he was diagnosed with seronegative symmetric inflammatory polyarthritis 2 weeks ago and was started on low-dose methotrexate, a folic acid supplement, low-dose prednisone, and naproxen at that time. He also has a history of refractory otitis media and underwent bilateral tympanostomy tube placement 6 months ago. He is in respiratory failure and is intubated, mechanically ventilated, and admitted to the hospital. Blood is noted when he is intubated. On physical examination on admission, temperature is 38.5 °C (101.3 °F), blood pressure is 135/95 mm Hg, pulse rate is 125/min, and respiration rate is 24/min. There is no bleeding from the gums. Pulmonary examination reveals diffuse crackles throughout all lung fields. The metacarpophalangeal and proximal interphalangeal joints are swollen, and both knees have medium-sized effusions. Palpable purpura is present on the calves. Laboratory studies: Hemoglobin 10 g/dL (100 g/L) Leukocyte count 12,500/µL (12.5 × 109/L) (80% neutrophils) Serum creatinine 2.6 mg/dL (198.4 µmol/L) Rheumatoid factor Negative Antinuclear antibodies Negative c-ANCA Positive Anti-cyclic citrullinated peptide antibodies Negative Antiproteinase-3 antibodies Positive Serologic test for HIV antibodies Negative Urinalysis 2+ protein; 1+ blood; 15 erythrocytes/hpf A chest radiograph shows normal heart size and diffuse alveolar infiltrates in both lung fields. Ceftriaxone, azithromycin, and hydrocortisone are started. His previous medications are discontinued. Which of the following is the most likely diagnosis? A Interstitial pneumonitis B Methotrexate-induced pneumonitis C Pneumocystis pneumonia D Wegener granulomatosis

D Wegener granulomatosis should be considered in patients with upper- and lower-airway manifestations, renal involvement, and inflammatory arthritis. This patient most likely has Wegener granulomatosis, a necrotizing vasculitis that typically affects the upper- and lower-respiratory tract and the kidneys. This patient's purpura is consistent with vasculitis. His diffuse pulmonary infiltrates (generally associated with alveolar hemorrhage), history of refractory otitis media, renal failure, and urinalysis findings that suggest glomerulonephritis particularly raise suspicion for Wegener granulomatosis. Wegener granulomatosis may be associated with inflammatory arthritis involving the small and large joints and joint effusions. The presence of c-ANCA and antiproteinase-3 antibodies is approximately 90% specific for this condition. The presentation of Wegener granulomatosis is highly nonspecific and evolves slowly over a period of months; therefore, diagnosis of this condition is often delayed by several months. Patients with severe, long-standing rheumatoid arthritis may develop interstitial pneumonitis, and this condition is particularly likely to develop in men. Radiographs of patients with this condition usually show bibasilar interstitial markings. Interstitial lung disease associated with rheumatoid arthritis most characteristically has an insidious onset and is associated with seropositive, erosive joint disease. In most patients, the lung disease appears 5 years or more after the diagnosis of rheumatoid arthritis. Methotrexate-induced pneumonitis can occur at any time in the course of therapy with this agent, regardless of the dosage or duration of treatment. However, this condition would not explain this patient's entire clinical picture, including c-ANCA positivity, vasculitis, renal failure, and his urinalysis findings. Pneumocystis pneumonia may manifest as fever, dyspnea, tachypnea, and crackles heard on pulmonary examination. However, dyspnea is typically progressive and not acute and would not result in rapid pulmonary failure. Chest radiography in patients with this condition may show diffuse infiltrates. Pneumocystis pneumonia also usually develops in patients who are significantly immunosuppressed, whereas this patient has received only a short course of low-dose methotrexate. Furthermore, pneumocystis pneumonia would not explain this patient's additional findings.

Rheuma 52 A 28-year-old woman is evaluated in the emergency department for a 1-day history of nausea, vomiting, and blood per rectum. For the past several months, she has had fatigue and malaise. Two weeks ago, she developed arthralgia involving the hands and feet, intermittent pleuritic chest pain, and abdominal pain. She also has a 1-week history of low-grade fever and worsening of her abdominal pain. On physical examination, she appears ill. Temperature is 38.3 °C (100.9 °F), blood pressure is 145/85 mm Hg, pulse rate is 112/min, and respiration rate is 16/min. There is an erythematous rash over the cheeks and forehead. Cardiopulmonary examination reveals a friction rub. Abdominal examination reveals mild distention, rare bowel sounds, and diffuse abdominal tenderness. The wrists are tender and mildly swollen. Bilateral 1+ peripheral edema is present. A stool specimen is positive for occult blood. Laboratory studies: Hemoglobin 8.9 g/dL (89 g/L) Leukocyte count 2800/µL (2.8 × 109/L) Platelet count 48,000/µL (48 × 109/L) Erythrocyte sedimentation rate 116 mm/h Reticulocyte count 2.6% of erythrocytes Haptoglobin 5 mg/dL (50 mg/L) Serum creatinine 1.8 mg/dL (137.3 µmol/L) Lactate dehydrogenase 580 U/L Serum complement (C3 and C4) Decreased ANCA Negative Antinuclear antibodies Titer of 1:1280 Anti-double-stranded DNA antibodies Positive Hepatitis B surface antigen (HBsAg) Negative Urinalysis 2+ protein; 2+ blood; 15 leukocytes, 15-20 erythrocytes/hpf; occasional erythrocyte casts Direct antiglobulin (Coombs) test Positive A peripheral blood smear is normal. Chest radiograph shows a small pleural effusion, and radiograph of the abdomen shows dilatation of the bowel loops without obstruction or free air. CT of the abdomen reveals symmetric thickening of the bowel wall, dilatation of bowel segments, and an increased number of vessels in a comb-like pattern consistent with bowel ischemia. Colonoscopy reveals scattered ulcerations suggestive of ischemia. Which of the following is the most likely diagnosis? A Crohn disease B Hemolytic uremic syndrome C Henoch-Schönlein purpura D Polyarteritis nodosa E Systemic lupus erythematosus

E Approximately one third of patients with active systemic lupus erythematosus who have abdominal pain have mesenteric vasculitis. This patient most likely has systemic lupus erythematosus (SLE). Her extraintestinal manifestations consist of butterfly rash, physician-confirmed arthritis, pancytopenia including Coombs-positive hemolytic anemia, serositis, pleuritis, renal insufficiency, and urine sediment findings suggestive of glomerulonephritis. She also has antinuclear antibodies, anti-double-stranded DNA antibodies, and hypocomplementemia. This constellation of features is consistent with SLE. This patient also has abdominal pain, nausea, vomiting, and blood per rectum. Her imaging studies are consistent with bowel ischemia. Mesenteric vasculitis is the most likely cause of this clinical presentation. Approximately one third of patients with active SLE who have abdominal pain have mesenteric vasculitis. This condition typically warrants immediate treatment with high-dose corticosteroids and intravenous cyclophosphamide; occasionally, surgical intervention is indicated. Patients with Crohn disease may present with fever, weight loss, abdominal pain, nausea, vomiting, or diarrhea. Extraintestinal manifestations of this condition include arthralgia and arthritis, pyoderma gangrenosum, or erythema nodosum. Inflammation in patients with Crohn disease involves the bowel mucosa; endoscopic examination may show aphthous ulcers or large ulcers that can coalesce and cause a "cobblestone" appearance. Rectal sparing is common, and areas of disease activity may be separated by areas of normal mucosa ("skip areas"). Bowel ischemia is rare in these patients. Anemia is a common manifestation of Crohn disease but usually is caused by iron deficiency or chronic disease, not hemolysis. Pancytopenia, glomerulonephritis, anti-double-stranded DNA antibodies, hypocomplementemia, and high titers of antinuclear antibodies also are not associated with this condition. Hemolytic uremic syndrome (HUS) is characterized by acute kidney injury accompanied by microangiopathic hemolytic anemia and thrombocytopenia. This condition also may manifest as painful bloody diarrhea. HUS would be unlikely in the absence of schistocytes on a peripheral blood smear and would not explain this patient's positive results on a direct antiglobulin (Coombs) test, antinuclear and anti-double-stranded DNA antibodies, or hypocomplementemia. Bowel ischemia and glomerulonephritis should raise strong suspicion for Henoch-Schönlein purpura (HSP). HSP is a systemic IgA-mediated vasculitis that may involve the skin, joints, gastrointestinal tract, and kidneys. This condition most commonly affects children and usually is associated with a purpuric rash. HSP also would not explain this patient's pancytopenia, hypocomplementemia, or autoantibodies. Polyarteritis nodosa is a medium-sized vessel vasculitis that may manifest as arthritis and, occasionally, as a vasculitic rash; affected patients also may develop involvement of the renal arteries and arterioles, which may cause hypertension. Suspicion for polyarteritis nodosa always should be raised in patients who present with mesenteric ischemia. However, polyarteritis nodosa would not cause glomerulonephritis or pancytopenia and would not be associated with autoantibodies.

Rheuma 63 A 26-year-old female electrical engineer is evaluated for a 2-year history of persistent pain and stiffness involving the low back. These symptoms are worse in the morning and are alleviated with exercise and hot showers. There are no radicular symptoms. Her only medication is ibuprofen, which has helped to relieve her symptoms. She is married and is sexually monogamous with her husband. She has no other medical problems and takes no additional medications. Family history is negative for arthritis. On physical examination, vital signs are normal. Cutaneous examination is normal. Palpation of the pelvis and low back elicits pain. There is loss of normal lumbar lordosis, and forward flexion of the lumbar spine is decreased. Reflexes and strength are intact. Radiographs of the lumbar spine and pelvis are normal. Which of the following studies is most likely to establish the diagnosis in this patient? A Anti-cyclic citrullinated peptide antibodies B Bone scan C Erythrocyte sedimentation rate D HLA-B27 E MRI of the sacroiliac joints

E MRI, especially with gadolinium enhancement, is a sensitive method for detecting early erosive inflammatory changes in the sacroiliac joints and spine. This patient most likely has ankylosing spondylitis, and MRI of the sacroiliac joints is most likely to establish a diagnosis. Radiographic evidence of sacroiliitis is required for definitive diagnosis and is the most consistent finding associated with this condition. Onset of ankylosing spondylitis usually occurs in the teenage years or 20s and manifests as persistent pain and morning stiffness involving the low back that are alleviated with activity. This condition also may be associated with tenderness of the pelvis. Ankylosing spondylitis in women may have a less aggressive disease course than in men. Typically, the earliest radiographic changes in affected patients involve the sacroiliac joints, but these changes may not be visible for several years; therefore, this patient's normal radiographs of the pelvis do not exclude sacroiliitis. MRI, especially with gadolinium enhancement, is considered a sensitive method for detecting early erosive inflammatory changes in the sacroiliac joints and spine and can assess sites of active disease and response to effective therapy. Anti-cyclic citrullinated peptide antibodies are highly specific for rheumatoid arthritis. However, rheumatoid arthritis does not involve the sacroiliac joints or lumbar spine, and testing for this condition in this patient is therefore not indicated. Increased radionuclide uptake of the sacroiliac joints on a bone scan would be consistent with but not specific for ankylosing spondylitis and would not confirm this diagnosis. Furthermore, assessing uptake of the sacroiliac joint on a bone scan may be difficult, because an increased signal usually exists in this site and differentiating normal uptake from abnormal uptake is not precise. An elevated erythrocyte sedimentation rate would raise suspicion for an inflammatory process but would not help to establish a specific diagnosis. In addition, the erythrocyte sedimentation rate does not correlate with disease activity in patients with ankylosing spondylitis, and measurement of this value is therefore not useful in diagnosing or monitoring patients with this condition. HLA-B27 positivity is a strong risk factor for ankylosing spondylitis. However, less than 5% of patients who have this allele develop this condition. In addition, not all patients who have ankylosing spondylitis have this allele. Therefore, it is neither 100% sensitive nor 100% specific for the diagnosis of ankylosing spondylitis.

Rheuma 51 A 67-year-old man comes for evaluation of knee pain. Two months ago, he developed pain in the right knee that worsened when he played tennis and was relieved with rest. He now has pain with most activities and occasionally at rest that is often associated with swelling. He has no morning stiffness. Maximum doses of acetaminophen provide only mild to moderate relief of pain. One year ago, he was diagnosed with coronary artery disease with a myocardial infarction and underwent intracoronary stent placement. He also has hypertension and hyperlipidemia. Current medications are atorvastatin, atenolol, isosorbide mononitrate, and low-dose aspirin. On physical examination, blood pressure is 130/80 mm Hg. Cardiac examination shows an S4, normal S1 and S2, and no murmurs or rubs. Range of motion of the right knee is painful and limited. The remainder of the musculoskeletal examination is normal. Radiograph of the right knee shows medial joint-space narrowing, subchondral sclerosis, and osteophytes. Which of the following is the most appropriate next therapeutic step for this patient's knee pain? A Arthroscopic lavage and débridement B Celecoxib C Ibuprofen D Total knee arthroplasty E Tramadol

E Tramadol is an appropriate second-line agent in the treatment of osteoarthritis and is particularly useful in patients with osteoarthritis of the knee who are intolerant of or who do not achieve adequate pain control using NSAIDs. This patient's clinical presentation is consistent with osteoarthritis, and the most appropriate next step in his treatment is to begin tramadol. Osteoarthritis is characterized by pain on activity that is relieved with rest. This condition also is associated with a painful, limited range of motion and the presence of joint-space narrowing, subchondral sclerosis, and osteophyte formation on radiography. Tramadol is an appropriate second-line agent in the treatment of osteoarthritis and is particularly useful in patients with osteoarthritis of the knee or hip who are intolerant of or who do not achieve adequate pain control using NSAIDs. Arthroscopic lavage and/or débridement is not indicated for osteoarthritis of the knee. A randomized controlled trial comparing arthroscopic lavage and arthroscopic lavage plus débridement compared with a sham procedure for knee osteoarthritis showed no benefit of the intervention compared with sham arthroscopy. Patients with hip and knee osteoarthritis who have not responded to conservative therapy and who have functional limitations should be referred to an orthopedic surgeon for consideration of arthroplasty. However, this patient is not severely incapacitated, and other medical options are available to manage his knee pain. Use of an NSAID such as celecoxib or ibuprofen would be appropriate in a patient with moderate to severe osteoarthritis or a patient with osteoarthritis in whom acetaminophen did not provide sufficient pain relief. However, both selective and nonselective NSAIDs are associated with an increased risk of cardiovascular disease and should be avoided in patients with cardiovascular conditions. Finally, coadministration of an NSAID with aspirin also may inhibit the antiplatelet effects of aspirin.

Rheuma 4 A 37-year-old man is evaluated for a 4-month history of progressively worsening red, scaly lesions involving the scalp, trunk, and extremities. During the past month, he also has had pain in the hands and feet and has lost 4.5 kg (10.0 lb). Aspirin and acetaminophen have helped to relieve his joint symptoms. He does not drink alcoholic beverages, has no other medical problems, and takes no additional medications. He denies allergies. He has sex with men and women and uses condoms inconsistently (widersprüchlich). On physical examination, vital signs are normal. The appearance of lesions found on the arms, hands, upper torso, back, inside the umbilicus, and legs is shown . There is swelling of the first and fourth digits on the left hand and of the feet. The left first metatarsophalangeal joint and left ankle are swollen and tender to palpation. Which of the following diagnostic studies should be performed next in this patient? A Anti-cyclic citrullinated peptide antibody assay B Antinuclear antibody assay C Hepatitis C virus antibody assay D HLA-B27 testing E Serologic testing for HIV antibodies

E Underlying HIV infection should be excluded in all patients with new-onset psoriasis or psoriatic arthritis, as well as in those with mild forms of these conditions who experience an explosion of their disease. This patient should be tested for HIV infection. He has extensive new-onset psoriasis accompanied by asymmetric peripheral arthritis that predominantly involves the small joints and dactylitis. This constellation of symptoms is consistent with psoriatic arthritis. Psoriasis, psoriatic arthritis, or reactive arthritis may be presenting signs of HIV infection or may appear after the diagnosis of HIV infection is established. Compared with psoriasis that develops in the general population, HIV-associated psoriasis is more likely to have an explosive onset and more severe disease course that may include nail changes, palmar and plantar involvement, and erythroderma. Underlying HIV infection should therefore be excluded in all patients with new-onset psoriasis or psoriatic arthritis, as well as in those with mild forms of these conditions who experience an explosion of their disease. Diagnosis of HIV infection is important for the purposes of identifying and managing a comorbidity but also because a patient's immune status may affect the selection of treatment for psoriasis; for example, immunosuppressive therapy would not be indicated in a patient with HIV infection. The presence of anti-cyclic citrullinated peptide antibodies is relatively specific for rheumatoid arthritis. However, rheumatoid arthritis is unlikely in a patient with an asymmetric pattern of joint disease and would not explain the temporal association between this patient's arthritis and new-onset psoriasis. Patients with systemic lupus erythematosus (SLE) and hepatitis C virus infection may have musculoskeletal and cutaneous manifestations. However, joint involvement in SLE, which is associated with antinuclear antibodies, typically manifests as symmetric arthritis that involves the small and large joints. In addition, patients with SLE may present with many types of cutaneous lesions, including photosensitivity, malar rash, subacute cutaneous lupus, and discoid lupus. However, none of these conditions resembles the erythematous plaque with a thick silvery scale that is characteristic of chronic plaque psoriasis. Joint involvement in patients with hepatitis C virus infection also is more likely to be symmetric and more commonly manifests as arthralgia instead of frank arthritis. Furthermore, skin disease in these patients usually consists of ulcerations and purpuric rash that is often limited to the lower extremities. The spondyloarthropathies are associated with an increased incidence of HLA-B27 compared with the general population. However, patients with psoriatic arthritis that predominantly manifests as peripheral joint involvement are only slightly more likely to have HLA-B27 than the general population. Furthermore, most HLA-B27-positive patients do not develop a spondyloarthropathy. This study therefore would not help to diagnose this patient's condition because it lacks both sensitivity and specificity.


संबंधित स्टडी सेट्स

Chapter 16: The Endocrine System

View Set

Immunization and Vaccine Training

View Set

High Speed Data Customer Service

View Set

BACTERIAL CELL WALL SYNTHESIS INHIBITORS/BACTERIAL PROTEIN SYNTHESIS INHIBITORS/INHIBITORS OF BACTERIAL NUCLEIC ACID SYNTHESIS OR FUNCTION/ANTIMYCOBACTERIAL DRUGS

View Set

Private insurance plan for seniors

View Set

Regular Chemistry Final (Ch: 8, 9, 10 & 11 Tests and Ch 12 quiz)

View Set